MedSurg2 Exam #1

अब Quizwiz के साथ अपने होमवर्क और परीक्षाओं को एस करें!

Identify five hormonees that are replaced when panhypopituitarism from radiation therapy or total hypophysectomy as treatment for pituitary tumors

1) cortisol 2) thyroid 3) vasopressin/ ADH analog 4) Growth hormone 5) sex hormones: testosterone, follicle stimulating hormone (FSH), and leutinizing hormone (LH) if fertility is desired; estrogen/progesterone if fertility is not an issue

A 22-year-old patient is being seen in the clinic with increased secretion of the anterior pituitary hormones. The nurse would expect the laboratory results to show a. increased urinary cortisol. b. decreased serum thyroxine. c. elevated serum aldosterone levels. d. low urinary catecholamines excretion.

A

The nurse is caring for a 63-year-old with a possible pituitary tumor who is scheduled for a computed tomography (CT) scan with contrast. Which information about the patient is most important to discuss with the health care provider before the test? a. History of renal insufficiency b. Complains of chronic headache c. Recent bilateral visual field loss d. Blood glucose level of 134 mg/dL

A

A 28-year-old pregnant woman is spilling sugar in her urine. The physician orders a glucose tolerance test, which reveals gestational diabetes. The patient is shocked by the diagnosis, stating that she is conscientious about her health, and asks the nurse what causes gestational diabetes. The nurse should explain that gestational diabetes is a result of what etiologic factor? A) Increased caloric intake during the first trimester B) Changes in osmolality and fluid balance C) The effects of hormonal changes during pregnancy D) Overconsumption of carbohydrates during the first two trimesters

A 28-year-old pregnant woman is spilling sugar in her urine. The physician orders a glucose tolerance test, which reveals gestational diabetes. The patient is shocked by the diagnosis, stating that she is conscientious about her health, and asks the nurse what causes gestational diabetes. The nurse should explain that gestational diabetes is a result of what etiologic factor? A) Increased caloric intake during the first trimester B) Changes in osmolality and fluid balance C) The effects of hormonal changes during pregnancy D) Overconsumption of carbohydrates during the first two trimesters

Which statements will the nurse include when teaching a patient who is scheduled for oral glucose tolerance testing in the outpatient clinic (select all that apply)? a. "You will need to avoid smoking before the test." b. "Exercise should be avoided until the testing is complete." c. "Several blood samples will be obtained during the testing." d. "You should follow a low-calorie diet the day before the test." e. "The test requires that you fast for at least 8 hours before testing."

A C E

A patient who has stage IIB breast cancer has a left mastectomy with axillary node dissection. Ten lymph nodes are resected and three contain malignant cells. What is teh best goal for the nurse to use in planning care? A Assess the pt's response to pathology report B. Maintain joint flexibility and l. arm function B. Prevent L. arm lymphatic fluid accumulation D. Prepare her for chemo and radiation therapy

A. Because the nurse acts as the pt. advocate and because this time is an extremely stressful period for the patient and family, the nurses initial plan focuses on the patients response to the pathology report as part of a comprhensive patient assessment.

What is the nurse's priority assessment in a 41 year old female paitent who reports a lump in the upper, outer margin of the right breast? A. Related pain B. Family history C. Onset of mass D. Characteristics

A. Breast cancer is usually associated with nonpainful and nontender lesions of the breast, so this information is very important in assessing breast masses

What should the nurse include in patient teaching when preparing a 45 year old patient for a routine mammography? A. Additional xrays can be needed for dense breast tissue B. Ultrasound confirms the presence of nonvisible mass C. Early detection of malignany tumore prevents mettastasis D. Continue to obtain clinical breast examinations every 3 years

A. Dense breast tissue on mammography can appear as a suspicious region of the breast. Once identified, additional mammograms can target the region for further examination

Two years after breast conserving surgery for a malignany left breast tumor, the nurse assesses the patient. Which patient assessment data is a priority? A. Irregular, fixed mass i the left axilla B. Temperature, HR, lung sounds C. Left upper extremity persistent edema D. Keloid formation around wound edges

A. The most frequent site of breast cancer recurrence is the site of the original tumor, so the presence of an irregular and fixed mass in the L. axilla is the nurse priority. Fixed and irregular are characteristics of a malignant breast tumor and require follow up care

The nurse determines that additional instruction is needed for a 60-year-old patient with chronic syndrome of inappropriate antidiuretic hormone (SIADH) when the patient says which of the following?

A. "I need to shop for foods low in sodium and avoid adding salt to food." b. "I should weigh myself daily and report any sudden weight loss or gain." c. "I need to limit my fluid intake to no more than 1 quart of liquids a day." d. "I will eat foods high in potassium because diuretics cause potassium loss." ANSWER A

After a vasectomy, what teaching should be included in the discharge teaching? A. "You will want to use an alternative form of contraception for 6 weeks." B. "You may lose some secondary sexual characteristics after this surgery." C. "You may have erectile dysfunction for several months after this surgery." D. "You will be uncomfortable, but you may safely have sexual intercourse today."

A. "You will want to use an alternative form of contraception for 6 weeks." As vasectomies are usually done for sterilization purposes, to safely have sexual intercourse, the patient will need to use an alternative form of contraception until semen examination reveals no sperm, usually at least 10 ejaculations or 6 weeks to evacuate sperm distal to the surgical site. Hormones are not affected, so there is no loss of secondary sexual characteristics or erectile function. Most men experience too much pain to have sexual intercourse on the day of their surgery, so this is not an appropriate comment by the nurse.

The nurse is teaching clinic patients about risk factors for testicular cancer. Which individual is at highest risk for developing testicular cancer? A. A 30-year-old white male with a history of cryptorchidism B. A 48-year-old African American male with erectile dysfunction C. A 19-year-old Asian male who had surgery for testicular torsion D. A 28-year-old Hispanic male with infertility caused by a varicocele

A. A 30-year-old white male with a history of cryptorchidism The incidence of testicular cancer is four times higher in white males than in African American males. Testicular tumors are also more common in males who have had undescended testes (cryptorchidism) or a family history of testicular cancer or anomalies. Other predisposing factors include orchitis, human immunodeficiency virus infection, maternal exposure to DES, and testicular cancer in the contralateral testis.

The nurse is caring for a 62-year-old man after a transurethral resection of the prostate (TURP). Which instructions should the nurse include in the teaching plan? A. Avoid straining during defecation. B. Restrict fluids to prevent incontinence. C. Sexual functioning will not be affected. D. Prostate exams are not needed after surgery.

A. Avoid straining during defecation. Activities that increase abdominal pressure, such as sitting or walking for prolonged periods and straining to have a bowel movement (Valsalva maneuver), should be avoided in the postoperative recovery period to prevent a postoperative hemorrhage. Instruct the patient to drink at least 2 L of fluid every day. Digital rectal examinations should be performed yearly. The prostate gland is not totally removed and may enlarge after a TURP. Sexual functioning may change after prostate surgery. Changes may include retrograde ejaculation, erectile dysfunction, and decreased orgasmic sensation.

A pt. who had breast cancer is now having breast reconstructive surgery. To enhance the quality of the reconstructive surgery, what is the nurse's priority patient assessment when the patient returns for enlargement of the expander? A. Color of the affected skin B. Attitiude on reconstruction C. Desired size of new breast D. Tenderness near expander

A. Color of the affected skin. The nurses priority assessment is the color of the skin over and surrounding the expander because abnormal skin color can indicate poor perfusion to the region and risk the viability to the tissue. This is the nurses priority because the purpose of teh expander is to stretch the patiesn skin that will eventually cover the breast implant. The skin over the expander should be pink, warm, and supple. Skin with poor perfusion can look dusky and pale and be cool and friable.

Which of the following factors are poor prognostic indicators of breast cancer (select all that apply)? A. Previous breast cancer B. Well-differentiated tumor C. High incidence of aneuploidy D. Estrogen and progesterone receptor positive E. Many cells in synthesis phase of cell cycle

A. History of previous breast cancer increases the risk of recurrence. Tumors that are well differentiated tend to be less affressive than undifferentiated tumors. Tumors unreponsive to hormonal therapy tend to be more aggressive

A patient who was admitted with myxedema coma and diagnosed with hypothyroidism is improving and expected to be discharged in 2 days. Which teaching strategy will be best for the nurse to use?

A. Provide written reminders of self-care information. b. Offer multiple options for management of therapies. c. Ensure privacy for teaching by asking visitors to leave. d. Delay teaching until patient discharge date is confirmed. ANSWER A

A 71-year-old patient with a diagnosis of benign prostatic hyperplasia (BPH) has been scheduled for a contact laser technique. What is the primary goal of this intervention? A. Resumption of normal urinary drainage B. Maintenance of normal sexual functioning C. Prevention of acute or chronic renal failure D. Prevention of fluid and electrolyte imbalances

A. Resumption of normal urinary drainage The most significant signs and symptoms of BPH relate to the disruption of normal urinary drainage and consequent urine retention, incontinence, and pain. A laser technique vaporizes prostate tissue and cauterizes blood vessels and is used as an effective alternative to a TURP to resolve these problems. Fluid imbalances, sexual functioning, and kidney disease may result from uncontrolled BPH, but the central focus remains urinary drainage.

A 33-year-old patient noticed a painless lump in his scrotum on self-examination of his testicles and a feeling of heaviness. The nurse should first teach him about what diagnostic test? A. Ultrasound B. Cremasteric reflex C. Doppler ultrasound D. Transillumination with a flashlight

A. Ultrasound When the scrotum has a painless lump, scrotal swelling, and a feeling of heaviness, testicular cancer is suspected, and an ultrasound of the testes is indicated. Blood tests will also be done. The cremasteric reflex and Doppler ultrasound are done to diagnose testicular torsion. Transillumination with a flashlight is done to diagnose a hydrocele.

The nurse is volunteering at a community center to teach women regarding breast cancer. The nurse would include which of the following when discussing risk factors (select all that apply)? A. Nulliparity B. Age 30 or over C. Early menarche D. Late menopause E. Personal history of colon cancer

ACDE. Women are at an increased risk for development of breast cancer if they are over the age of 50; have a family history of breast cancer; have a personal history of breast, colon, endometrial, or ovarian cancer; have a long menstrual history as seen with early menarche or late menopause; and have had a first full-term pregnancy after the age of 30 or are nulliparous.

Which question during the assessment of a diabetic patient will help the nurse identify autonomic neuropathy? a. "Do you feel bloated after eating?" b. "Have you seen any skin changes?" c. "Do you need to increase your insulin dosage when you are stressed?" d. "Have you noticed any painful new ulcerations or sores on your feet?"

ANS: A Autonomic neuropathy can cause delayed gastric emptying, which results in a bloated feeling for the patient. The other questions are also appropriate to ask but would not help in identifying autonomic neuropathy.

In order to assist an older diabetic patient to engage in moderate daily exercise, which action is most important for the nurse to take? a. Determine what type of activities the patient enjoys. b. Remind the patient that exercise will improve self-esteem. c. Teach the patient about the effects of exercise on glucose level. d. Give the patient a list of activities that are moderate in intensity.

ANS: A Because consistency with exercise is important, assessment for the types of exercise that the patient finds enjoyable is the most important action by the nurse in ensuring adherence to an exercise program. The other actions will also be implemented but are not the most important in improving compliance.

Which nursing action can the nurse delegate to unlicensed assistive personnel (UAP) who are working in the diabetic clinic? a. Measure the ankle-brachial index. b. Check for changes in skin pigmentation. c. Assess for unilateral or bilateral foot drop. d. Ask the patient about symptoms of depression.

ANS: A Checking systolic pressure at the ankle and brachial areas and calculating the ankle-brachial index is a procedure that can be done by UAP who have been trained in the procedure. The other assessments require more education and critical thinking and should be done by the registered nurse (RN).

Which patient action indicates a good understanding of the nurse's teaching about the use of an insulin pump? a. The patient programs the pump for an insulin bolus after eating. b. The patient changes the location of the insertion site every week. c. The patient takes the pump off at bedtime and starts it again each morning. d. The patient plans for a diet that is less flexible when using the insulin pump

ANS: A In addition to the basal rate of insulin infusion, the patient will adjust the pump to administer a bolus after each meal, with the dosage depending on the oral intake. The insertion site should be changed every 2 or 3 days. There is more flexibility in diet and exercise when an insulin pump is used. The pump will deliver a basal insulin rate 24 hours a day.

A female patient is scheduled for an oral glucose tolerance test. Which information from the patient's health history is most important for the nurse to communicate to the health care provider? a. The patient uses oral contraceptives. b. The patient runs several days a week. c. The patient has been pregnant three times. d. The patient has a family history of diabetes.

ANS: A Oral contraceptive use may falsely elevate oral glucose tolerance test (OGTT) values. Exercise and a family history of diabetes both can affect blood glucose but will not lead to misleading information from the OGTT. History of previous pregnancies may provide informational about gestational glucose tolerance, but will not lead to misleading information from the OGTT.

A 32-year-old patient with diabetes is starting on intensive insulin therapy. Which type of insulin will the nurse discuss using for mealtime coverage? a. Lispro (Humalog) b. Glargine (Lantus) c. Detemir (Levemir) d. NPH (Humulin N)

ANS: A Rapid- or short-acting insulin is used for mealtime coverage for patients receiving intensive insulin therapy. NPH, glargine, or detemir will be used as the basal insulin.

Which statement by the patient indicates a need for additional instruction in administering insulin? a. "I need to rotate injection sites among my arms, legs, and abdomen each day." b. "I can buy the 0.5 mL syringes because the line markings will be easier to see." c. "I should draw up the regular insulin first after injecting air into the NPH bottle." d. "I do not need to aspirate the plunger to check for blood before injecting insulin."

ANS: A Rotating sites is no longer recommended because there is more consistent insulin absorption when the same site is used consistently. The other patient statements are accurate and indicate that no additional instruction is needed.

The nurse has administered 4 oz of orange juice to an alert patient whose blood glucose was 62 mg/dL. Fifteen minutes later, the blood glucose is 67 mg/dL. Which action should the nurse take next? a. Give the patient 4 to 6 oz more orange juice. b. Administer the PRN glucagon (Glucagon) 1 mg IM. c. Have the patient eat some peanut butter with crackers. d. Notify the health care provider about the hypoglycemia.

ANS: A The "rule of 15" indicates that administration of quickly acting carbohydrates should be done 2 to 3 times for a conscious patient whose glucose remains less than 70 mg/dL before notifying the health care provider. More complex carbohydrates and fats may be used once the glucose has stabilized. Glucagon should be used if the patient's level of consciousness decreases so that oral carbohydrates can no longer be given.

A 38-year-old patient who has type 1 diabetes plans to swim laps daily at 1:00 PM. The clinic nurse will plan to teach the patient to a. check glucose level before, during, and after swimming. b. delay eating the noon meal until after the swimming class. c. increase the morning dose of neutral protamine Hagedorn (NPH) insulin. d. time the morning insulin injection so that the peak occurs while swimming.

ANS: A The change in exercise will affect blood glucose, and the patient will need to monitor glucose carefully to determine the need for changes in diet and insulin administration. Because exercise tends to decrease blood glucose, patients are advised to eat before exercising. Increasing the morning NPH or timing the insulin to peak during exercise may lead to hypoglycemia, especially with the increased exercise.

A 55-year-old female patient with type 2 diabetes has a nursing diagnosis of imbalanced nutrition: more than body requirements. Which goal is most important for this patient? a. The patient will reach a glycosylated hemoglobin level of less than 7%. b. The patient will follow a diet and exercise plan that results in weight loss. c. The patient will choose a diet that distributes calories throughout the day. d. The patient will state the reasons for eliminating simple sugars in the diet.

ANS: A The complications of diabetes are related to elevated blood glucose, and the most important patient outcome is the reduction of glucose to near-normal levels. The other outcomes also are appropriate but are not as high in priority.

A 54-year-old patient is admitted with diabetic ketoacidosis. Which admission order should the nurse implement first? a. Infuse 1 liter of normal saline per hour. b. Give sodium bicarbonate 50 mEq IV push. c. Administer regular insulin 10 U by IV push. d. Start a regular insulin infusion at 0.1 units/kg/hr.

ANS: A The most urgent patient problem is the hypovolemia associated with diabetic ketoacidosis (DKA), and the priority is to infuse IV fluids. The other actions can be done after the infusion of normal saline is initiated.

Which information will the nurse include in teaching a female patient who has peripheral arterial disease, type 2 diabetes, and sensory neuropathy of the feet and legs? a. Choose flat-soled leather shoes. b. Set heating pads on a low temperature. c. Use callus remover for corns or calluses. d. Soak feet in warm water for an hour each day.

ANS: A The patient is taught to avoid high heels and that leather shoes are preferred. The feet should be washed, but not soaked, in warm water daily. Heating pad use should be avoided. Commercial callus and corn removers should be avoided. The patient should see a specialist to treat these problems

A patient receives aspart (NovoLog) insulin at 8:00 AM. Which time will it be most important for the nurse to monitor for symptoms of hypoglycemia? a. 10:00 AM b. 12:00 AM c. 2:00 PM d. 4:00 PM

ANS: A The rapid-acting insulins peak in 1 to 3 hours. The patient is not at a high risk for hypoglycemia at the other listed times, although hypoglycemia may occur.

A 56-year-old woman is concerned about having a moderate amount of vaginal bleeding after 4 years of menopause. The nurse will anticipate teaching the patient about a. endometrial biopsy. b. endometrial ablation. c. uterine balloon therapy. d. dilation and curettage (D&C).

ANS: A A postmenopausal woman with vaginal bleeding should be evaluated for endometrial cancer, and endometrial biopsy is the primary test for endometrial cancer. D&C will be needed only if the biopsy does not provide sufficient information to make a diagnosis. Endometrial ablation and balloon therapy are used to treat menorrhagia, which is unlikely in this patient

Which prescribed medication should the nurse administer first to a 60-year-old patient admitted to the emergency department in thyroid storm? a. Propranolol (Inderal) b. Propylthiouracil (PTU) c. Methimazole (Tapazole) d. Iodine (Lugol's solution)

ANS: A Adrenergic blockers work rapidly to decrease the cardiovascular manifestations of thyroid storm. The other medications take days to weeks to have an impact on thyroid function.

Which finding for a patient who has hypothyroidism and hypertension indicates that the nurse should contact the health care provider before administering levothyroxine (Synthroid)? a. Increased thyroxine (T4) level b. Blood pressure 112/62 mm Hg c. Distant and difficult to hear heart sounds d. Elevated thyroid stimulating hormone level

ANS: A An increased thyroxine level indicates the levothyroxine dose needs to be decreased. The other data are consistent with hypothyroidism and the nurse should administer the levothyroxine.

Which finding by the nurse when assessing a patient with Hashimoto's thyroiditis and a goiter will require the most immediate action? a. New-onset changes in the patient's voice b. Apical pulse rate at rest 112 beats/minute c. Elevation in the patient's T3 and T4 levels d. Bruit audible bilaterally over the thyroid gland

ANS: A Changes in the patient's voice indicate that the goiter is compressing the laryngeal nerve and may lead to airway compression. The other findings will also be reported but are expected with Hashimoto's thyroiditis and do not require immediate action.

Which statement by the patient indicates that the nurse's teaching about treating vaginal candidiasis has been effective? a. "I should clean carefully after each urination and bowel movement." b. "I can douche with warm water if the itching continues to bother me." c. "I will insert the antifungal cream right before I get up in the morning." d. "I will tell my husband that we cannot have intercourse for the next month."

ANS: A Cleaning of the perineal area will decrease itching caused by contact of the irritated tissues with urine and reduce the chance of further infection of irritated tissues by bacteria in the stool. Sexual intercourse should be avoided for 1 week. Douching will disrupt normal protective mechanisms in the vagina. The cream should be used at night so that it will remain in the vagina for longer periods of time

Which finding indicates to the nurse that the current therapies are effective for a patient with acute adrenal insufficiency? a. Increasing serum sodium levels b. Decreasing blood glucose levels c. Decreasing serum chloride levels d. Increasing serum potassium levels

ANS: A Clinical manifestations of Addison's disease include hyponatremia and an increase in sodium level indicates improvement. The other values indicate that treatment has not been effective.

A 28-year-old patient with endometriosis asks why she is being treated with medroxyprogesterone (Depo-Provera), a medication that she thought was an oral contraceptive. The nurse explains that this therapy a. suppresses the menstrual cycle by mimicking pregnancy. b. will relieve symptoms such as vaginal atrophy and hot flashes. c. prevents a pregnancy that could worsen the menstrual bleeding. d. will lead to permanent suppression of abnormal endometrial tissues.

ANS: A Depo-Provera induces a pseudopregnancy, which suppresses ovulation and causes shrinkage of endometrial tissue. Menstrual bleeding does not occur during pregnancy. Vaginal atrophy and hot flashes are caused by synthetic androgens such as danazol or gonadotropin-releasing hormone agonists (GNRH) such as leuprolide. Although hormonal therapies will control endometriosis while the therapy is used, endometriosis will recur once the menstrual cycle is reestablished

The nurse will plan to teach the female patient with genital warts about the a. importance of regular Pap tests. b. increased risk for endometrial cancer. c. appropriate use of oral contraceptives. d. symptoms of pelvic inflammatory disease (PID).

ANS: A Genital warts are caused by the human papillomavirus (HPV) and increase the risk for cervical cancer. There is no indication that the patient needs teaching about PID, oral contraceptives, or endometrial cancer

A 56-year-old female patient has an adrenocortical adenoma, causing hyperaldosteronism. The nurse providing care should a. monitor the blood pressure every 4 hours. b. elevate the patient's legs to relieve edema. c. monitor blood glucose level every 4 hours. d. order the patient a potassium-restricted diet.

ANS: A Hypertension caused by sodium retention is a common complication of hyperaldosteronism. Hyperaldosteronism does not cause an elevation in blood glucose. The patient will be hypokalemic and require potassium supplementation before surgery. Edema does not usually occur with hyperaldosteronism.

A 50-year-old patient is diagnosed with uterine bleeding caused by a leiomyoma. Which information will the nurse include in the patient teaching plan? a. The symptoms may decrease after the patient undergoes menopause. b. The tumor size is likely to increase throughout the patient's lifetime. c. Aspirin or acetaminophen may be used to control mild to moderate pain. d. The patient will need frequent monitoring to detect any malignant changes.

ANS: A Leiomyomas appear to depend on ovarian hormones and will atrophy after menopause, leading to a decrease in symptoms. Aspirin use is discouraged because the antiplatelet effects may lead to heavier uterine bleeding. The size of the tumor will shrink after menopause. Leiomyomas are benign tumors that do not undergo malignant changes.

After scheduling a patient with a possible ovarian cyst for ultrasound, the nurse will teach the patient that she should a. expect to receive IV contrast during the procedure. b. drink several glasses of fluids before the procedure. c. experience mild abdominal cramps after the procedure. d. discontinue taking aspirin for 7 days before the procedure.

ANS: B A full bladder is needed for many ultrasound procedures, so the nurse will have the patient drink fluids before arriving for the ultrasound. The other instructions are not accurate for this procedure.

The nurse determines that additional instruction is needed for a 60-year-old patient with chronic syndrome of inappropriate antidiuretic hormone (SIADH) when the patient says which of the following? a. "I need to shop for foods low in sodium and avoid adding salt to food." b. "I should weigh myself daily and report any sudden weight loss or gain." c. "I need to limit my fluid intake to no more than 1 quart of liquids a day." d. "I will eat foods high in potassium because diuretics cause potassium loss."

ANS: A Patients with SIADH are at risk for hyponatremia, and a sodium supplement may be prescribed. The other patient statements are correct and indicate successful teaching has occurred.

A 22-year-old tells the nurse that she has not had a menstrual period for the last 2 months. Which action is most important for the nurse to take? a. Obtain a urine specimen for a pregnancy test. b. Ask about any recent stressful lifestyle changes. c. Measure the patient's current height and weight. d. Question the patient about prescribed medications.

ANS: A Pregnancy should always be considered a possible cause of amenorrhea in women of childbearing age. The other actions are also appropriate, but it is important to check for pregnancy in this patient because pregnancy will require rapid implementation of actions to promote normal fetal development such as changes in lifestyle, folic acid intake, etc.

A 19-year-old visits the health clinic for a routine checkup. Which question should the nurse ask to determine whether a Pap test is needed? a. "Have you had sexual intercourse?" b. "Do you use any illegal substances?" c. "Do you have cramping with your periods?" d. "At what age did your menstrual periods start?"

ANS: A The current American Cancer Society recommendation is that a Pap test be done every 3 years, starting 3 years after the first sexual intercourse and no later than age 21. The information about menstrual periods and substance abuse will not help determine whether the patient requires a Pap test

A 54-year-old patient is on the surgical unit after a radical abdominal hysterectomy. Which finding is most important to report to the health care provider? a. Urine output of 125 mL in the first 8 hours after surgery b. Decreased bowel sounds in all four abdominal quadrants c. One-inch area of bloody drainage on the abdominal dressing d. Complaints of abdominal pain at the incision site with coughing

ANS: A The decreased urine output indicates possible low blood volume and further assessment is needed to assess for possible internal bleeding. Decreased bowel sounds, minor drainage on the dressing, and abdominal pain with coughing are expected after this surgery

The nurse is caring for a patient admitted with diabetes insipidus (DI). Which information is most important to report to the health care provider? a. The patient is confused and lethargic. b. The patient reports a recent head injury. c. The patient has a urine output of 400 mL/hr. d. The patient's urine specific gravity is 1.003.

ANS: A The patient's confusion and lethargy may indicate hypernatremia and should be addressed quickly. In addition, patients with DI compensate for fluid losses by drinking copious amounts of fluids, but a patient who is lethargic will be unable to drink enough fluids and will become hypovolemic. A high urine output, low urine specific gravity, and history of a recent head injury are consistent with diabetes insipidus, but they do not require immediate nursing action to avoid life-threatening complications.

The nurse is caring for a 45-year-old male patient during a water deprivation test. Which finding is most important for the nurse to communicate to the health care provider? a. The patient complains of intense thirst. b. The patient has a 5-lb (2.3 kg) weight loss. c. The patient's urine osmolality does not increase. d. The patient feels dizzy when sitting on the edge of the bed.

B

A 28-year-old patient reports anxiety, headaches with dizziness, and abdominal bloating occurring before her menstrual periods. Which action is best for the nurse to take at this time? a. Ask the patient to keep track of her symptoms in a diary for 3 months. b. Suggest that the patient try aerobic exercise to decrease her symptoms. c. Teach the patient about appropriate lifestyle changes to reduce premenstrual syndrome (PMS) symptoms. d. Advise the patient to use nonsteroidal antiinflammatory drugs (NSAIDs) such as ibuprofen (Advil) to control symptoms.

ANS: A The patient's symptoms indicate possible PMS, but they also may be associated with other diagnoses. Having the patient keep a symptom diary for 2 or 3 months will help in confirming a diagnosis of PMS. The nurse should not implement interventions for PMS until a diagnosis is made

The nurse is assessing the sexual-reproductive functional health pattern of a 32-year-old woman. Which question is most useful in determining the patient's sexual orientation and risk factors? a. "Do you have sex with men, women, or both?" b. "Which gender do you prefer to have sex with?" c. "What types of sexual activities do you prefer?" d. "Are you heterosexual, homosexual, or bisexual?"

ANS: A This question is the most simply stated and will increase the likelihood of obtaining the relevant information about sexual orientation and possible risk factors associated with sexual activity. A patient who prefers sex with women may also have intercourse at times with men. The types of sexual activities engaged in may not indicate sexual orientation. Many patients who have sex with both men and women do not identify themselves as homosexual or bisexual.

Which topic will the nurse include in the preoperative teaching for a patient admitted for an abdominal hysterectomy? a. Purpose of ambulation and leg exercises b. Adverse effects of systemic chemotherapy c. Decrease in vaginal sensation after surgery d. Symptoms caused by the drop in estrogen level

ANS: A Venous thromboembolism (VTE) is a potential complication after the surgery, and the nurse will instruct the patient about ways to prevent it. Vaginal sensation is decreased after a vaginal hysterectomy but not after abdominal hysterectomy. Leiomyomas are benign tumors, so chemotherapy and radiation will not be prescribed. Because the patient will still have her ovaries, the estrogen level will not decrease

A patient who was admitted with myxedema coma and diagnosed with hypothyroidism is improving and expected to be discharged in 2 days. Which teaching strategy will be best for the nurse to use? a. Provide written reminders of self-care information. b. Offer multiple options for management of therapies. c. Ensure privacy for teaching by asking visitors to leave. d. Delay teaching until patient discharge date is confirmed.

ANS: A Written instructions will be helpful to the patient because initially the hypothyroid patient may be unable to remember to take medications and other aspects of self-care. Because the treatment regimen is somewhat complex, teaching should be initiated well before discharge. Family members or friends should be included in teaching because the hypothyroid patient is likely to forget some aspects of the treatment plan. A simpler regimen will be easier to understand until the patient is euthyroid.

Which nonhormonal therapies will the nurse suggest for a healthy perimenopausal woman who prefers not to use hormone therapy (HT) (select all that apply)? a. Reduce coffee intake. b. Exercise several times a week. c. Take black cohosh supplements. d. Have a glass of wine in the evening. e. Increase intake of dietary soy products.

ANS: A, B, C, E Reduction in caffeine intake, use of black cohosh, increasing dietary soy intake, and exercising three to four times weekly are recommended to reduce symptoms associated with menopause. Alcohol intake in the evening may increase the sleep problems associated with menopause

In which order will the nurse take these steps to prepare NPH 20 units and regular insulin 2 units using the same syringe? (Put a comma and a space between each answer choice [A, B, C, D, E]). a. Rotate NPH vial. b. Withdraw regular insulin. c. Withdraw 20 units of NPH. d. Inject 20 units of air into NPH vial. e. Inject 2 units of air into regular insulin vial.

ANS: A, D, E, B, C When mixing regular insulin with NPH, it is important to avoid contact between the regular insulin and the additives in the NPH that slow the onset, peak, and duration of activity in the longer-acting insulin.

A 34-year-old has a new diagnosis of type 2 diabetes. The nurse will discuss the need to schedule a dilated eye exam a. every 2 years. b. as soon as possible. c. when the patient is 39 years old. d. within the first year after diagnosis.

ANS: B Because many patients have some diabetic retinopathy when they are first diagnosed with type 2 diabetes, a dilated eye exam is recommended at the time of diagnosis and annually thereafter. Patients with type 1 diabetes should have dilated eye exams starting 5 years after they are diagnosed and then annually.

After change-of-shift report, which patient should the nurse assess first? a. 19-year-old with type 1 diabetes who has a hemoglobin A1C of 12% b. 23-year-old with type 1 diabetes who has a blood glucose of 40 mg/dL c. 40-year-old who is pregnant and whose oral glucose tolerance test is 202 mg/dL d. 50-year-old who uses exenatide (Byetta) and is complaining of acute abdominal pain

ANS: B Because the brain requires glucose to function, untreated hypoglycemia can cause unconsciousness, seizures, and death. The nurse will rapidly assess and treat the patient with low blood glucose. The other patients also have symptoms that require assessments and/or interventions, but they are not at immediate risk for life-threatening complications.

The nurse is preparing to teach a 43-year-old man who is newly diagnosed with type 2 diabetes about home management of the disease. Which action should the nurse take first? a. Ask the patient's family to participate in the diabetes education program. b. Assess the patient's perception of what it means to have diabetes mellitus. c. Demonstrate how to check glucose using capillary blood glucose monitoring. d. Discuss the need for the patient to actively participate in diabetes management.

ANS: B Before planning teaching, the nurse should assess the patient's interest in and ability to self-manage the diabetes. After assessing the patient, the other nursing actions may be appropriate, but planning needs to be individualized to each patient.

Which patient action indicates good understanding of the nurse's teaching about administration of aspart (NovoLog) insulin? a. The patient avoids injecting the insulin into the upper abdominal area. b. The patient cleans the skin with soap and water before insulin administration. c. The patient stores the insulin in the freezer after administering the prescribed dose. d. The patient pushes the plunger down while removing the syringe from the injection site.

ANS: B Cleaning the skin with soap and water or with alcohol is acceptable. Insulin should not be frozen. The patient should leave the syringe in place for about 5 seconds after injection to be sure that all the insulin has been injected. The upper abdominal area is one of the preferred areas for insulin injection.

An unresponsive patient with type 2 diabetes is brought to the emergency department and diagnosed with hyperosmolar hyperglycemic syndrome (HHS). The nurse will anticipate the need to a. give a bolus of 50% dextrose. b. insert a large-bore IV catheter. c. initiate oxygen by nasal cannula. d. administer glargine (Lantus) insulin.

ANS: B HHS is initially treated with large volumes of IV fluids to correct hypovolemia. Regular insulin is administered, not a long-acting insulin. There is no indication that the patient requires oxygen. Dextrose solutions will increase the patient's blood glucose and would be contraindicated.

The nurse is taking a health history from a 29-year-old pregnant patient at the first prenatal visit. The patient reports no personal history of diabetes but has a parent who is diabetic. Which action will the nurse plan to take first? a. Teach the patient about administering regular insulin. b. Schedule the patient for a fasting blood glucose level. c. Discuss an oral glucose tolerance test for the twenty-fourth week of pregnancy. d. Provide teaching about an increased risk for fetal problems with gestational diabetes.

ANS: B Patients at high risk for gestational diabetes should be screened for diabetes on the initial prenatal visit. An oral glucose tolerance test may also be used to check for diabetes, but it would be done before the twenty-fourth week. The other actions may also be needed (depending on whether the patient develops gestational diabetes), but they are not the first actions that the nurse should take.

Which action should the nurse take after a 36-year-old patient treated with intramuscular glucagon for hypoglycemia regains consciousness? a. Assess the patient for symptoms of hyperglycemia. b. Give the patient a snack of peanut butter and crackers. c. Have the patient drink a glass of orange juice or nonfat milk. d. Administer a continuous infusion of 5% dextrose for 24 hours.

ANS: B Rebound hypoglycemia can occur after glucagon administration, but having a meal containing complex carbohydrates plus protein and fat will help prevent hypoglycemia. Orange juice and nonfat milk will elevate blood glucose rapidly, but the cheese and crackers will stabilize blood glucose. Administration of IV glucose might be used in patients who were unable to take in nutrition orally. The patient should be assessed for symptoms of hypoglycemia after glucagon administration.

Which laboratory value reported to the nurse by the unlicensed assistive personnel (UAP) indicates the most urgent need for the nurse's assessment of the patient? a. Bedtime glucose of 140 mg/dL b. Noon blood glucose of 52 mg/dL c. Fasting blood glucose of 130 mg/dL d. 2-hr postprandial glucose of 220 mg/dL

ANS: B The nurse should assess the patient with a blood glucose level of 52 mg/dL for symptoms of hypoglycemia and give the patient a carbohydrate-containing beverage such as orange juice. The other values are within an acceptable range or not immediately dangerous for a diabetic patient.

The nurse identifies a need for additional teaching when the patient who is self-monitoring blood glucose a. washes the puncture site using warm water and soap. b. chooses a puncture site in the center of the finger pad. c. hangs the arm down for a minute before puncturing the site. d. says the result of 120 mg indicates good blood sugar control.

ANS: B The patient is taught to choose a puncture site at the side of the finger pad because there are fewer nerve endings along the side of the finger pad. The other patient actions indicate that teaching has been effective.

Which information will the nurse include when teaching a 50-year-old patient who has type 2 diabetes about glyburide (Micronase, DiaBeta, Glynase)? a. Glyburide decreases glucagon secretion from the pancreas. b. Glyburide stimulates insulin production and release from the pancreas. c. Glyburide should be taken even if the morning blood glucose level is low. d. Glyburide should not be used for 48 hours after receiving IV contrast media.

ANS: B The sulfonylureas stimulate the production and release of insulin from the pancreas. If the glucose level is low, the patient should contact the health care provider before taking the glyburide, because hypoglycemia can occur with this class of medication. Metformin should be held for 48 hours after administration of IV contrast media, but this is not necessary for glyburide. Glucagon secretion is not affected by glyburide.

An active 28-year-old male with type 1 diabetes is being seen in the endocrine clinic. Which finding may indicate the need for a change in therapy? a. Hemoglobin A1C level 6.2% b. Blood pressure 146/88 mmHg c. Heart rate at rest 58 beats/minute d. High density lipoprotein (HDL) level 65 mg/dL

ANS: B To decrease the incidence of macrovascular and microvascular problems in patients with diabetes, the goal blood pressure is usually 130/80. An A1C less than 6.5%, a low resting heart rate (consistent with regular aerobic exercise in a young adult), and an HDL level of 65 mg/dL all indicate that the patient's diabetes and risk factors for vascular disease are well controlled.

A diabetic patient who has reported burning foot pain at night receives a new prescription. Which information should the nurse teach the patient about amitriptyline (Elavil)? a. Amitriptyline decreases the depression caused by your foot pain. b. Amitriptyline helps prevent transmission of pain impulses to the brain. c. Amitriptyline corrects some of the blood vessel changes that cause pain. d. Amitriptyline improves sleep and makes you less aware of nighttime pain.

ANS: B Tricyclic antidepressants decrease the transmission of pain impulses to the spinal cord and brain. Tricyclic antidepressants also improve sleep quality and are used for depression, but that is not the major purpose for their use in diabetic neuropathy. The blood vessel changes that contribute to neuropathy are not affected by tricyclic antidepressants.

The nurse will plan to monitor a patient diagnosed with a pheochromocytoma for a. flushing. b. headache. c. bradycardia. d. hypoglycemia.

ANS: B The classic clinical manifestations of pheochromocytoma are hypertension, tachycardia, severe headache, diaphoresis, and abdominal or chest pain. Elevated blood glucose may also occur because of sympathetic nervous system stimulation. Bradycardia and flushing would not be expected.

A 45-year-old male patient with suspected acromegaly is seen at the clinic. To assist in making the diagnosis, which question should the nurse ask? a. "Have you had a recent head injury?" b. "Do you have to wear larger shoes now?" c. "Is there a family history of acromegaly?" d. "Are you experiencing tremors or anxiety?"

ANS: B Acromegaly causes an enlargement of the hands and feet. Head injury and family history are not risk factors for acromegaly. Tremors and anxiety are not clinical manifestations of acromegaly.

The nurse is planning postoperative care for a patient who is being admitted to the surgical unit form the recovery room after transsphenoidal resection of a pituitary tumor. Which nursing action should be included? a. Palpate extremities for edema. b. Measure urine volume every hour. c. Check hematocrit every 2 hours for 8 hours. d. Monitor continuous pulse oximetry for 24 hours.

ANS: B After pituitary surgery, the patient is at risk for diabetes insipidus caused by cerebral edema. Monitoring of urine output and urine specific gravity is essential. Hemorrhage is not a common problem. There is no need to check the hematocrit hourly. The patient is at risk for dehydration, not volume overload. The patient is not at high risk for problems with oxygenation, and continuous pulse oximetry is not needed.

A 31-year-old patient who has been diagnosed with human papillomavirus (HPV) infection gives a health history that includes smoking tobacco, taking oral contraceptives, and having been treated twice for vaginal candidiasis. Which topic will the nurse include in patient teaching? a. Use of water-soluble lubricants b. Risk factors for cervical cancer c. Antifungal cream administration d. Possible difficulties with conception

ANS: B Because HPV infection and smoking are both associated with increased cervical cancer risk, the nurse should emphasize the importance of avoiding smoking. An HPV infection does not decrease vaginal lubrication, decrease ability to conceive, or require the use of antifungal creams.

When caring for a 58-year-old patient with persistent menorrhagia, the nurse will plan to monitor the a. estrogen level. b. complete blood count (CBC). c. gonadotropin-releasing hormone (GNRH) level. d. serial human chorionic gonadotropin (hCG) results.

ANS: B Because anemia is a likely complication of menorrhagia, the nurse will need to check the CBC. Estrogen and GNRH levels are checked for patients with other problems, such as infertility. Serial hCG levels are monitored in patients who may be pregnant, which is not likely for this patient

A 24-year-old female says she wants to begin using oral contraceptives. Which information from the nursing assessment is most important to report to the health care provider? a. The patient quit smoking 5 months previously. b. The patient's blood pressure is 154/86 mm Hg. c. The patient has not been vaccinated for rubella. d. The patient has chronic iron-deficiency anemia.

ANS: B Because hypertension increases the risk for morbidity and mortality in women taking oral contraceptives, the patient's blood pressure should be controlled before oral contraceptives are prescribed. The other information also will be reported but will not affect the choice of contraceptive.

A 25-year-old woman has an induced abortion with suction curettage at an ambulatory surgical center. Which instructions will the nurse include when discharging the patient? a. "Heavy vaginal bleeding is expected for about 2 weeks." b. "You should abstain from sexual intercourse for 2 weeks." c. "Contraceptives should be avoided until your reexamination." d. "Irregular menstrual periods are expected for the next few months."

ANS: B Because infection is a possible complication of this procedure, the patient is advised to avoid intercourse until the reexamination in 2 weeks. Patients may be started on contraceptives on the day of the procedure. The patient should call the doctor if heavy vaginal bleeding occurs. No change in the regularity of the menstrual periods is expected

A 32-year-old who was admitted to the emergency department with severe abdominal pain is diagnosed with an ectopic pregnancy. The patient begins to cry and asks the nurse to leave her alone to grieve. Which action should the nurse take next? a. Stay with the patient and encourage her to discuss her feelings. b. Explain the reason for taking vital signs every 15 to 30 minutes. c. Close the door to the patient's room and minimize disturbances. d. Provide teaching about options for termination of the pregnancy.

ANS: B Because the patient is at risk for rupture of the fallopian tube and hemorrhage, frequent monitoring of vital signs is needed. The patient has asked to be left alone, so staying with her and encouraging her to discuss her feelings are inappropriate actions. Minimizing contact with her and closing the door of the room is unsafe because of the risk for hemorrhage. Because the patient has requested time to grieve, it would be inappropriate to provide teaching about options for pregnancy termination

The nurse is assessing a 41-year-old African American male patient diagnosed with a pituitary tumor causing panhypopituitarism. Assessment findings consistent with panhypopituitarism include a. high blood pressure. b. decreased facial hair. c. elevated blood glucose. d. tachycardia and cardiac palpitations.

ANS: B Changes in male secondary sex characteristics such as decreased facial hair, testicular atrophy, diminished spermatogenesis, loss of libido, impotence, and decreased muscle mass are associated with decreases in follicle stimulating hormone (FSH) and luteinizing hormone (LH). Fasting hypoglycemia and hypotension occur in panhypopituitarism as a result of decreases in adrenocorticotropic hormone (ACTH) and cortisol. Bradycardia is likely due to the decrease in thyroid stimulating hormone (TSH) and thyroid hormones associated with panhypopituitarism.

Which finding from the nurse's physical assessment of a 42-year-old male patient should be reported to the health care provider? a. One testis hangs lower than the other. b. Genital hair distribution is diamond shaped. c. Clear discharge is present at the penile meatus. d. Inguinal lymph nodes are nonpalpable bilaterally.

ANS: C Clear penile discharge may be indicative of a sexually transmitted infection (STI). The other findings are normal and do not need to be reported.

The nurse determines that demeclocycline (Declomycin) is effective for a patient with syndrome of inappropriate antidiuretic hormone (SIADH) based on finding that the patient's a. weight has increased. b. urinary output is increased. c. peripheral edema is decreased. d. urine specific gravity is increased.

ANS: B Demeclocycline blocks the action of antidiuretic hormone (ADH) on the renal tubules and increases urine output. An increase in weight or an increase in urine specific gravity indicates that the SIADH is not corrected. Peripheral edema does not occur with SIADH. A sudden weight gain without edema is a common clinical manifestation of this disorder.

The cardiac telemetry unit charge nurse receives status reports from other nursing units about four patients who need cardiac monitoring. Which patient should be transferred to the cardiac unit first? a. Patient with Hashimoto's thyroiditis and a heart rate of 102 b. Patient with tetany who has a new order for IV calcium chloride c. Patient with Cushing syndrome and a blood glucose of 140 mg/dL d. Patient with Addison's disease who takes hydrocortisone twice daily

ANS: B Emergency treatment of tetany requires IV administration of calcium; ECG monitoring will be required because cardiac arrest may occur if high calcium levels result from too-rapid administration. The information about the other patients indicates that they are more stable than the patient with tetany.

A 22-year-old patient reports her concern about not having a menstrual period for the past 7 months. Which statement by the patient indicates a possible related factor to the amenorrhea? a. "I drink at least 3 glasses of nonfat milk every day." b. "I run 7 to 8 miles every day to keep my weight down." c. "I was treated for a sexually transmitted infection 2 years ago." d. "I am not sexually active but currently I have an IUD."

ANS: B Excessive exercise can cause amenorrhea. The other statements by the patient do not suggest any urgent teaching needs.

A couple is scheduled to have a Huhner test for infertility. In preparation for the test, the nurse will instruct the couple about a. being sedated during the procedure. b. determining the estimated time of ovulation. c. experiencing shoulder pain after the procedure. d. refraining from intercourse before the appointment.

ANS: B For the Huhner test, the couple should have intercourse at the estimated time of ovulation and then arrive for the test 2 to 8 hours after intercourse. The other instructions would be used for other types of fertility testing.

Which nursing assessment of a 69-year-old patient is most important to make during initiation of thyroid replacement with levothyroxine (Synthroid)? a. Fluid balance b. Apical pulse rate c. Nutritional intake d. Orientation and alertness

ANS: B In older patients, initiation of levothyroxine therapy can increase myocardial oxygen demand and cause angina or dysrhythmias. The medication also is expected to improve mental status and fluid balance and will increase metabolic rate and nutritional needs, but these changes will not result in potentially life-threatening complications.

After receiving change-of-shift report about the following four patients, which patient should the nurse assess first? a. A 31-year-old female with Cushing syndrome and a blood glucose level of 244 mg/dL b. A 70-year-old female taking levothyroxine (Synthroid) who has an irregular pulse of 134 c. A 53-year-old male who has Addison's disease and is due for a scheduled dose of hydrocortisone (Solu-Cortef). d. A 22-year-old male admitted with syndrome of inappropriate antidiuretic hormone (SIADH) who has a serum sodium level of 130 mEq/L

ANS: B Initiation of thyroid replacement in older adults may cause angina and cardiac dysrhythmias. The patient's high pulse rate needs rapid investigation by the nurse to assess for and intervene with any cardiac problems. The other patients also require nursing assessment and/or actions but are not at risk for life-threatening complications.

Which question will the nurse in the endocrine clinic ask to help determine a patient's risk factors for goiter? a. "How much milk do you drink?" b. "What medications are you taking?" c. "Are your immunizations up to date?" d. "Have you had any recent neck injuries?"

ANS: B Medications that contain thyroid-inhibiting substances can cause goiter. Milk intake, neck injury, and immunization history are not risk factors for goiter.

A patient who had radical neck surgery to remove a malignant tumor developed hypoparathyroidism. The nurse should plan to teach the patient about a. bisphosphonates to reduce bone demineralization. b. calcium supplements to normalize serum calcium levels. c. increasing fluid intake to decrease risk for nephrolithiasis. d. including whole grains in the diet to prevent constipation.

ANS: B Oral calcium supplements are used to maintain the serum calcium in normal range and prevent the complications of hypocalcemia. Whole grain foods decrease calcium absorption and will not be recommended. Bisphosphonates will lower serum calcium levels further by preventing calcium from being reabsorbed from bone. Kidney stones are not a complication of hypoparathyroidism and low calcium levels.

A 29-year-old patient who is trying to become pregnant asks the nurse how to determine when she is most likely to conceive. The nurse explains that a. ovulation is unpredictable unless there are regular menstrual periods. b. ovulation prediction kits provide accurate information about ovulation. c. she will need to bring a specimen of cervical mucus to the clinic for testing. d. she should take her body temperature daily and have intercourse when it drops.

ANS: B Ovulation prediction kits indicate when luteinizing hormone (LH) levels first rise. Ovulation occurs about 28 to 36 hours after the first rise of LH. This information can be used to determine the best time for intercourse. Body temperature rises at ovulation. Postcoital cervical smears are used in infertility testing, but they do not predict the best time for conceiving and are not obtained by the patient. Determination of the time of ovulation can be predicted by basal body temperature charts or ovulation prediction kits and is not dependent on regular menstrual periods

An 18-year-old female patient who has been admitted to the emergency department after a motor vehicle crash is scheduled for chest and abdominal x-rays. Which information is most important to report to the health care provider before the x-rays are obtained? a. Severity of abdominal pain b. Positive result of hCG test c. Blood pressure 172/88 mm Hg d. Temperature 102.1° F (38.9° C)

ANS: B Positive hCG testing indicates that the patient is pregnant and that unnecessary abdominal x-rays should be avoided. The other information is also important to report, but it will not affect whether the x-rays should be done.

Which information will the nurse include when teaching a 50-year-old male patient about somatropin (Genotropin)? a. The medication will be needed for 3 to 6 months. b. Inject the medication subcutaneously every day. c. Blood glucose levels may decrease when taking the medication. d. Stop taking the medication if swelling of the hands or feet occurs.

ANS: B Somatropin is injected subcutaneously on a daily basis, preferably in the evening. The patient will need to continue on somatropin for life. If swelling or other common adverse effects occur, the health care provider should be notified. Growth hormone will increase blood glucose levels.

Which intervention will the nurse include in the plan of care for a 52-year-old male patient with syndrome of inappropriate antidiuretic hormone (SIADH)? a. Monitor for peripheral edema. b. Offer patient hard candies to suck on. c. Encourage fluids to 2 to 3 liters per day. d. Keep head of bed elevated to 30 degrees.

ANS: B Sucking on hard candies decreases thirst for a patient on fluid restriction. Patients with SIADH are on fluid restrictions of 800 to 1000 mL/day. Peripheral edema is not seen with SIADH. The head of the bed is elevated no more than 10 degrees to increase left atrial filling pressure and decrease antidiuretic hormone (ADH) release.

A healthy 28-year-old who has been vaccinated against human papillomavirus (HPV) has a normal Pap test. Which information will the nurse include in patient teaching when calling the patient with the results of the Pap test? a. You can wait until age 30 before having another Pap test. b. Pap testing is recommended every 3 years for women your age. c. No further Pap testing is needed until you decide to become pregnant. d. Yearly Pap testing is suggested for women with multiple sexual partners.

ANS: B The current national guidelines suggest Pap testing every 3 years for patients between ages 21 to 65. Although HPV immunization does protect against cervical cancer, the recommendations are unchanged for individuals who have received the HPV vaccination

A 37-year-old patient has just arrived in the postanesthesia recovery unit (PACU) after a thyroidectomy. Which information is most important to communicate to the surgeon? a. The patient reports 7/10 incisional pain. b. The patient has increasing neck swelling. c. The patient is sleepy and difficult to arouse. d. The patient's cardiac rate is 112 beats/minute.

ANS: B The neck swelling may lead to respiratory difficulty, and rapid intervention is needed to prevent airway obstruction. The incisional pain should be treated but is not unusual after surgery. A heart rate of 112 is not unusual in a patient who has been hyperthyroid and has just arrived in the PACU from surgery. Sleepiness in the immediate postoperative period is expected.

A 38-year-old male patient is admitted to the hospital in Addisonian crisis. Which patient statement supports a nursing diagnosis of ineffective self-health management related to lack of knowledge about management of Addison's disease? a. "I frequently eat at restaurants, and my food has a lot of added salt." b. "I had the stomach flu earlier this week, so I couldn't take the hydrocortisone." c. "I always double my dose of hydrocortisone on the days that I go for a long run." d. "I take twice as much hydrocortisone in the morning dose as I do in the afternoon."

ANS: B The need for hydrocortisone replacement is increased with stressors such as illness, and the patient needs to be taught to call the health care provider because medication and IV fluids and electrolytes may need to be given. The other patient statements indicate appropriate management of the Addison's disease.

A 24-year-old patient with pelvic inflammatory disease (PID) is being treated with oral antibiotics as an outpatient. Which instruction will be included in patient teaching? a. Abdominal pain may persist for several weeks. b. Return for a follow-up appointment in 2 to 3 days. c. Instruct a male partner to use a condom during sexual intercourse for the next week. d. Nonsteroidal antiinflammatory drug (NSAID) use may prevent pelvic organ scarring

ANS: B The patient is instructed to return for follow-up in 48 to 72 hours. The patient should abstain from intercourse for 3 weeks. Abdominal pain should subside with effective antibiotic therapy. Corticosteroids may help prevent inflammation and scarring, but NSAIDs will not decrease scarring

Which nursing action will be included in the plan of care for a 55-year-old patient with Graves' disease who has exophthalmos? a. Place cold packs on the eyes to relieve pain and swelling. b. Elevate the head of the patient's bed to reduce periorbital fluid. c. Apply alternating eye patches to protect the corneas from irritation. d. Teach the patient to blink every few seconds to lubricate the corneas.

ANS: B The patient should sit upright as much as possible to promote fluid drainage from the periorbital area. With exophthalmos, the patient is unable to close the eyes completely to blink. Lubrication of the eyes, rather than eye patches, will protect the eyes from developing corneal scarring. The swelling of the eye is not caused by excessive blood flow to the eye, so cold packs will not be helpful.

A 63-year-old woman undergoes an anterior and posterior (A&P) colporrhaphy for repair of a cystocele and rectocele. Which nursing action will be included in the postoperative care plan? a. Encourage a high-fiber diet. b. Perform indwelling catheter care. c. Repack the vagina with gauze daily. d. Teach the patient to insert a pessary.

ANS: B The patient will have a retention catheter for several days after surgery to keep the bladder empty and decrease strain on the suture. A pessary will not be needed after the surgery. Vaginal wound packing is not usually used after an A&P repair. A low-residue diet will be ordered after posterior colporrhaphy

A 63-year-old patient with primary hyperparathyroidism has a serum phosphorus level of 1.7 mg/dL (0.55 mmol/L) and calcium of 14 mg/dL (3.5 mmol/L). Which nursing action should be included in the plan of care? a. Restrict the patient to bed rest. b. Encourage 4000 mL of fluids daily. c. Institute routine seizure precautions. d. Assess for positive Chvostek's sign.

ANS: B The patient with hypercalcemia is at risk for kidney stones, which may be prevented by a high fluid intake. Seizure precautions and monitoring for Chvostek's or Trousseau's sign are appropriate for hypocalcemic patients. The patient should engage in weight-bearing exercise to decrease calcium loss from bone.

A patient who had a subtotal thyroidectomy earlier today develops laryngeal stridor and a cramp in the right hand upon returning to the surgical nursing unit. Which collaborative action will the nurse anticipate next? a. Suction the patient's airway. b. Administer IV calcium gluconate. c. Plan for emergency tracheostomy. d. Prepare for endotracheal intubation.

ANS: B The patient's clinical manifestations of stridor and cramping are consistent with tetany caused by hypocalcemia resulting from damage to the parathyroid glands during surgery. Endotracheal intubation or tracheostomy may be needed if the calcium does not resolve the stridor. Suctioning will not correct the stridor.

A 34-year-old woman who is discussing contraceptive options with the nurse says, "I want to have children, but not for a few years." Which response by the nurse is appropriate? a. "If you do not become pregnant within the next few years, you never will." b. "You may have more difficulty becoming pregnant after about age 35." c. "You have many years of fertility left, so there is no rush to have children." d. "You should plan to stop taking oral contraceptives several years before you want to become pregnant."

ANS: B The probability of successfully becoming pregnant decreases after age 35, although some patients may have no difficulty in becoming pregnant. Oral contraceptives do not need to be withdrawn for several years for a woman to become pregnant. Although the patient may be fertile for many years, it would be inaccurate to indicate that there is no concern about fertility as she becomes older. Although the risk for infertility increases after age 35, not all patients have difficulty in conceiving

Which assessment finding in a woman who recently started taking hormone therapy (HT) is most important for the nurse to report to the health care provider? a. Breast tenderness b. Left calf swelling c. Weight gain of 3 lb d. Intermittent spotting

ANS: B Unilateral calf swelling may indicate deep vein thrombosis caused by the changes in coagulation associated with HT and would indicate that the HT should be discontinued. Breast tenderness, weight gain, and intermittent spotting are common side effects of HT and do not indicate a need for a change in therapy

A 32-year-old woman is scheduled for an induced abortion using instillation of hypertonic saline solution. Which information will the nurse plan to discuss with the patient before the procedure? a. The patient will require a general anesthetic. b. The expulsion of the fetus may take 1 to 2 days. c. There is a possibility that the patient may deliver a live fetus. d. The procedure may be unsuccessful in terminating the pregnancy.

ANS: B Uterine contractions take 12 to 36 hours to begin after the hypertonic saline is instilled. Because the saline is feticidal, the nurse does not need to discuss any possibility of a live delivery or that the pregnancy termination will not be successful. General anesthesia is not needed for this procedure

A 56-year-old patient who is disoriented and reports a headache and muscle cramps is hospitalized with possible syndrome of inappropriate antidiuretic hormone (SIADH). The nurse would expect the initial laboratory results to include a(n) a. elevated hematocrit. b. decreased serum sodium. c. low urine specific gravity. d. increased serum chloride.

ANS: B When water is retained, the serum sodium level will drop below normal, causing the clinical manifestations reported by the patient. The hematocrit will decrease because of the dilution caused by water retention. Urine will be more concentrated with a higher specific gravity. The serum chloride level will usually decrease along with the sodium level.

Which nursing actions can the nurse working in a women's health clinic delegate to unlicensed assistive personnel (UAP) (select all that apply)? a. Call a patient with the results of an endometrial biopsy. b. Assist the health care provider with performing a Pap test. c. Draw blood for CA-125 levels for a patient with ovarian cancer. d. Screen a patient for use of medications that may cause amenorrhea. e. Teach the parent of a 10-year-old about the human papilloma virus (HPV) vaccine (Gardasil).

ANS: B, C Assisting with a Pap test and drawing blood (if trained) are skills that require minimal critical thinking and judgment and can be safely delegated to UAP. Patient teaching, calling a patient who may have questions about results of diagnostic testing, and risk-factor screening all require more education and critical thinking and should be done by the registered nurse (RN).

To monitor for complications in a patient with type 2 diabetes, which tests will the nurse in the diabetic clinic schedule at least annually (select all that apply)? a. Chest x-ray b. Blood pressure c. Serum creatinine d. Urine for microalbuminuria e. Complete blood count (CBC) f. Monofilament testing of the foot

ANS: B, C, D, F Blood pressure, serum creatinine, urine testing for microalbuminuria, and monofilament testing of the foot are recommended at least annually to screen for possible microvascular and macrovascular complications of diabetes. Chest x-ray and CBC might be ordered if the diabetic patient presents with symptoms of respiratory or infectious problems but are not routinely included in screening.

Which statement by a nurse to a patient newly diagnosed with type 2 diabetes is correct? a. Insulin is not used to control blood glucose in patients with type 2 diabetes. b. Complications of type 2 diabetes are less serious than those of type 1 diabetes. c. Changes in diet and exercise may control blood glucose levels in type 2 diabetes. d. Type 2 diabetes is usually diagnosed when the patient is admitted with a hyperglycemic coma.

ANS: C For some patients with type 2 diabetes, changes in lifestyle are sufficient to achieve blood glucose control. Insulin is frequently used for type 2 diabetes, complications are equally severe as for type 1 diabetes, and type 2 diabetes is usually diagnosed with routine laboratory testing or after a patient develops complications such as frequent yeast infections.

When a patient who takes metformin (Glucophage) to manage type 2 diabetes develops an allergic rash from an unknown cause, the health care provider prescribes prednisone (Deltasone). The nurse will anticipate that the patient may a. need a diet higher in calories while receiving prednisone. b. develop acute hypoglycemia while taking the prednisone. c. require administration of insulin while taking prednisone. d. have rashes caused by metformin-prednisone interactions.

ANS: C Glucose levels increase when patients are taking corticosteroids, and insulin may be required to control blood glucose. Hypoglycemia is not a side effect of prednisone. Rashes are not an adverse effect caused by taking metformin and prednisone simultaneously. The patient may have an increased appetite when taking prednisone, but will not need a diet that is higher in calories.

The health care provider suspects the Somogyi effect in a 50-year-old patient whose 6:00 AM blood glucose is 230 mg/dL. Which action will the nurse teach the patient to take? a. Avoid snacking at bedtime. b. Increase the rapid-acting insulin dose. c. Check the blood glucose during the night d. Administer a larger dose of long-acting insulin.

ANS: C If the Somogyi effect is causing the patient's increased morning glucose level, the patient will experience hypoglycemia between 2:00 and 4:00 AM. The dose of insulin will be reduced, rather than increased. A bedtime snack is used to prevent hypoglycemic episodes during the night.

A 26-year-old female with type 1 diabetes develops a sore throat and runny nose after caring for her sick toddler. The patient calls the clinic for advice about her symptoms and a blood glucose level of 210 mg/dL despite taking her usual glargine (Lantus) and lispro (Humalog) insulin. The nurse advises the patient to a. use only the lispro insulin until the symptoms are resolved. b. limit intake of calories until the glucose is less than 120 mg/dL. c. monitor blood glucose every 4 hours and notify the clinic if it continues to rise. d. decrease intake of carbohydrates until glycosylated hemoglobin is less than 7%.

ANS: C Infection and other stressors increase blood glucose levels and the patient will need to test blood glucose frequently, treat elevations appropriately with lispro insulin, and call the health care provider if glucose levels continue to be elevated. Discontinuing the glargine will contribute to hyperglycemia and may lead to diabetic ketoacidosis (DKA). Decreasing carbohydrate or caloric intake is not appropriate because the patient will need more calories when ill. Glycosylated hemoglobin testing is not used to evaluate short-term alterations in blood glucose.

When a patient with type 2 diabetes is admitted for a cholecystectomy, which nursing action can the nurse delegate to a licensed practical/vocational nurse (LPN/LVN)? a. Communicate the blood glucose level and insulin dose to the circulating nurse in surgery. b. Discuss the reason for the use of insulin therapy during the immediate postoperative period. c. Administer the prescribed lispro (Humalog) insulin before transporting the patient to surgery. d. Plan strategies to minimize the risk for hypoglycemia or hyperglycemia during the postoperative period.

ANS: C LPN/LVN education and scope of practice includes administration of insulin. Communication about patient status with other departments, planning, and patient teaching are skills that require RN education and scope of practice.

The nurse determines a need for additional instruction when the patient with newly diagnosed type 1 diabetes says which of the following? a. "I can have an occasional alcoholic drink if I include it in my meal plan." b. "I will need a bedtime snack because I take an evening dose of NPH insulin." c. "I can choose any foods, as long as I use enough insulin to cover the calories." d. "I will eat something at meal times to prevent hypoglycemia, even if I am not hungry."

ANS: C Most patients with type 1 diabetes need to plan diet choices very carefully. Patients who are using intensified insulin therapy have considerable flexibility in diet choices but still should restrict dietary intake of items such as fat, protein, and alcohol. The other patient statements are correct and indicate good understanding of the diet instruction.

A 26-year-old patient with diabetes rides a bicycle to and from work every day. Which site should the nurse teach the patient to administer the morning insulin? a. thigh. b. buttock. c. abdomen. d. upper arm.

ANS: C Patients should be taught not to administer insulin into a site that will be exercised because exercise will increase the rate of absorption. The thigh, buttock, and arm are all exercised by riding a bicycle.

After the nurse has finished teaching a patient who has a new prescription for exenatide (Byetta), which patient statement indicates that the teaching has been effective? a. "I may feel hungrier than usual when I take this medicine." b. "I will not need to worry about hypoglycemia with the Byetta." c. "I should take my daily aspirin at least an hour before the Byetta." d. "I will take the pill at the same time I eat breakfast in the morning."

ANS: C Since exenatide slows gastric emptying, oral medications should be taken at least an hour before the exenatide to avoid slowing absorption. Exenatide is injected and increases feelings of satiety. Hypoglycemia can occur with this medication.

A few weeks after an 82-year-old with a new diagnosis of type 2 diabetes has been placed on metformin (Glucophage) therapy and taught about appropriate diet and exercise, the home health nurse makes a visit. Which finding by the nurse is most important to discuss with the health care provider? a. Hemoglobin A1C level is 7.9%. b. Last eye exam was 18 months ago. c. Glomerular filtration rate is decreased. d. Patient has questions about the prescribed diet.

ANS: C The decrease in renal function may indicate a need to adjust the dose of metformin or change to a different medication. In older patients, the goal for A1C may be higher in order to avoid complications associated with hypoglycemia. The nurse will plan on scheduling the patient for an eye exam and addressing the questions about diet, but the biggest concern is the patient's decreased renal function.

A patient who was admitted with diabetic ketoacidosis secondary to a urinary tract infection has been weaned off an insulin drip 30 minutes ago. The patient reports feeling lightheaded and sweaty. Which action should the nurse take first? a. Infuse dextrose 50% by slow IV push. b. Administer 1 mg glucagon subcutaneously. c. Obtain a glucose reading using a finger stick. d. Have the patient drink 4 ounces of orange juice.

ANS: C The patient's clinical manifestations are consistent with hypoglycemia and the initial action should be to check the patient's glucose with a finger stick or order a stat blood glucose. If the glucose is low, the patient should ingest a rapid-acting carbohydrate, such as orange juice. Glucagon or dextrose 50% might be given if the patient's symptoms become worse or if the patient is unconscious.

After change-of-shift report, which patient will the nurse assess first? a. 19-year-old with type 1 diabetes who was admitted with possible dawn phenomenon b. 35-year-old with type 1 diabetes whose most recent blood glucose reading was 230 mg/dL c. 60-year-old with hyperosmolar hyperglycemic syndrome who has poor skin turgor and dry oral mucosa d. 68-year-old with type 2 diabetes who has severe peripheral neuropathy and complains of burning foot pain

ANS: C The patient's diagnosis of HHS and signs of dehydration indicate that the nurse should rapidly assess for signs of shock and determine whether increased fluid infusion is needed. The other patients also need assessment and intervention but do not have life-threatening complications.

A 48-year-old male patient screened for diabetes at a clinic has a fasting plasma glucose level of 120 mg/dL (6.7 mmol/L). The nurse will plan to teach the patient about a. self-monitoring of blood glucose. b. using low doses of regular insulin. c. lifestyle changes to lower blood glucose. d. effects of oral hypoglycemic medications.

ANS: C The patient's impaired fasting glucose indicates prediabetes, and the patient should be counseled about lifestyle changes to prevent the development of type 2 diabetes. The patient with prediabetes does not require insulin or oral hypoglycemics for glucose control and does not need to self-monitor blood glucose.

Which information is most important for the nurse to report to the health care provider before a patient with type 2 diabetes is prepared for a coronary angiogram? a. The patient's most recent HbA1C was 6.5%. b. The patient's admission blood glucose is 128 mg/dL. c. The patient took the prescribed metformin (Glucophage) today. d. The patient took the prescribed captopril (Capoten) this morning.

ANS: C To avoid lactic acidosis, metformin should be discontinued a day or 2 before the coronary arteriogram and should not be used for 48 hours after IV contrast media are administered. The other patient data will also be reported but do not indicate any need to reschedule the procedure.

The nurse is assessing a 22-year-old patient experiencing the onset of symptoms of type 1 diabetes. Which question is most appropriate for the nurse to ask? a. "Are you anorexic?" b. "Is your urine dark colored?" c. "Have you lost weight lately?" d. "Do you crave sugary drinks?"

ANS: C Weight loss occurs because the body is no longer able to absorb glucose and starts to break down protein and fat for energy. The patient is thirsty but does not necessarily crave sugar-containing fluids. Increased appetite is a classic symptom of type 1 diabetes. With the classic symptom of polyuria, urine will be very dilute.

A 29-year-old woman with systemic lupus erythematosus has been prescribed 2 weeks of high-dose prednisone therapy. Which information about the prednisone is most important for the nurse to include? a. "Weigh yourself daily to monitor for weight gain caused by increased appetite." b. "A weight-bearing exercise program will help minimize the risk for osteoporosis." c. "The prednisone dose should be decreased gradually rather than stopped suddenly." d. "Call the health care provider if you experience mood alterations with the prednisone."

ANS: C Acute adrenal insufficiency may occur if exogenous corticosteroids are suddenly stopped. Mood alterations and weight gain are possible adverse effects of corticosteroid use, but these are not life-threatening effects. Osteoporosis occurs when patients take corticosteroids for longer periods.

The nurse is caring for a patient following an adrenalectomy. The highest priority in the immediate postoperative period is to a. protect the patient's skin. b. monitor for signs of infection. c. balance fluids and electrolytes. d. prevent emotional disturbances.

ANS: C After adrenalectomy, the patient is at risk for circulatory instability caused by fluctuating hormone levels, and the focus of care is to assess and maintain fluid and electrolyte status through the use of IV fluids and corticosteroids. The other goals are also important for the patient but are not as immediately life threatening as the circulatory collapse that can occur with fluid and electrolyte disturbances.

A patient has just arrived on the unit after a thyroidectomy. Which action should the nurse take first? a. Observe the dressing for bleeding. b. Check the blood pressure and pulse. c. Assess the patient's respiratory effort. d. Support the patient's head with pillows.

ANS: C Airway obstruction is a possible complication after thyroidectomy because of swelling or bleeding at the site or tetany. The priority nursing action is to assess the airway. The other actions are also part of the standard nursing care postthyroidectomy but are not as high of a priority.

After a 22-year-old female patient with a pituitary adenoma has had a hypophysectomy, the nurse will teach about the need for a. sodium restriction to prevent fluid retention. b. insulin to maintain normal blood glucose levels. c. oral corticosteroids to replace endogenous cortisol. d. chemotherapy to prevent malignant tumor recurrence.

ANS: C Antidiuretic hormone (ADH), cortisol, and thyroid hormone replacement will be needed for life after hypophysectomy. Without the effects of adrenocorticotropic hormone (ACTH) and cortisol, the blood glucose and serum sodium will be low unless cortisol is replaced. An adenoma is a benign tumor, and chemotherapy will not be needed.

A 19-year-old patient calls the school clinic and tells the nurse, "My menstrual period is very heavy this time. I have to change my tampon every 4 hours." Which action should the nurse take next? a. Tell the patient that her flow is not unusually heavy. b. Schedule the patient for an appointment later that day. c. Ask the patient how heavy her usual menstrual flow is. d. Have the patient call again if the heavy flow continues.

ANS: C Because a heavy menstrual flow is usually indicated by saturating a pad or tampon in 1 to 2 hours, the nurse should first assess how heavy the patient's usual flow is. There is no need to schedule the patient for an appointment that day. The patient may need to call again, but this is not the first action that the nurse should take. Telling the patient that she does not have a heavy flow implies that the patient's concern is not important.

The nurse is providing teaching by telephone to a patient who is scheduled for a pelvic examination and Pap test next week. The nurse instructs the patient that she should a. shower, but not take a tub bath, before the examination. b. not have sexual intercourse the day before the Pap test. c. avoid douching for at least 24 hours before the examination. d. schedule to have the Pap test just after her menstrual period.

ANS: C Because the results of a Pap test may be affected by douching, the patient should not douche before the examination. The exam may be scheduled without regard to the menstrual period. The patient may shower or bathe before the examination. Sexual intercourse does not affect the results of the examination or Pap test.

Which information will the nurse include when teaching a patient who has developed a small vesicovaginal fistula 2 weeks into the postpartum period? a. Take stool softeners to prevent fecal contamination of the vagina. b. Limit oral fluid intake to minimize the quantity of urinary drainage. c. Change the perineal pad frequently to prevent perineal skin breakdown. d. Call the health care provider immediately if urine drains from the vagina.

ANS: C Because urine will leak from the bladder, the patient should plan to use perineal pads and change them frequently. A high fluid intake is recommended to decrease the risk for urinary tract infections. Drainage of urine from the vagina is expected with vesicovaginal fistulas. Fecal contamination is not a concern with vesicovaginal fistulas.

A 25-year-old woman who is scheduled for a routine gynecologic examination tells the nurse that she has had intercourse during the last year with several men. The nurse will plan to teach about the reason for a. contraceptive use. b. antibiotic therapy. c. Chlamydia testing. d. pregnancy testing.

ANS: C Chlamydia testing is recommended annually for women with multiple sex partners. There is no indication that the patient needs teaching about contraceptives, pregnancy testing, or antibiotic therapy

A 49-year-old woman is considering the use of combined estrogen-progesterone hormone replacement therapy (HT) during menopause. Which information will the nurse include during their discussion? a. Use of estrogen-containing vaginal creams provides most of the same benefits as oral HT. b. Increased incidence of colon cancer in women taking HT requires more frequent colonoscopy. c. HT decreases osteoporosis risk and increases the risk for cardiovascular disease and breast cancer. d. Use of HT for up to 10 years to prevent symptoms such as hot flashes is generally considered safe.

ANS: C Data from the Women's Health Initiative indicate an increased risk for cardiovascular disease and breast cancer in women taking combination HT but a decrease in hip fractures. Vaginal creams decrease symptoms related to vaginal atrophy and dryness, but they do not offer the other benefits of HT, such as decreased hot flashes. Most women who use HT are placed on short-term treatment and are not treated for up to 10 years. The incidence of colon cancer decreases in women taking HRT

Which question should the nurse ask when assessing a 60-year-old patient who has a history of benign prostatic hyperplasia (BPH)? a. "Have you noticed any unusual discharge from your penis?" b. "Has there been any change in your sex life in the last year?" c. "Has there been a decrease in the force of your urinary stream?" d. "Have you been experiencing any difficulty in achieving an erection?"

ANS: C Enlargement of the prostate blocks the urethra, leading to urinary changes such as a decrease in the force of the urinary stream. The other questions address possible problems with infection or sexual difficulties, but they would not be helpful in determining whether there were functional changes caused by BPH.

Which information will the nurse teach a 48-year-old patient who has been newly diagnosed with Graves' disease? a. Exercise is contraindicated to avoid increasing metabolic rate. b. Restriction of iodine intake is needed to reduce thyroid activity. c. Antithyroid medications may take several months for full effect. d. Surgery will eventually be required to remove the thyroid gland.

ANS: C Medications used to block the synthesis of thyroid hormones may take 2 to 3 months before the full effect is seen. Large doses of iodine are used to inhibit the synthesis of thyroid hormones. Exercise using large muscle groups is encouraged to decrease the irritability and hyperactivity associated with high levels of thyroid hormones. Radioactive iodine is the most common treatment for Graves' disease although surgery may be used.

An expected nursing diagnosis for a 30-year-old patient admitted to the hospital with symptoms of diabetes insipidus is a. excess fluid volume related to intake greater than output. b. impaired gas exchange related to fluid retention in lungs. c. sleep pattern disturbance related to frequent waking to void. d. risk for impaired skin integrity related to generalized edema.

ANS: C Nocturia occurs as a result of the polyuria caused by diabetes insipidus. Edema, excess fluid volume, and fluid retention are not expected.

A 27-year-old patient tells the nurse that she would like a prescription for oral contraceptives to control her premenstrual dysphoric disorder (PMD-D) symptoms. Which patient information is most important to communicate to the health care provider? a. Bilateral breast tenderness b. Frequent abdominal bloating c. History of migraine headaches d. Previous spontaneous abortion

ANS: C Oral contraceptives are contraindicated in patients with a history of migraine headaches. The other patient information would not prevent the patient from receiving oral contraceptives

An 18-year-old requests a prescription for birth control pills to control severe abdominal cramping and headaches during her menstrual periods. Which should the nurse take first? a. Determine whether the patient is sexually active. b. Teach about the side effects of oral contraceptives. c. Take a personal and family health history from the patient. d. Suggest nonsteroidal antiinflammatory drugs (NSAIDs) for relief.

ANS: C Oral contraceptives may be appropriate to control this patient's symptoms, but the patient's health history may indicate contraindications to oral contraceptive use. Because the patient is requesting contraceptives for management of dysmenorrhea, whether she is sexually active is irrelevant. Because the patient is asking for birth control pills, responding that she should try NSAIDs is nontherapeutic. The patient does not need teaching about oral contraceptive side effects at this time.

Which action should the nurse take when a 35-year-old patient has a result of minor cellular changes on her Pap test? a. Teach the patient about colposcopy. b. Teach the patient about punch biopsy. c. Schedule another Pap test in 4 months. d. Administer the human papillomavirus (HPV) vaccine.

ANS: C Patients with minor changes on the Pap test can be followed with Pap tests every 4 to 6 months because these changes may revert to normal. Punch biopsy or colposcopy may be used if the Pap test shows more prominent changes. The HPV vaccine may reduce the risk for cervical cancer, but it is recommended only for ages 9 through 26.

A 37-year-old patient is being admitted with a diagnosis of Cushing syndrome. Which findings will the nurse expect during the assessment? a. Chronically low blood pressure b. Bronzed appearance of the skin c. Purplish streaks on the abdomen d. Decreased axillary and pubic hair

ANS: C Purplish-red striae on the abdomen are a common clinical manifestation of Cushing syndrome. Hypotension and bronzed-appearing skin are manifestations of Addison's disease. Decreased axillary and pubic hair occur with androgen deficiency.

A 49-year-old man who has type 2 diabetes, high blood pressure, hyperlipidemia, and gastroesophageal reflux tells the nurse that he has had recent difficulty in achieving an erection. Which of the following drugs from his current medications list may cause erectile dysfunction (ED)? a. Ranitidine (Zantac) b. Atorvastatin (Lipitor) c. Propranolol (Inderal) d. Metformin (Glucophage)

ANS: C Some antihypertensives may cause erectile dysfunction, and the nurse should anticipate a change in antihypertensive therapy. The other medications will not affect erectile function.

Which information obtained by the nurse in the endocrine clinic about a patient who has been taking prednisone (Deltasone) 40 mg daily for 3 weeks is most important to report to the health care provider? a. Patient's blood pressure is 148/94 mm Hg. b. Patient has bilateral 2+ pitting ankle edema. c. Patient stopped taking the medication 2 days ago. d. Patient has not been taking the prescribed vitamin D.

ANS: C Sudden cessation of corticosteroids after taking the medication for a week or more can lead to adrenal insufficiency, with problems such as severe hypotension and hypoglycemia. The patient will need immediate evaluation by the health care provider to prevent and/or treat adrenal insufficiency. The other information will also be reported, but does not require rapid treatment.

Which assessment finding of a 42-year-old patient who had a bilateral adrenalectomy requires the most rapid action by the nurse? a. The blood glucose is 176 mg/dL. b. The lungs have bibasilar crackles. c. The blood pressure (BP) is 88/50 mm Hg. d. The patient reports 5/10 incisional pain.

ANS: C The decreased BP indicates possible adrenal insufficiency. The nurse should immediately notify the health care provider so that corticosteroid medications can be administered. The nurse should also address the elevated glucose, incisional pain, and crackles with appropriate collaborative or nursing actions, but prevention and treatment of acute adrenal insufficiency is the priority after adrenalectomy.

The nurse is caring for a 20-year-old patient with pelvic inflammatory disease (PID) requiring hospitalization. Which nursing intervention will be included in the plan of care? a. Monitor liver function tests. b. Use cold packs PRN for pelvic pain. c. Elevate the head of the bed to at least 30 degrees. d. Teach the patient how to perform Kegel exercises.

ANS: C The head of the bed should be elevated to at least 30 degrees to promote drainage of the pelvic cavity and prevent abscess formation higher in the abdomen. Although a possible complication of PID is acute perihepatitis, liver function tests will remain normal. There is no indication for increased fluid intake. Application of heat is used to reduce pain. Kegel exercises are not helpful in PID

A 47-year-old woman asks whether she is going into menopause if she has not had a menstrual period for 3 months. The best response by the nurse is which of the following? a. "Have you thought about using hormone replacement therapy?" b. "Most women feel a little depressed about entering menopause." c. "What was your menstrual pattern before your periods stopped?" d. "Since you are in your mid-40s, it is likely that you are menopausal."

ANS: C The initial response by the nurse should be to assess the patient's baseline menstrual pattern. Although many women do enter menopause in the mid-40s, more information about this patient is needed before telling her that it is likely she is menopausal. Although hormone therapy (HT) may be prescribed, further assessment of the patient is needed before discussing therapies for menopause. Because the response to menopause is very individual, the nurse should not assume that the patient is experiencing any adverse emotional reactions.

The nurse has just received change-of-shift report about the following four patients. Which patient should be assessed first? a. A patient with a cervical radium implant in place who is crying in her room b. A patient who is complaining of 5/10 pain after an abdominal hysterectomy c. A patient with a possible ectopic pregnancy who is complaining of shoulder pain d. A patient in the fifteenth week of gestation who has uterine cramping and spotting

ANS: C The patient with the ectopic pregnancy has symptoms consistent with rupture and needs immediate assessment for signs of hemorrhage and possible transfer to surgery. The other patients should also be assessed as quickly as possible but do not have symptoms of life-threatening complications

The following patients call the outpatient clinic. Which phone call should the nurse return first? a. A 44-year-old patient who has bloody discharge after a hysteroscopy earlier today b. A 64-year-old patient who is experiencing shoulder pain after a laparoscopy yesterday c. A 34-year-old patient who is short of breath after pelvic computed tomography (CT) with contrast d. A 54-year-old patient who has severe breast tenderness following a needle aspiration breast biopsy

ANS: C The patient's dyspnea suggests a delayed reaction to the iodine dye used for the CT scan. The other patient's symptoms are not unusual after the procedures they had done.

A patient develops carpopedal spasms and tingling of the lips following a parathyroidectomy. Which action should the nurse take first? a. Administer the ordered muscle relaxant. b. Give the ordered oral calcium supplement. c. Have the patient rebreathe from a paper bag. d. Start the PRN oxygen at 2 L/min per cannula.

ANS: C The patient's symptoms suggest mild hypocalcemia. The symptoms of hypocalcemia will be temporarily reduced by having the patient breathe into a paper bag, which will raise the PaCO2 and create a more acidic pH. The muscle relaxant will have no impact on the ionized calcium level. Although severe hypocalcemia can cause laryngeal stridor, there is no indication that this patient is experiencing laryngeal stridor or needs oxygen. Calcium supplements will be given to normalize calcium levels quickly, but oral supplements will take time to be absorbed.

Which assessment finding for a 33-year-old female patient admitted with Graves' disease requires the most rapid intervention by the nurse? a. Bilateral exophthalmos b. Heart rate 136 beats/minute c. Temperature 103.8° F (40.4° C) d. Blood pressure 166/100 mm Hg

ANS: C The patient's temperature indicates that the patient may have thyrotoxic crisis and that interventions to lower the temperature are needed immediately. The other findings also require intervention but do not indicate potentially life-threatening complications.

The nurse in the infertility clinic is explaining in vitro fertilization (IVF) to a couple. The woman tells the nurse that they cannot afford IVF on her husband's salary. The man replies that if his wife worked outside the home, they would have enough money. Which nursing diagnosis is appropriate? a. Decisional conflict related to inadequate financial resources b. Ineffective sexuality patterns related to psychological stress c. Defensive coping related to anxiety about lack of conception d. Ineffective denial related to frustration about continued infertility

ANS: C The statements made by the couple are consistent with the diagnosis of defensive coping. No data indicate that ineffective sexuality and ineffective denial are problems. Although the couple is quarreling about finances, the data do not provide information indicating that the finances are inadequate

A 30-year-old patient seen in the emergency department for severe headache and acute confusion is found to have a serum sodium level of 118 mEq/L. The nurse will anticipate the need for which diagnostic test? a. Urinary 17-ketosteroids b. Antidiuretic hormone level c. Growth hormone stimulation test d. Adrenocorticotropic hormone level

B

A 62-year-old patient with hyperthyroidism is to be treated with radioactive iodine (RAI). The nurse instructs the patient a. about radioactive precautions to take with all body secretions. b. that symptoms of hyperthyroidism should be relieved in about a week. c. that symptoms of hypothyroidism may occur as the RAI therapy takes effect. d. to discontinue the antithyroid medications taken before the radioactive therapy.

ANS: C There is a high incidence of postradiation hypothyroidism after RAI, and the patient should be monitored for symptoms of hypothyroidism. RAI has a delayed response, with the maximum effect not seen for 2 to 3 months, and the patient will continue to take antithyroid medications during this time. The therapeutic dose of radioactive iodine is low enough that no radiation safety precautions are needed.

A 42-year-old female patient is scheduled for transsphenoidal hypophysectomy to treat a pituitary adenoma. During preoperative teaching, the nurse instructs the patient about the need to a. cough and deep breathe every 2 hours postoperatively. b. remain on bed rest for the first 48 hours after the surgery. c. avoid brushing teeth for at least 10 days after the surgery. d. be positioned flat with sandbags at the head postoperatively.

ANS: C To avoid disruption of the suture line, the patient should avoid brushing the teeth for 10 days after surgery. It is not necessary to remain on bed rest after this surgery. Coughing is discouraged because it may cause leakage of cerebrospinal fluid (CSF) from the suture line. The head of the bed should be elevated 30 degrees to reduce pressure on the sella turcica and decrease the risk for headaches.

An 82-year-old patient in a long-term care facility has several medications prescribed. After the patient is newly diagnosed with hypothyroidism, the nurse will need to consult with the health care provider before administering a. docusate (Colace). b. ibuprofen (Motrin). c. diazepam (Valium). d. cefoxitin (Mefoxin).

ANS: C Worsening of mental status and myxedema coma can be precipitated by the use of sedatives, especially in older adults. The nurse should discuss the use of diazepam with the health care provider before administration. The other medications may be given safely to the patient.

A hospitalized diabetic patient received 38 U of NPH insulin at 7:00 AM. At 1:00 PM, the patient has been away from the nursing unit for 2 hours, missing the lunch delivery while awaiting a chest x-ray. To prevent hypoglycemia, the best action by the nurse is to a. save the lunch tray for the patient's later return to the unit. b. ask that diagnostic testing area staff to start a 5% dextrose IV. c. send a glass of milk or orange juice to the patient in the diagnostic testing area. d. request that if testing is further delayed, the patient be returned to the unit to eat.

ANS: D Consistency for mealtimes assists with regulation of blood glucose, so the best option is for the patient to have lunch at the usual time. Waiting to eat until after the procedure is likely to cause hypoglycemia. Administration of an IV solution is unnecessarily invasive for the patient. A glass of milk or juice will keep the patient from becoming hypoglycemic but will cause a rapid rise in blood glucose because of the rapid absorption of the simple carbohydrate in these items.

Which action by a patient indicates that the home health nurse's teaching about glargine and regular insulin has been successful? a. The patient administers the glargine 30 minutes before each meal. b. The patient's family prefills the syringes with the mix of insulins weekly. c. The patient draws up the regular insulin and then the glargine in the same syringe. d. The patient disposes of the open vials of glargine and regular insulin after 4 weeks.

ANS: D Insulin can be stored at room temperature for 4 weeks. Glargine should not be mixed with other insulins or prefilled and stored. Short-acting regular insulin is administered before meals, while glargine is given once daily.

The nursing is interviewing a new patient with diabetes who receives rosiglitazone (Avandia) through a restricted access medication program. What is the most important for the nurse to report immediately to the health care provider? a. The patient's blood pressure is 154/92. b. The patient has a history of emphysema. c. The patient's blood glucose is 86 mg/dL. d. The patient has chest pressure when walking.

ANS: D Rosiglitazone can cause myocardial ischemia. The nurse should immediately notify the health care provider and expect orders to discontinue the medication. There is no urgent need to discuss the other data with the health care provider.

Which finding indicates a need to contact the health care provider before the nurse administers metformin (Glucophage)? a. The patient's blood glucose level is 174 mg/dL. b. The patient has gained 2 lb (0.9 kg) since yesterday. c. The patient is scheduled for a chest x-ray in an hour. d. The patient's blood urea nitrogen (BUN) level is 52 mg/dL.

ANS: D The BUN indicates possible renal failure, and metformin should not be used in patients with renal failure. The other findings are not contraindications to the use of metformin.

A patient with type 2 diabetes is scheduled for a follow-up visit in the clinic several months from now. Which test will the nurse schedule to evaluate the effectiveness of treatment for the patient? a. Urine dipstick for glucose b. Oral glucose tolerance test c. Fasting blood glucose level d. Glycosylated hemoglobin level

ANS: D The glycosylated hemoglobin (A1C or HbA1C) test shows the overall control of glucose over 90 to 120 days. A fasting blood level indicates only the glucose level at one time. Urine glucose testing is not an accurate reflection of blood glucose level and does not reflect the glucose over a prolonged time. Oral glucose tolerance testing is done to diagnose diabetes, but is not used for monitoring glucose control once diabetes has been diagnosed.

The nurse has been teaching a patient with type 2 diabetes about managing blood glucose levels and taking glipizide (Glucotrol). Which patient statement indicates a need for additional teaching? a. "If I overeat at a meal, I will still take the usual dose of medication." b. "Other medications besides the Glucotrol may affect my blood sugar." c. "When I am ill, I may have to take insulin to control my blood sugar." d. "My diabetes won't cause complications because I don't need insulin."

ANS: D The patient should understand that type 2 diabetes places the patient at risk for many complications and that good glucose control is as important when taking oral agents as when using insulin. The other statements are accurate and indicate good understanding of the use of glipizide.

A 28-year-old male patient with type 1 diabetes reports how he manages his exercise and glucose control. Which behavior indicates that the nurse should implement additional teaching? a. The patient always carries hard candies when engaging in exercise. b. The patient goes for a vigorous walk when his glucose is 200 mg/dL. c. The patient has a peanut butter sandwich before going for a bicycle ride. d. The patient increases daily exercise when ketones are present in the urine.

ANS: D When the patient is ketotic, exercise may result in an increase in blood glucose level. Type 1 diabetic patients should be taught to avoid exercise when ketosis is present. The other statements are correct.

Which information is most important for the nurse to communicate rapidly to the health care provider about a patient admitted with possible syndrome of inappropriate antidiuretic hormone (SIADH)? a. The patient has a recent weight gain of 9 lb. b. The patient complains of dyspnea with activity. c. The patient has a urine specific gravity of 1.025. d. The patient has a serum sodium level of 118 mEq/L.

ANS: D A serum sodium of less than 120 mEq/L increases the risk for complications such as seizures and needs rapid correction. The other data are not unusual for a patient with SIADH and do not indicate the need for rapid action

A 23-year-old patient is admitted with diabetes insipidus. Which action will be most appropriate for the registered nurse (RN) to delegate to an experienced licensed practical/vocational nurse (LPN/LVN)? a. Titrate the infusion of 5% dextrose in water. b. Teach the patient how to use desmopressin (DDAVP) nasal spray. c. Assess the patient's hydration status every 8 hours. d. Administer subcutaneous DDAVP.

ANS: D Administration of medications is included in LPN/LVN education and scope of practice. Assessments, patient teaching, and titrating fluid infusions are more complex skills and should be done by the RN.

A 44-year-old patient in the sexually transmitted infection clinic has a positive Venereal Disease Research Laboratory (VDRL) test, but no chancre is visible on assessment. The nurse will plan to send specimens for a. gram stain. b. cytologic studies. c. rapid plasma reagin (RPR) agglutination. d. fluorescent treponemal antibody absorption (FTA-Abs).

ANS: D Because false positives are common with VDRL and RPR testing, FTA-Abs testing is recommended to confirm a diagnosis of syphilis. Gram staining is used for other sexually transmitted infections (STIs) such as gonorrhea and Chlamydia and cytologic studies are used to detect abnormal cells (such as neoplastic cells).

The nurse notes that a patient who has a large cystocele, admitted 10 hours ago, has not yet voided. Which action should the nurse take first? a. Insert a straight catheter per the PRN order. b. Encourage the patient to increase oral fluids. c. Notify the health care provider of the inability to void. d. Use an ultrasound scanner to check for urinary retention.

ANS: D Because urinary retention is common with a large cystocele, the nurse's first action should be to use an ultrasound bladder scanner to check for the presence of urine in the bladder. The other actions may be appropriate, depending on the findings with the bladder scanner

A 31-year-old patient has just been instructed in the treatment for a Chlamydia trachomatis vaginal infection. Which patient statement indicates that the nurse's teaching has been effective? a. "I can purchase an over-the-counter medication to treat this infection." b. "The symptoms are due to the overgrowth of normal vaginal bacteria." c. "The medication will need to be inserted once daily with an applicator." d. "Both my partner and I will need to take the medication for a full week."

ANS: D Chlamydia is a sexually transmitted bacterial infection that requires treatment of both partners with antibiotics for 7 days. The other statements are true for the treatment of Candida albicans infection

The nurse explains to a 37-year-old patient being prepared for colposcopy with a cervical biopsy that the procedure a. involves dilation of the cervix and biopsy of the tissue lining the uterus. b. will take place in a same-day surgery center so that local anesthesia can be used. c. requires that the patient have nothing to eat or drink for 6 hours before the procedure. d. is similar to a speculum examination of the cervix and should result in little discomfort.

ANS: D Colposcopy involves visualization of the cervix with a binocular microscope and is similar to a speculum examination. Anesthesia is not required and fasting is not necessary. A cervical biopsy may cause a minimal amount of pain.

A 49-year-old woman tells the nurse that she is postmenopausal but has occasional spotting. Which initial response by the nurse is most appropriate? a. "A frequent cause of spotting is endometrial cancer." b. "How long has it been since your last menstrual period?" c. "Breakthrough bleeding is not unusual in women your age." d. "Are you using prescription hormone replacement therapy?"

ANS: D In postmenopausal women, a common cause of spotting is hormone therapy (HT). Because breakthrough bleeding may be a sign of problems such as cancer or infection, the nurse would not imply that this is normal. The length of time since the last menstrual period is not relevant to the patient's symptoms. Although endometrial cancer may cause spotting, this information is not appropriate as an initial response

A 19-year-old has been diagnosed with primary dysmenorrhea. How will the nurse suggest that the patient prevent discomfort? a. Avoid aerobic exercise during her menstrual period. b. Use cold packs on the abdomen and back for pain relief. c. Talk with her health care provider about beginning antidepressant therapy. d. Take nonsteroidal antiinflammatory drugs (NSAIDs) when her period starts.

ANS: D NSAIDs should be started as soon as the menstrual period begins and taken at regular intervals during the usual time frame in which pain occurs. Aerobic exercise may help reduce symptoms. Heat therapy, such as warm packs, is recommended for relief of pain. Antidepressant therapy is not a typical treatment for dysmenorrhea

Which finding by the nurse when assessing a patient with a large pituitary adenoma is most important to report to the health care provider? a. Changes in visual field b. Milk leaking from breasts c. Blood glucose 150 mg/dL d. Nausea and projectile vomiting

ANS: D Nausea and projectile vomiting may indicate increased intracranial pressure, which will require rapid actions for diagnosis and treatment. Changes in the visual field, elevated blood glucose, and galactorrhea are common with pituitary adenoma, but these do not require rapid action to prevent life-threatening complications.

19. A 28-year-old patient was recently diagnosed with polycystic ovary syndrome. It is most important for the nurse to teach the patient a. reasons for a total hysterectomy. b. how to decrease facial hair growth. c. ways to reduce the occurrence of acne. d. methods to maintain appropriate weight.

ANS: D Obesity exacerbates the problems associated with polycystic ovary syndrome, such as insulin resistance and type 2 diabetes. The nurse should also address the problems of acne and hirsutism, but these symptoms are lower priority because they do not have long-term health consequences. Although some patients do require total hysterectomy, this is usually performed only after other therapies have been unsuccessful

The nurse will plan to teach a 51-year-old man who is scheduled for an annual physical exam about a(n) a. increased risk for testicular cancer. b. possible changes in erectile function. c. normal decreases in testosterone level. d. prostate specific antigen (PSA) testing.

ANS: D PSA testing may be recommended annually for men, starting at age 50. There is no indication that the other patient teaching topics are appropriate for this patient.

After a 26-year-old patient has been treated for pelvic inflammatory disease, the nurse will plan to teach about a. use of hormone therapy (HT). b. irregularities in the menstrual cycle. c. changes in secondary sex characteristics. d. possible difficulty with becoming pregnant.

ANS: D Pelvic inflammatory disease may cause scarring of the fallopian tubes and result in difficulty in fertilization or implantation of the fertilized egg. Because ovarian function is not affected, the patient will not require HT, have irregular menstrual cycles, or experience changes in secondary sex characteristics.

A woman calls the clinic because she is having an unusually heavy menstrual flow. She tells the nurse that she has saturated three tampons in the past 2 hours. The nurse estimates that the amount of blood loss over the past 2 hours is _____ mL. a. 20 to 30 b. 30 to 40 c. 40 to 60 d. 60 to 90

ANS: D The average tampon absorbs 20 to 30 mL.

A 68-year-old male patient tells the nurse that he is worried because he does not respond to sexual stimulation the same way he did when he was younger. The nurse's best response to the patient's concern is which of the following? a. "Interest in sex frequently decreases as men get older." b. "Many men need additional sexual stimulation with aging." c. "Erectile dysfunction is a common problem with older men." d. "Tell me more about how your sexual response has changed."

ANS: D The initial response by the nurse should be further assessment of the problem. The other statements by the nurse are accurate but may not respond to the patient's concerns.

A 15-year-old child is brought to the emergency department with symptoms of hyperglycemia and is subsequently diagnosed with diabetes. Based on the fact that the child's pancreatic beta cells are being destroyed, the patient would be diagnosed with what type of diabetes? A) Type 1 diabetes B) Type 2 diabetes C) Non-insulin-dependent diabetes D) Prediabetes

Ans: A Feedback: Beta cell destruction is the hallmark of type 1 diabetes. Non-insulin-dependent diabetes is synonymous with type 2 diabetes, which involves insulin resistance and impaired insulin secretion, but not beta cell destruction. Prediabetes is characterized by normal glucose metabolism, but a previous history of hyperglycemia, often during illness or pregnancy.

A 44-year-old female patient with Cushing syndrome is admitted for adrenalectomy. Which intervention by the nurse will be most helpful for a nursing diagnosis of disturbed body image related to changes in appearance? a. Reassure the patient that the physical changes are very common in patients with Cushing syndrome. b. Discuss the use of diet and exercise in controlling the weight gain associated with Cushing syndrome. c. Teach the patient that the metabolic impact of Cushing syndrome is of more importance than appearance. d. Remind the patient that most of the physical changes caused by Cushing syndrome will resolve after surgery.

ANS: D The most reassuring communication to the patient is that the physical and emotional changes caused by the Cushing syndrome will resolve after hormone levels return to normal postoperatively. Reassurance that the physical changes are expected or that there are more serious physiologic problems associated with Cushing syndrome are not therapeutic responses. The patient's physiological changes are caused by the high hormone levels, not by the patient's diet or exercise choices.

The nurse in the women's health clinic has four patients who are waiting to be seen. Which patient should the nurse see first? a. 22-year-old with persistent red-brown vaginal drainage 3 days after having balloon thermotherapy b. 42-year-old with secondary amenorrhea who says that her last menstrual cycle was 3 months ago c. 35-year-old with heavy spotting after having a progestin-containing IUD (Mirena) inserted a month ago d. 19-year-old with menorrhagia who has been using superabsorbent tampons and has fever with weakness

ANS: D The patient's history and clinical manifestations suggest possible toxic shock syndrome, which will require rapid intervention. The symptoms for the other patients are consistent with their diagnoses and do not indicate life-threatening complications

A 27-year-old patient admitted with diabetic ketoacidosis (DKA) has a serum glucose level of 732 mg/dL and serum potassium level of 3.1 mEq/L. Which action prescribed by the health care provider should the nurse take first? a. Place the patient on a cardiac monitor. b. Administer IV potassium supplements. c. Obtain urine glucose and ketone levels. d.Start an insulin infusion at 0.1 units/kg/hr.

ANS:A Hypokalemia can lead to potentially fatal dysrhythmias such as ventricular tachycardia and ventricular fibrillation, which would be detected with electrocardiogram (ECG) monitoring. Because potassium must be infused over at least 1 hour, the nurse should initiate cardiac monitoring before infusion of potassium. Insulin should not be administered without cardiac monitoring because insulin infusion will further decrease potassium levels. Urine glucose and ketone levels are not urgently needed to manage the patient's care.

Which prescribed medication should the nurse administer first to a 60-year-old patient admitted to the emergency department in thyroid storm?

ANSWER A

A nurse is conducting a class on how to self-manage insulin regimens. A patient asks how long a vial of insulin can be stored at room temperature before it "goes bad." What would be the nurse's best answer? A) "If you are going to use up the vial within 1 month it can be kept at room temperature." B) "If a vial of insulin will be used up within 21 days, it may be kept at room temperature." C) "If a vial of insulin will be used up within 2 weeks, it may be kept at room temperature." D) "If a vial of insulin will be used up within 1 week, it may be kept at room temperature."

Ans: A Feedback: If a vial of insulin will be used up within 1 month, it may be kept at room temperature.

A diabetic nurse is working for the summer at a camp for adolescents with diabetes. When providing information on the prevention and management of hypoglycemia, what action should the nurse promote? A) Always carry a form of fast-acting sugar. B) Perform exercise prior to eating whenever possible. C) Eat a meal or snack every 8 hours. D) Check blood sugar at least every 24 hours.

Ans: A Feedback: The following teaching points should be included in information provided to the patient on how to prevent hypoglycemia: Always carry a form of fast-acting sugar, increase food prior to exercise, eat a meal or snack every 4 to 5 hours, and check blood sugar regularly.

A 35-year-old female patient with a possible pituitary adenoma is scheduled for a computed tomography (CT) scan with contrast media. Which patient information is most important for the nurse to communicate to the health care provider before the test? a. Bilateral poor peripheral vision b. Allergies to iodine and shellfish c. Recent weight loss of 20 pounds d. Complaint of ongoing headaches

B

The most recent blood work of a patient with a longstanding diagnosis of type 1 diabetes has shown the presence of microalbuminuria. What is the nurse's most appropriate action? A) Teach the patient about actions to slow the progression of nephropathy. B) Ensure that the patient receives a comprehensive assessment of liver function. C) Determine whether the patient has been using expired insulin. D) Administer a fluid challenge and have the test repeated.

Ans: A Feedback: Clinical nephropathy eventually develops in more than 85% of people with microalbuminuria. As such, educational interventions addressing this microvascular complication are warranted. Expired insulin does not cause nephropathy, and the patient's liver function is not likely affected. There is no indication for the use of a fluid challenge.

A patient presents to the clinic complaining of symptoms that suggest diabetes. What criteria would support checking blood levels for the diagnosis of diabetes? A) Fasting plasma glucose greater than or equal to 126 mg/dL B) Random plasma glucose greater than 150 mg/dL C) Fasting plasma glucose greater than 116 mg/dL on 2 separate occasions D) Random plasma glucose greater than 126 mg/dL

Ans: A Feedback: Criteria for the diagnosis of diabetes include symptoms of diabetes plus random plasma glucose greater than or equal to 200 mg/dL, or a fasting plasma glucose greater than or equal to 126 mg/dL.

A patient with a longstanding diagnosis of type 1 diabetes has a history of poor glycemic control. The nurse recognizes the need to assess the patient for signs and symptoms of peripheral neuropathy. Peripheral neuropathy constitutes a risk for what nursing diagnosis? A) Infection B) Acute pain C) Acute confusion D) Impaired urinary elimination

Ans: A Feedback: Decreased sensations of pain and temperature place patients with neuropathy at increased risk for injury and undetected foot infections. The neurologic changes associated with peripheral neuropathy do not normally result in pain, confusion, or impairments in urinary function.

A patient has been brought to the emergency department by paramedics after being found unconscious. The patient's Medic Alert bracelet indicates that the patient has type 1 diabetes and the patient's blood glucose is 22 mg/dL (1.2 mmol/L). The nurse should anticipate what intervention? A) IV administration of 50% dextrose in water B) Subcutaneous administration of 10 units of Humalog C) Subcutaneous administration of 12 to 15 units of regular insulin D) IV bolus of 5% dextrose in 0.45% NaCl

Ans: A Feedback: In hospitals and emergency departments, for patients who are unconscious or cannot swallow, 25 to 50 mL of 50% dextrose in water (D50W) may be administered IV for the treatment of hypoglycemia. Five percent dextrose would be inadequate and insulin would exacerbate the patient's condition.

A patient has been living with type 2 diabetes for several years, and the nurse realizes that the patient is likely to have minimal contact with the health care system. In order to ensure that the patient maintains adequate blood sugar control over the long term, the nurse should recommend which of the following? A) Participation in a support group for persons with diabetes B) Regular consultation of websites that address diabetes management C) Weekly telephone "check-ins" with an endocrinologist D) Participation in clinical trials relating to antihyperglycemics

Ans: A Feedback: Participation in support groups is encouraged for patients who have had diabetes for many years as well as for those who are newly diagnosed. This is more interactive and instructive than simply consulting websites. Weekly telephone contact with an endocrinologist is not realistic in most cases. Participation in research trials may or may not be beneficial and appropriate, depending on patients' circumstances.

A diabetes nurse educator is teaching a group of patients with type 1 diabetes about "sick day rules." What guideline applies to periods of illness in a diabetic patient? A) Do not eliminate insulin when nauseated and vomiting. B) Report elevated glucose levels greater than 150 mg/dL. C) Eat three substantial meals a day, if possible. D) Reduce food intake and insulin doses in times of illness.

Ans: A Feedback: The most important issue to teach patients with diabetes who become ill is not to eliminate insulin doses when nausea and vomiting occur. Rather, they should take their usual insulin or oral hypoglycemic agent dose, then attempt to consume frequent, small portions of carbohydrates. In general, blood sugar levels will rise but should be reported if they are greater than 300 mg/dL.

A diabetic educator is discussing "sick day rules" with a newly diagnosed type 1 diabetic. The educator is aware that the patient will require further teaching when the patient states what? A) "I will not take my insulin on the days when I am sick, but I will certainly check my blood sugar every 2 hours." B) "If I cannot eat a meal, I will eat a soft food such as soup, gelatin, or pudding six to eight times a day." C) "I will call the doctor if I am not able to keep liquids in my body due to vomiting or diarrhea." D) "I will call the doctor if my blood sugar is over 300 mg/dL or if I have ketones in my urine."

Ans: A Feedback: The nurse must explanation the "sick day rules" again to the patient who plans to stop taking insulin when sick. The nurse should emphasize that the patient should take insulin agents as usual and test one's blood sugar and urine ketones every 3 to 4 hours. In fact, insulin-requiring patients may need supplemental doses of regular insulin every 3 to 4 hours. The patient should report elevated glucose levels (greater than 300 mg/dL or as otherwise instructed) or urine ketones to the physician. If the patient is not able to eat normally, the patient should be instructed to substitute soft foods such a gelatin, soup, and pudding. If vomiting, diarrhea, or fever persists, the patient should have an intake of liquids every 30 to 60 minutes to prevent dehydration.

A nurse is caring for a patient newly diagnosed with type 1 diabetes. The nurse is educating the patient about self-administration of insulin in the home setting. The nurse should teach the patient to do which of the following? A) Avoid using the same injection site more than once in 2 to 3 weeks. B) Avoid mixing more than one type of insulin in a syringe. C) Cleanse the injection site thoroughly with alcohol prior to injecting. D) Inject at a 45º angle.

Ans: A Feedback: To prevent lipodystrophy, the patient should try not to use the same site more than once in 2 to 3 weeks. Mixing different types of insulin in a syringe is acceptable, within specific guidelines, and the needle is usually inserted at a 90º angle. Cleansing the injection site with alcohol is optional.

A 40-year-old male patient has been newly diagnosed with type 2 diabetes mellitus. Which information about the patient will be most useful to the nurse who is helping the patient develop strategies for successful adaptation to this disease? a. Ideal weight b. Value system c. Activity level d. Visual changes

B

The nurse is discussing macrovascular complications of diabetes with a patient. The nurse would address what topic during this dialogue? A) The need for frequent eye examinations for patients with diabetes B) The fact that patients with diabetes have an elevated risk of myocardial infarction C) The relationship between kidney function and blood glucose levels D) The need to monitor urine for the presence of albumin

Ans: B Feedback: Myocardial infarction and stroke are considered macrovascular complications of diabetes, while the effects on vision and renal function are considered to be microvascular.

An occupational health nurse is screening a group of workers for diabetes. What statement should the nurse interpret as suggestive of diabetes? A) "I've always been a fan of sweet foods, but lately I'm turned off by them." B) "Lately, I drink and drink and can't seem to quench my thirst." C) "No matter how much sleep I get, it seems to take me hours to wake up." D) "When I went to the washroom the last few days, my urine smelled odd."

Ans: B Feedback: Classic clinical manifestations of diabetes include the "three Ps": polyuria, polydipsia, and polyphagia. Lack of interest in sweet foods, fatigue, and foul-smelling urine are not suggestive of diabetes.

A patient with type 2 diabetes achieves adequate glycemic control through diet and exercise. Upon being admitted to the hospital for a cholecystectomy, however, the patient has required insulin injections on two occasions. The nurse would identify what likely cause for this short-term change in treatment? A) Alterations in bile metabolism and release have likely caused hyperglycemia. B) Stress has likely caused an increase in the patient's blood sugar levels. C) The patient has likely overestimated her ability to control her diabetes using nonpharmacologic measures. D) The patient's volatile fluid balance surrounding surgery has likely caused unstable blood sugars.

Ans: B Feedback: During periods of physiologic stress, such as surgery, blood glucose levels tend to increase, because levels of stress hormones (epinephrine, norepinephrine, glucagon, cortisol, and growth hormone) increase. The patient's need for insulin is unrelated to the action of bile, the patient's overestimation of previous blood sugar control, or fluid imbalance.

A diabetic patient calls the clinic complaining of having a "flu bug." The nurse tells him to take his regular dose of insulin. What else should the nurse tell the patient? A) "Make sure to stick to your normal diet." B) "Try to eat small amounts of carbs, if possible." C) "Ensure that you check your blood glucose every hour." D) "For now, check your urine for ketones every 8 hours."

Ans: B Feedback: For prevention of DKA related to illness, the patient should attempt to consume frequent small portions of carbohydrates (including foods usually avoided, such as juices, regular sodas, and gelatin). Drinking fluids every hour is important to prevent dehydration. Blood glucose and urine ketones must be assessed every 3 to 4 hours.

A patient with a history of type 1 diabetes has just been admitted to the critical care unit (CCU) for diabetic ketoacidosis. The CCU nurse should prioritize what assessment during the patient's initial phase of treatment? A) Monitoring the patient for dysrhythmias B) Maintaining and monitoring the patient's fluid balance C) Assessing the patient's level of consciousness D) Assessing the patient for signs and symptoms of venous thromboembolism

Ans: B Feedback: In addition to treating hyperglycemia, management of DKA is aimed at correcting dehydration, electrolyte loss, and acidosis before correcting the hyperglycemia with insulin. The nurse should monitor the patient for dysrhythmias, decreased LOC and VTE, but restoration and maintenance of fluid balance is the highest priority.

A nurse is caring for a patient with type 1 diabetes who is being discharged home tomorrow. What is the best way to assess the patient's ability to prepare and self-administer insulin? A) Ask the patient to describe the process in detail. B) Observe the patient drawing up and administering the insulin. C) Provide a health education session reviewing the main points of insulin delivery. D) Review the patient's first hemoglobin A1C result after discharge.

Ans: B Feedback: Nurses should assess the patient's ability to perform diabetes related self-care as soon as possible during the hospitalization or office visit to determine whether the patient requires further diabetes teaching. While consulting a home care nurse is beneficial, an initial assessment should be performed during the hospitalization or office visit. Nurses should directly observe the patient performing the skills such as insulin preparation and infection, blood glucose monitoring, and foot care. Simply questioning the patient about these skills without actually observing performance of the skill is not sufficient. Further education does not guarantee learning.

A medical nurse is caring for a patient with type 1 diabetes. The patient's medication administration record includes the administration of regular insulin three times daily. Knowing that the patient's lunch tray will arrive at 11:45, when should the nurse administer the patient's insulin? A) 10:45 B) 11:15 C) 11:45 D) 11:50

Ans: B Feedback: Regular insulin is usually administered 20-30 min before a meal. Earlier administration creates a risk for hypoglycemia; later administration creates a risk for hyperglycemia.

A diabetes nurse is assessing a patient's knowledge of self-care skills. What would be the most appropriate way for the educator to assess the patient's knowledge of nutritional therapy in diabetes? A) Ask the patient to describe an optimally healthy meal. B) Ask the patient to keep a food diary and review it with the nurse. C) Ask the patient's family what he typically eats. D) Ask the patient to describe a typical day's food intake.

Ans: B Feedback: Reviewing the patient's actual food intake is the most accurate method of gauging the patient's diet.

A patient has just been diagnosed with type 2 diabetes. The physician has prescribed an oral antidiabetic agent that will inhibit the production of glucose by the liver and thereby aid in the control of blood glucose. What type of oral antidiabetic agent did the physician prescribe for this patient? A) A sulfonylurea B) A biguanide C) A thiazolidinedione D) An alpha glucosidase inhibitor

Ans: B Feedback: Sulfonylureas exert their primary action by directly stimulating the pancreas to secrete insulin and therefore require a functioning pancreas to be effective. Biguanides inhibit the production of glucose by the liver and are in used in type 2 diabetes to control blood glucose levels. Thiazolidinediones enhance insulin action at the receptor site without increasing insulin secretion from the beta cells of the pancreas. Alpha glucosidase inhibitors work by delaying the absorption of glucose in the intestinal system, resulting in a lower postprandial blood glucose level.

An elderly patient comes to the clinic with her daughter. The patient is a diabetic and is concerned about foot care. The nurse goes over foot care with the patient and her daughter as the nurse realizes that foot care is extremely important. Why would the nurse feel that foot care is so important to this patient? A) An elderly patient with foot ulcers experiences severe foot pain due to the diabetic polyneuropathy. B) Avoiding foot ulcers may mean the difference between institutionalization and continued independent living. C) Hypoglycemia is linked with a risk for falls; this risk is elevated in older adults with diabetes. D) Oral antihyperglycemics have the possible adverse effect of decreased circulation to the lower extremities.

Ans: B Feedback: The nurse recognizes that providing information on the long-term complications—especially foot and eye problems—associated with diabetes is important. Avoiding amputation through early detection of foot ulcers may mean the difference between institutionalization and continued independent living for the elderly person with diabetes. While the nurse recognizes that hypoglycemia is a dangerous situation and may lead to falls, hypoglycemia is not directly connected to the importance of foot care. Decrease in circulation is related to vascular changes and is not associated with drugs administered for diabetes.

A patient is brought to the emergency department by the paramedics. The patient is a type 2 diabetic and is experiencing HHS. The nurse should identify what components of HHS? Select all that apply. A) Leukocytosis B) Glycosuria C) Dehydration D) Hypernatremia E) Hyperglycemia

Ans: B, C, D, E Feedback: In HHS, persistent hyperglycemia causes osmotic diuresis, which results in losses of water and electrolytes. To maintain osmotic equilibrium, water shifts from the intracellular fluid space to the extracellular fluid space. With glycosuria and dehydration, hypernatremia and increased osmolarity occur. Leukocytosis does not take place.

A patient newly diagnosed with type 2 diabetes is attending a nutrition class. What general guideline would be important to teach the patients at this class? A) Low fat generally indicates low sugar. B) Protein should constitute 30% to 40% of caloric intake. C) Most calories should be derived from carbohydrates. D) Animal fats should be eliminated from the diet.

Ans: C Feedback: Currently, the ADA and the Academy of Nutrition and Dietetics (formerly the American Dietetic Association) recommend that for all levels of caloric intake, 50% to 60% of calories should be derived from carbohydrates, 20% to 30% from fat, and the remaining 10% to 20% from protein.Low fat does not automatically mean low sugar. Dietary animal fat does not need to be eliminated from the diet.

A nurse is providing health education to an adolescent newly diagnosed with type 1 diabetes mellitus and her family. The nurse teaches the patient and family that which of the following nonpharmacologic measures will decrease the body's need for insulin? A) Adequate sleep B) Low stimulation C) Exercise D) Low-fat diet

Ans: C Feedback: Exercise lowers blood glucose, increases levels of HDLs, and decreases total cholesterol and triglyceride levels. Low fat intake and low levels of stimulation do not reduce a patient's need for insulin. Adequate sleep is beneficial in reducing stress, but does not have an effect that is pronounced as that of exercise.

A patient with type 1 diabetes mellitus is seeing the nurse to review foot care. What would be a priority instruction for the nurse to give the patient? A) Examine feet weekly for redness, blisters, and abrasions. B) Avoid the use of moisturizing lotions. C) Avoid hot-water bottles and heating pads. D) Dry feet vigorously after each bath.

Ans: C Feedback: High-risk behaviors, such as walking barefoot, using heating pads on the feet, wearing open-toed shoes, soaking the feet, and shaving calluses, should be avoided. Socks should be worn for warmth. Feet should be examined each day for cuts, blisters, swelling, redness, tenderness, and abrasions. Lotion should be applied to dry feet but never between the toes. After a bath, the patient should gently, not vigorously, pat feet dry to avoid injury.

A student with diabetes tells the school nurse that he is feeling nervous and hungry. The nurse assesses the child and finds he has tachycardia and is diaphoretic with a blood glucose level of 50 mg/dL (2.8 mmol/L). What should the school nurse administer? A) A combination of protein and carbohydrates, such as a small cup of yogurt B) Two teaspoons of sugar dissolved in a cup of apple juice C) Half of a cup of juice, followed by cheese and crackers D) Half a sandwich with a protein-based filling

Ans: C Feedback: Initial treatment for hypoglycemia is 15 g concentrated carbohydrate, such as two or three glucose tablets, 1 tube glucose gel, or 0.5 cup juice. After initial treatment, the nurse should follow with a snack including starch and protein, such as cheese and crackers, milk and crackers, or half of a sandwich. It is unnecessary to add sugar to juice, even it if is labeled as unsweetened juice, because the fruit sugar in juice contains enough simple carbohydrate to raise the blood glucose level and additional sugar may result in a sharp rise in blood sugar that will last for several hours.

A diabetes educator is teaching a patient about type 2 diabetes. The educator recognizes that the patient understands the primary treatment for type 2 diabetes when the patient states what? A) "I read that a pancreas transplant will provide a cure for my diabetes." B) "I will take my oral antidiabetic agents when my morning blood sugar is high." C) "I will make sure to follow the weight loss plan designed by the dietitian." D) "I will make sure I call the diabetes educator when I have questions about my insulin."

Ans: C Feedback: Insulin resistance is associated with obesity; thus the primary treatment of type 2 diabetes is weight loss. Oral antidiabetic agents may be added if diet and exercise are not successful in controlling blood glucose levels. If maximum doses of a single category of oral agents fail to reduce glucose levels to satisfactory levels, additional oral agents may be used. Some patients may require insulin on an ongoing basis or on a temporary basis during times of acute psychological stress, but it is not the central component of type 2 treatment. Pancreas transplantation is associated with type 1 diabetes.

A patient with type 1 diabetes has told the nurse that his most recent urine test for ketones was positive. What is the nurse's most plausible conclusion based on this assessment finding? A) The patient should withhold his next scheduled dose of insulin. B) The patient should promptly eat some protein and carbohydrates. C) The patient's insulin levels are inadequate. D) The patient would benefit from a dose of metformin (Glucophage).

Ans: C Feedback: Ketones in the urine signal that there is a deficiency of insulin and that control of type 1 diabetes is deteriorating. Withholding insulin or eating food would exacerbate the patient's ketonuria. Metformin will not cause short-term resolution of hyperglycemia.

A physician has explained to a patient that he has developed diabetic neuropathy in his right foot. Later that day, the patient asks the nurse what causes diabetic neuropathy. What would be the nurse's best response? A) "Research has shown that diabetic neuropathy is caused by fluctuations in blood sugar that have gone on for years." B) "The cause is not known for sure but it is thought to have something to do with ketoacidosis." C) "The cause is not known for sure but it is thought to involve elevated blood glucose levels over a period of years." D) "Research has shown that diabetic neuropathy is caused by a combination of elevated glucose levels and elevated ketone levels."

Ans: C Feedback: The etiology of neuropathy may involve elevated blood glucose levels over a period of years. High blood sugars (rather than fluctuations or variations in blood sugars) are thought to be responsible. Ketones and ketoacidosis are not direct causes of neuropathies.

Which of the following patients with type 1 diabetes is most likely to experience adequate glucose control? A) A patient who skips breakfast when his glucose reading is greater than 220 mg/dL B) A patient who never deviates from her prescribed dose of insulin C) A patient who adheres closely to a meal plan and meal schedule D) A patient who eliminates carbohydrates from his daily intake

Ans: C Feedback: The therapeutic goal for diabetes management is to achieve normal blood glucose levels without hypoglycemia. Therefore, diabetes management involves constant assessment and modification of the treatment plan by health professionals and daily adjustments in therapy (possibly including insulin) by patients. For patients who require insulin to help control blood glucose levels, maintaining consistency in the amount of calories and carbohydrates ingested at meals is essential. In addition, consistency in the approximate time intervals between meals, and the snacks, help maintain overall glucose control. Skipping meals is never advisable for person with type 1 diabetes.

A school nurse is teaching a group of high school students about risk factors for diabetes. Which of the following actions has the greatest potential to reduce an individual's risk for developing diabetes? A) Have blood glucose levels checked annually. B) Stop using tobacco in any form. C) Undergo eye examinations regularly. D) Lose weight, if obese.

Ans: D Feedback: Obesity is a major modifiable risk factor for diabetes. Smoking is not a direct risk factor for the disease. Eye examinations are necessary for persons who have been diagnosed with diabetes, but they do not screen for the disease or prevent it. Similarly, blood glucose checks do not prevent the diabetes.

A nurse is assessing a patient who has diabetes for the presence of peripheral neuropathy. The nurse should question the patient about what sign or symptom that would suggest the possible development of peripheral neuropathy? A) Persistently cold feet B) Pain that does not respond to analgesia C) Acute pain, unrelieved by rest D) The presence of a tingling sensation

Ans: D Feedback: Although approximately half of patients with diabetic neuropathy do not have symptoms, initial symptoms may include paresthesias (prickling, tingling, or heightened sensation) and burning sensations (especially at night). Cold and intense pain are atypical early signs of this complication.

A patient has received a diagnosis of type 2 diabetes. The diabetes nurse has made contact with the patient and will implement a program of health education. What is the nurse's priority action? A) Ensure that the patient understands the basic pathophysiology of diabetes. B) Identify the patient's body mass index. C) Teach the patient "survival skills" for diabetes. D) Assess the patient's readiness to learn.

Ans: D Feedback: Before initiating diabetes education, the nurse assesses the patient's (and family's) readiness to learn. This must precede other physiologic assessments (such as BMI) and providing health education.

A diabetes nurse educator is presenting the American Diabetes Association (ADA) recommendations for levels of caloric intake. What do the ADA's recommendations include? A) 10% of calories from carbohydrates, 50% from fat, and the remaining 40% from protein B) 10% to 20% of calories from carbohydrates, 20% to 30% from fat, and the remaining 50% to 60% from protein C) 20% to 30% of calories from carbohydrates, 50% to 60% from fat, and the remaining 10% to 20% from protein D) 50% to 60% of calories from carbohydrates, 20% to 30% from fat, and the remaining 10% to 20% from protein

Ans: D Feedback: Currently, the ADA and the Academy of Nutrition and Dietetics (formerly the American Dietetic Association) recommend that for all levels of caloric intake, 50% to 60% of calories come from carbohydrates, 20% to 30% from fat, and the remaining 10% to 20% from protein.

A medical nurse is aware of the need to screen specific patients for their risk of hyperglycemic hyperosmolar syndrome (HHS). In what patient population does hyperosmolar nonketotic syndrome most often occur? A) Patients who are obese and who have no known history of diabetes B) Patients with type 1 diabetes and poor dietary control C) Adolescents with type 2 diabetes and sporadic use of antihyperglycemics D) Middle-aged or older people with either type 2 diabetes or no known history of diabetes

Ans: D Feedback: HHS occurs most often in older people (50 to 70 years of age) who have no known history of diabetes or who have type 2 diabetes.

A nurse is teaching basic "survival skills" to a patient newly diagnosed with type 1 diabetes. What topic should the nurse address? A) Signs and symptoms of diabetic nephropathy B) Management of diabetic ketoacidosis C) Effects of surgery and pregnancy on blood sugar levels D) Recognition of hypoglycemia and hyperglycemia

Ans: D Feedback: It is imperative that newly diagnosed patients know the signs and symptoms and management of hypo- and hyperglycemia. The other listed topics are valid points for education, but are not components of the patient's immediate "survival skills" following a new diagnosis.

A patient has just been prescribed furosemide (Lasix). After reviewing the patient's medication history, what drug would cause the nurse concern when taken with furosemide (Lasix)? A) Acetaminophen B) Ferrous sulfate (Feosol) C) Naproxen sodium (Naprosyn) D) Ampicillin

Ans: D Feedback: Metformin has the potential to be nephrotoxic; consequently, the nurse should monitor the patient's renal function. This drug does not typically affect patients' neutrophils, liver function, or cognition.

An elderly male patient is experiencing difficulty in initiating voiding and a feeling of incomplete bladder emptying. What causes these symptoms in benign prostatic hyperplasia (BPH)? A. Obstruction of the urethra B. Untreated chronic prostatitis C. Decreased bladder compliance D. Excessive secretion of testosterone

Answer: A BPH is a benign enlargement of the prostate gland. The enlargement of the prostate gradually compresses the urethra, eventually causing partial or complete obstruction. Compression of the urethra ultimately leads to the development of clinical symptoms.

An older adult patient with type 2 diabetes is brought to the emergency department by his daughter. The patient is found to have a blood glucose level of 623 mg/dL. The patient's daughter reports that the patient recently had a gastrointestinal virus and has been confused for the last 3 hours. The diagnosis of hyperglycemic hyperosmolar syndrome (HHS) is made. What nursing action would be a priority? A) Administration of antihypertensive medications B) Administering sodium bicarbonate intravenously C) Reversing acidosis by administering insulin D) Fluid and electrolyte replacement

Ans: D Feedback: The overall approach to HHS includes fluid replacement, correction of electrolyte imbalances, and insulin administration. Antihypertensive medications are not indicated, as hypotension generally accompanies HHS due to dehydration. Sodium bicarbonate is not administered to patients with HHS, as their plasma bicarbonate level is usually normal. Insulin administration plays a less important role in the treatment of HHS because it is not needed for reversal of acidosis, as in diabetic ketoacidosis (DKA).

A newly admitted patient with type 1 diabetes asks the nurse what caused her diabetes. When the nurse is explaining to the patient the etiology of type 1 diabetes, what process should the nurse describe? A) "The tissues in your body are resistant to the action of insulin, making the glucose levels in your blood increase." B) "Damage to your pancreas causes an increase in the amount of glucose that it releases, and there is not enough insulin to control it." C) "The amount of glucose that your body makes overwhelms your pancreas and decreases your production of insulin." D) "Destruction of special cells in the pancreas causes a decrease in insulin production. Glucose levels rise because insulin normally breaks it down."

Ans: D Feedback: Type 1 diabetes is characterized by the destruction of pancreatic beta cells, resulting in decreased insulin production, unchecked glucose production by the liver, and fasting hyperglycemia. Also, glucose derived from food cannot be stored in the liver and remains circulating in the blood, which leads to postprandial hyperglycemia. Type 2 diabetes involves insulin resistance and impaired insulin secretion. The body does not "make" glucose.

A patient with type 2 diabetes has been managing his blood glucose levels using diet and metformin (Glucophage). Following an ordered increase in the patient's daily dose of metformin, the nurse should prioritize which of the following assessments? A) Monitoring the patient's neutrophil levels B) Assessing the patient for signs of impaired liver function C) Monitoring the patient's level of consciousness and behavior D) Reviewing the patient's creatinine and BUN levels

Ans: D) To avoid rebound edema

Six weeks after a vasectomy, the patient has a sperm-free semen analysis result. What should the couple conclude? A. The patient is sterile. B. One additional semen free test is required. C. Alternative contraception is still required. D. Intercourse should be avoided for 2 more weeks.

Answer: A After vasectomy, the patient should not notice any difference in the look or feel of the ejaculate because its major component is seminal and prostatic fluid. The patient should use an alternative form of contraception until semen examination reveals no sperm. This usually requires at least 10 ejaculations or 6 weeks to evacuate sperm distal to the surgical site.

Where is the urethra opening located in cases of hypospadias? A. On the ventral surface of the penis B. On the dorsal surface of the penis C. At the upper portion of the testicular vault D. Within a bladder extrophy

Answer: A Hypospadias is a urologic abnormality in which the urethral opening is located on the ventral surface of the penis anywhere from the corona to the perineum. Hormonal influences in utero, environmental factors, and genetic factors are possible causes.

Which instructions should you include in the discharge instructions for a patient with epididymitis? A. Refrain from sexual intercourse. B. Continue to participate in liberal exercise. C. Take frequent sitz baths. D. Eat a low-salt diet.

Answer: A Patients with epididymitis should be encouraged to refrain from sexual intercourse during the acute phase. Conservative treatment consists of bed rest with elevation of the scrotum, use of ice packs, and analgesics. Ambulation places the scrotum in a dependent position and increases pain. Salt does not affect epididymitis recovery.

During the bath of a male patient, the nursing assistive personnel reports to you that the patient's foreskin cannot be retracted over the head of the penis. You recognize this condition as A. phimosis. B. prostatitis. C. priapism. D. epispadias.

Answer: A Phimosis is tightness or constriction of the foreskin around the head of the penis, which makes retraction difficult. Paraphimosis is tightness of the foreskin resulting in the inability to pull it forward from a retracted position and preventing normal return over the glans. Prostatitis is inflammation of the prostate, and epispadias occurs when the urethral opening is on the dorsal surface of the penis.

What is an erection lasting longer than 6 hours called? A. Priapism B. Peyronie's disease C. Hydrocele D. Hypospadias

Answer: A Priapism is a painful erection lasting longer than 6 hours. It is caused by an obstruction of the venous outflow in the penis. The condition may constitute a medical emergency. Causes of priapism include thrombosis of the corpus cavernosal veins, leukemia, sickle cell anemia, diabetes mellitus, degenerative lesions of the spine, neoplasms of the brain or spinal cord, vasoactive medications injected into the corpus cavernosa, and medications (e.g., sildenafil, cocaine, trazodone).

What is the primary purpose of a three-way urinary catheter after a transurethral resection of the prostate (TURP)? A. Promote hemostasis and drainage of clots B. Relieve bladder spasms C. Reduce edema D. Increase bladder tone

Answer: A A large three-way indwelling catheter with a 30-mL balloon is inserted into the bladder after the procedure to provide hemostasis and to facilitate urinary drainage. The bladder is irrigated, either continuously or intermittently, usually for the first 24 hours to prevent obstruction from mucus and blood clots.

A patient scheduled for a prostatectomy for prostate cancer expresses the fear that he will have erectile dysfunction. In responding to the patient, you should keep in mind that A. erectile dysfunction can occur even with a nerve-sparing procedure. B. retrograde ejaculation affects sexual functioning more frequently than erectile dysfunction. C. the most common complication of this surgery is postoperative bowel incontinence. D. preoperative sexual functioning is the most important factor in determining postoperative erectile dysfunction.

Answer: A A major complication after a prostatectomy (even with nerve-sparing procedures) is erectile dysfunction

After you perform teaching regarding medication therapy for erectile dysfunction (ED), you know the teaching was successful when the patient states, A. "I will take the drug once each day about an hour before sexual activity." B. "I will take the drug two times each day but not more than three." C. "I can take the drug in the morning, and I will be able to have an erection anytime throughout the day." D. "There are no precautions to take while I am on this medication."

Answer: A Medication is taken orally about 1 hour before sexual activity, but not more than once each day. These drugs have been found to be generally safe and effective for the treatment of most types of ED.

During examination of a 67-year-old man, the nurse notes bilateral enlargement of the breasts. The nurse's first action should be to a. palpate the breasts for the presence of any discrete lumps. b. explain that this is a temporary condition caused by hormonal changes. c. refer the patient for mammography and biopsy of the breast tissue. d. teach the patient about dietary changes to reduce the breast size.

Answer: A Rationale: If discrete, circumscribed lumps are present, the patient should be referred for further testing to determine whether breast cancer is present. Gynecomastia is usually a temporary change, but it can be caused by breast cancer. Mammography and biopsy will not be needed unless lumps are present in the breast tissue. Dietary changes will not affect the condition. Cognitive Level: Application Text Reference: p. 1348 Nursing Process: Implementation NCLEX: Physiological Integrity

Which of these nursing interventions for the patient who has had right-sided breast-conservation surgery and an axillary lymph node dissection is appropriate to assign to an LPN/LVN? a. Administering an analgesic 30 minutes before the scheduled arm exercises b. Teaching the patient how to avoid injury to the right arm c. Assessment of the patient's range of motion for the right arm d. Evaluation of the patient's understanding of discharge instructions about drain care

Answer: A Rationale: LPN/LVN education and scope of practice include administration and evaluation of the effects of analgesics. Assessment, teaching, and evaluation of a patient's understanding of instructions are more complex tasks that are more appropriate to RN-level education and scope of practice. Cognitive Level: Application Text Reference: pp. 1356-1361 Nursing Process: Implementation NCLEX: Safe and Effective Care Environment

A 38-year-old woman is scheduled for a breast-conservation therapy with a lumpectomy. As the nurse prepares her for surgery, she begins to cry and says, "I just do not know how to handle all of this." An appropriate response to the patient by the nurse is, a. "Would you like to talk about how you are feeling right now?" b. "I can see you are really upset. Would you like to be alone for a while?" c. "The important thing is that the tumor was found and is going to be removed." d. "With this surgery you will have very little change in the appearance of your breast."

Answer: A Rationale: The nurse encourages the patient to express feelings about the diagnosis and surgery. The response beginning, "I can see you are really upset" may indicate that the nurse is uncomfortable being with the patient while she is upset. The response beginning, "The important thing is that the tumor was found" places the nurse's value system above the patient's current concerns. And the response, "With this surgery you will have very little change in the appearance of your breast" does not address all the patient's possible concerns and is not true. Cognitive Level: Application Text Reference: p. 1360 Nursing Process: Implementation NCLEX: Psychosocial Integrity

After the nurse completes discharge teaching for a patient who has had a left modified radical mastectomy and lymph node dissection, which statement by the patient indicates that no further teaching is needed? a. "I will avoid reaching over the stove with my left hand." b. "I will need to do breast self-examination on my right breast monthly." c. "I will keep my left arm elevated until I go to bed." d. "I will remember to use my right arm and to rest the left one."

Answer: A Rationale: The patient should avoid any activity that might injure the left arm, such as reaching over a burner. Breast self-examination should be done to the right breast and the left mastectomy site. The left arm should be elevated when the patient is lying down also. The left arm should be used to improve range of motion and function. Cognitive Level: Application Text Reference: pp. 1359-1360 Nursing Process: Evaluation NCLEX: Physiological Integrity

Which ethnic group has the highest incidence of prostate cancer? A. African Americans B. Asians C. Whites D. Hispanics

Answer: A The incidence of prostate cancer worldwide is higher among African Americans than in any other ethnic group. The reasons for the higher rate are unknown.

A patient with benign prostatic hyperplasia is scheduled for TURP. After you assess the patient's knowledge of the procedure and its effects on reproductive function, you determine a need for further teaching when the patient says, A. "It is possible that I'll be sterile after this procedure." B. "I understand that some retrograde ejaculation may occur." C. "I will have a catheter for several days to keep my urinary system open." D. "It is unlikely that I would become impotent from this procedure."

Answer: A The patient will not be sterile; he may experience retrograde ejaculation and some erectile dysfunction. It is unlikely he will become impotent. He will need a catheter.

You notice that the patient's urinary drainage 4 hours after TURP is redder than 1 hour ago. What is your priority intervention? A. To increase the rate of bladder irrigation B. To manually irrigate the urinary catheter C. To notify the physician D. To obtain vital signs

Answer: A With CBI, irrigating solution is continuously infused and drained from the bladder. The rate of infusion is based on the color of drainage. Ideally, urine drainage should be light pink without clots. Continuously monitor the inflow and outflow of the irrigant.

A patient asks you, "How can I decrease my risk of prostate cancer?" You teach the patient to avoid which foods (select all that apply)? A. Red meat B. High-fat dairy products C. Fruits D. Vegetables E. Chicken

Answer: A, B Dietary factors may be associated with prostate cancer. A diet high in red meat and high-fat dairy products, along with a low intake of vegetables and fruits, may increase the risk of prostate cancer.

You expect which finding in a patient as a complication of prostatic hyperplasia? A. Dysuria B. Hematuria C. Urinary retention D. Urinary frequency

Answer: C Obstructive symptoms caused by prostate enlargement include a decrease in the caliber and force of the urinary stream, difficulty in initiating voiding, intermittency (stopping and starting stream several times while voiding), and dribbling at the end of urination. These symptoms result from urinary retention.

Which factors place a patient at high risk for prostate cancer (select all that apply)? A. Age older than 65 years B. Asian or Native American ethnicity C. Personal history of BPH D. Brother diagnosed and treated for prostate cancer E. History of undescended testicle and testicular cancer

Answer: A,D Age, ethnicity, and family history are known risk factors for prostate cancer. The incidence of prostate cancer rises markedly after age 50, and more than 66% of men diagnosed are older than 65 years. The incidence of prostate cancer worldwide is higher among African Americans than in any other ethnic group. A family history of prostate cancer, especially first-degree relatives (fathers, brothers), is associated with an increased risk.

Which childhood disease is often associated with orchitis? A. Measles B. Mumps C. Strep throat D. Chickenpox

Answer: B Orchitis refers to an acute inflammation of the testis. In orchitis, the testis is painful, tender, and swollen. It usually occurs after an episode of bacterial or viral infection, such as mumps. The other disorders are not associated with mumps.

Which diagnostic evaluation is used for a patient with a suspected testicular cancer? A. Prostate-specific antigen (PSA) B. α-Fetoprotein (AFP) C. Complete blood cell count D. Urine and semen analyses

Answer: B Palpation of the scrotal contents is the first step in diagnosing testicular cancer. A cancerous mass is firm and does not transilluminate. Ultrasound of the testes is indicated when testicular cancer (e.g., palpable mass) is suspected or when persistent or painful testicular swelling is present. If testicular cancer is suspected, blood is obtained to determine the serum levels of AFP, lactate dehydrogenase (LDH), and human chorionic gonadotropin (hCG).

For the patient with inoperable prostate cancer, you expect the physician to order which type of hormone? A. Gonadotropin-releasing hormone B. Androgen deprivation C. Luteinizing hormone D. Estrogen

Answer: B Prostate cancer growth largely depends on androgens, and androgen deprivation is a primary therapeutic approach for some men with prostatic cancer. Hormone therapy, also known as androgen-deprivation therapy (ADT), focuses on reducing the levels of circulating androgens to diminish tumor growth.

Which fact in the patient's history could be related to the presence of testicular cancer? A. Epispadias B. Cryptorchidism C. Hernia repair D. Uncircumcised penis

Answer: B The incidence of testicular cancer is four times higher among white men (especially those of Scandinavian descent) than African American males. It occurs more commonly in the right testicle than the left. Testicular tumors are also more common in men who have had undescended testes (cryptorchidism) or have a family history of testicular cancer or anomalies. Epispadias and lack of circumcision are not related to testicular cancer. Hernia repair is related to future hernias, but not testicular cancer.

What is the most common cause of male infertility? A. A hydrocele B. A varicocele C. Alcohol use D. Epididymitis.

Answer: B The most common cause of male infertility is a varicocele, not a hydrocele. Other factors that influence the testes include infection such as epididymitis and alcohol use, but they are not the most common causes.

You have provided teaching to the male patient regarding a vasectomy procedure. Which statement by the patient indicates the teaching has been successful? A. "I will need to not eat or drink anything for 24 hours before the procedure." B. "The physician will use local anesthetic, and I will be awake." C. "I will be able to have unprotected intercourse in 3 weeks." D. "I acknowledge there is a risk of impotence."

Answer: B The procedure requires only 15 to 30 minutes and is usually performed with the patient under local anesthesia on an outpatient basis. Vasectomy is considered a permanent form of sterilization but does not affect the ability to achieve an erection. After vasectomy, the patient should not notice any difference in the look or feel of the ejaculate because its major component is seminal and prostatic fluid. The patient should use an alternative form of contraception until semen examination reveals no sperm. This usually requires at least 10 ejaculations or 6 weeks to evacuate sperm distal to the surgical site.

A 73-year-old man admitted for total knee replacement states during the health history interview that he has no problems with urinary elimination except that the "stream is less than it used to be." You would give the patient anticipatory guidance that which condition is likely to be developing? A. A tumor of the prostate B. Benign prostatic hyperplasia C. Bladder atony because of age D. Age-related altered innervation of the bladder

Answer: B BPH is an enlarged prostate gland caused by an increased number of epithelial cells and stromal tissue. It occurs in about 50% of men older than 50 years and 80% of men older than 80 years.

You should encourage a patient with bacterial prostatitis to A. remain on bed rest until antibiotic therapy has been in place for 72 hours. B. significantly increase fluid intake of noncaffeinated beverages. C. empty the bladder every hour with a straight catheter. D. weigh daily before the first voiding of the morning.

Answer: B Because the prostate can serve as a source of bacteria, fluid intake should be kept at a high level for all patients experiencing prostatitis. Nursing interventions are aimed at encouraging the patient to drink plenty of fluids. This is especially important for those with acute bacterial prostatitis because of the increased fluid needs associated with fever and infection. Bed rest is not indicated, and activity should be encouraged. The bladder should be emptied, although not with a catheter, and it is not necessary to do so every hour. Weight gain should be reported, but the patient does not need to weigh before voiding.

You are teaching the patient with BPH about interventions that can assist in alleviating symptoms. Which behavior in the patient indicates successful teaching? A. The patient increases use of decongestants. B. The patient decreases intake of caffeinated beverages and artificial sweeteners. C. The patient increases activities such as walking. D. The patient voids every 30 minutes.

Answer: B In some patients who have symptoms that appear and then disappear, a conservative treatment approach has value. Dietary changes (decreasing intake of caffeine and artificial sweeteners, limiting spicy or acidic foods), avoiding medications such as decongestants and anticholinergics, and restricting evening fluid intake may result in improvement of symptoms. A timed voiding schedule may reduce or eliminate symptoms, negating the need for further intervention, but 30 minutes is too frequent.

A nurse will teach a patient who is scheduled to complete a 24-hour urine collection for 17-ketosteroids to a. insert and maintain a retention catheter. b. keep the specimen refrigerated or on ice. c. drink at least 3 L of fluid during the 24 hours. d. void and save that specimen to start the collection.

B

When assessing a patient for breast cancer risk, the nurse considers that the patient has a significant family history of breast cancer if she has a a. cousin who was diagnosed with breast cancer at age 38. b. mother who was diagnosed with breast cancer at age 42. c. sister who died from ovarian cancer at age 56. d. grandmother who died from breast cancer at age 72.

Answer: B Rationale: A significant family history of breast cancer means that the patient has a first-degree relative who developed breast cancer, especially if the relative was premenopausal. Cognitive Level: Application Text Reference: p. 1348 Nursing Process: Assessment NCLEX: Health Promotion and Maintenance

A patient with a small breast lump is advised to have a fine needle aspiration (FNA) biopsy. The nurse explains that an advantage to this procedure is that a. only a small incision is necessary, resulting in minimal breast pain and scarring. b. if the specimen is positive for malignancy, the patient can be told at the visit. c. if the specimen is negative for malignancy, the patient's fears of cancer can be put to rest. d. FNA is guided by a mammogram, ensuring that cells are taken from the lesion.

Answer: B Rationale: An FNA should only be done when an experienced cytologist is available to read the specimen immediately. If the specimen is positive for malignancy, the patient can be given this information immediately. No incision is needed. If the specimen is negative for malignancy, the patient will require biopsy of the lump. FNA is not guided by mammography. Cognitive Level: Application Text Reference: p. 1345 Nursing Process: Planning NCLEX: Physiological Integrity

A 51-year-old woman at menopause is considering the use of hormone replacement therapy (HRT) but is concerned about the risk of breast cancer. When discussing this issue with the patient, the nurse explains that a. HRT does not appear to increase the risk for breast cancer unless there are other risk factors. b. she and her health care provider must weigh the benefits of HRT against the possible risks of breast cancer. c. HRT is a safe therapy for menopausal symptoms if there is no family history of BRCA genes. d. alternative therapies with herbs and natural drugs are as effective as estrogen in relieving the symptoms of menopause.

Answer: B Rationale: Because HRT has been linked to increased risk for breast cancer, the patient and provider must determine whether or not to use HRT. Breast cancer incidence is increased in women using HRT, independent of other risk factors. HRT increase the risk for non-BRCA-associated cancer as well as for BRCA-related cancers. Alternative therapies can be used but are not consistent in relieving menopausal symptoms. Cognitive Level: Application Text Reference: p. 1348 Nursing Process: Implementation NCLEX: Physiological Integrity

At a routine health examination, a woman whose mother had breast cancer asks the nurse about the genetic basis of breast cancer and the genes involved. The nurse explains that a. her risk of inheriting BRCA gene mutations is small unless her mother had both ovarian and breast cancer. b. changes in BRCA genes that normally suppress cancer growth can be passed to offspring, increasing the risk for breast cancer. c. because her mother had breast cancer, she has inherited a 50% to 85% chance of developing breast cancer from mutated genes. d. genetic mutations increase cancer risk only in combination with other risk factors such as obesity.

Answer: B Rationale: Family history is a risk factor for breast cancer, and the nurse should discuss testing for BRCA genes with the patient. Although the BRCA gene is associated with increased risk for breast and ovarian cancer, the patient may be at risk if her mother had either one. About 5% to 10% of patients with breast cancer may have a genetic abnormality that contributes to breast cancer development. Risk factors are cumulative, but a family history alone will increase breast cancer risk. Cognitive Level: Application Text Reference: p. 1349 Nursing Process: Implementation NCLEX: Physiological Integrity

A 20-year-old student comes to the student health center after discovering a small painless lump in her right breast. She is worried that she might have cancer because her mother had cervical cancer. The nurse's response to the patient is based on the knowledge that the most likely cause of the breast lump is a. fibrocystic complex. b. fibroadenoma. c. breast abscess. d. adenocarcinoma.

Answer: B Rationale: Fibroadenoma is the most frequent cause of breast lumps in women under 25 years of age. Fibrocystic changes occur most frequently in women ages 35 to 50. Breast abscess is associated with pain and other systemic symptoms. Breast cancer is uncommon in women younger than 25. Cognitive Level: Application Text Reference: p. 1347 Nursing Process: Implementation NCLEX: Physiological Integrity

When teaching a 22-year-old patient about breast self-examination (BSE), the nurse will instruct the patient that a. BSE will reduce the risk of dying from breast cancer. b. performing BSE right after the menstrual period will improve comfort. c. BSE should be done daily while taking a bath or shower. d. annual mammograms should be scheduled in addition to BSE.

Answer: B Rationale: Performing BSE at the end of the menstrual period will reduce the breast tenderness associated with the procedure. The evidence is not clear that BSE reduces breast cancer mortality. BSE should be done monthly. Annual mammograms are not routinely scheduled for women under age 40. Cognitive Level: Application Text Reference: p. 1344 Nursing Process: Implementation NCLEX: Health Promotion and Maintenance

A patient with a breast biopsy positive for cancer is to undergo lymphatic mapping and sentinel lymph node dissection (SLND). The nurse explains that this procedure a. can identify specific lymph nodes that have malignant cells, so only involved nodes need to be excised. b. reduces the need for extensive lymph node dissection for pathologic examination. c. eliminates the need for excision of more than one lymph node for staging of breast cancer. d. will confirm the absence of tumor spread if the sentinel lymph node is negative for malignant changes.

Answer: B Rationale: The SLND may eliminate further lymph node dissection if the initial nodes are negative for malignancy. The procedure identifies which lymph nodes drain first from the tumor site, but not which ones are malignant. Several lymph nodes may be dissected for pathologic examination. Tumor may have distant metastases even when no malignancies are found in the lymph nodes. Cognitive Level: Comprehension Text Reference: p. 1351 Nursing Process: Implementation NCLEX: Physiological Integrity

During the physical examination of a 36-year-old female, the nurse finds that the patient's thyroid gland cannot be palpated. The most appropriate action by the nurse is to a. palpate the patient's neck more deeply. b. document that the thyroid was nonpalpable. c. notify the health care provider immediately. d. teach the patient about thyroid hormone testing.

B

A woman with a positive biopsy for breast cancer is considering whether to have a modified radical mastectomy or breast conservation surgery (lumpectomy) with radiation therapy. Which information should the nurse provide? a. The postoperative survival rate for each is about the same, but there is a decreased rate of cancer recurrence after mastectomy. b. The lumpectomy and radiation will preserve the breast, but this method can cause changes in breast sensitivity. c. The hair loss associated with post-lumpectomy chemotherapy is not acceptable to some patients. d. The treatment period for the mastectomy is shorter, and breast reconstruction can provide a normal-appearing breast.

Answer: B Rationale: The impact on breast function and appearance is less with lumpectomy and radiation, but there is some effect on breast sensitivity. The rate of cancer recurrence is the same for the two procedures. Chemotherapy may be used after either lumpectomy or mastectomy, but it is not always needed. The treatment period is shorter after mastectomy, but breast reconstruction does not provide a normal-appearing breast. Cognitive Level: Application Text Reference: pp. 1352-1353 Nursing Process: Implementation NCLEX: Physiological Integrity

A patient has a permanent breast implant inserted in the outpatient surgery area. Which instructions will the nurse include in the discharge teaching? a. Resume normal activities 2 to 3 days after the mammoplasty. b. Check wound drains for excessive blood or any foul odor. c. Wear a loose-fitting bra to decrease irritation of the sutures. d. Take aspirin every 4 hours to reduce inflammation.

Answer: B Rationale: The patient should be taught drain care because the drains will be in place for 2 or 3 days after surgery. Normal activities can be resumed after 2 to 3 weeks. A bra that provides good support is typically ordered. Aspirin will decrease coagulation and is typically not given after surgery. Cognitive Level: Comprehension Text Reference: p. 1362 Nursing Process: Implementation NCLEX: Physiological Integrity

Which best indicates that treatment for cancer of the prostate is effective? A. Increase in urinary stream B. Decrease of PSA to 2 ng/mL C. Decreased blood in the urine D. White blood cell (WBC) count of 10,000/μL

Answer: B The PSA value is used to detect prostate cancer and to monitor the success of treatment. When treatment has been successful in removing prostate cancer, PSA levels should decrease and reach normal levels (less than 4 ng/mL). The regular measurement of PSA levels after treatment is important to evaluate the effectiveness of treatment and possible recurrence of prostate cancer.

What is the most significant factor in the development of clinical symptoms associated with BPH? A. Size of the prostate B. Location of the enlargement C. Age of the patient D. Length of the urethra

Answer: B There is no direct relationship between the size of the prostate and degree of obstruction. The location of the enlargement significantly affects development of obstructive symptoms. For example, it is possible for mild hyperplasia to cause severe obstruction, and it is possible for extreme hyperplasia to cause few obstructive symptoms.

A complication of prostatitis is A. prostatic cancer. B. benign prostatic hyperplasia. C. acute urinary retention. D. hydrocele.

Answer: C Acute urinary retention can develop in acute prostatitis, and it requires bladder drainage with suprapubic catheterization. Passage of a catheter through an inflamed urethra is contraindicated in acute prostatitis.

When providing patient teaching about medication therapy for BPH with 5α-reductase inhibitors such as finasteride (Proscar), what information should you include? A. Ninety percent of patients show improvement with the drug. B. The drug can be taken periodically as symptoms occur. C. Women who are pregnant should not handle the drug. D. Effects are seen in 1 week.

Answer: C Although more than 50% of men who are treated with the drug show symptom improvement, it takes about 6 months to be effective. The drug must be taken on a continuous basis to maintain therapeutic results. Women who may be or are pregnant should not handle tablets.

To decrease the patient's discomfort about care involving his reproductive organs, you should A. relate his sexual concerns to his sexual partner. B. arrange to have male nurses care for the patient. C. maintain a nonjudgmental attitude toward his sexual practices. D. use technical terminology when discussing reproductive function.

Answer: C Conducting routine health assessments on men places you in a unique position. It provides an opportunity to ask the patient questions pertaining to general health and to sexual health and function. Given the opportunity, men are less hesitant to answer these questions when they know that someone cares and can provide them with answers. You must remain nonjudgmental about sexual practices.

The patient with epididymitis asks you when he can resume his exercise routine. You should respond with which piece of information? A. When there is no evidence of urethral discharge B. After 1 week of antibiotic therapy C. When scrotal pain has subsided D. As soon as he feels able

Answer: C Most tenderness subsides within 1 week, although swelling may last for weeks or months. One week of antibiotic therapy may not relieve pain and swelling. "When the patient feels able" is a vague response, and the patient should remain on rest until scrotal swelling has subsided. There may be no urethral discharge in epididymitis.

What is the first step in the identification of the cause of male infertility? A. Estrogen levels B. Progesterone levels C. Semen analysis D. Time to ejaculation

Answer: C The first test in an infertility study is a semen analysis. The test determines the sperm concentration, motility, and morphology

What is the most common cause of erectile dysfunction? A. Aging B. Benign prostatic hyperplasia C. Vascular disease D. Psychological distress

Answer: C Common causes of ED include diabetes, vascular disease, side effects from medications, results of surgery (e.g., prostatectomy), trauma, chronic illness, decreased gonadal hormone secretion, stress, difficulty in a relationship, or depression. The most common cause of ED is vascular disease.

A 62-year-old man is seen at the health clinic because he is concerned about a gradual decrease in sexual performance. The nursing history does not identify any specific risk factors related to erectile dysfunction. What is your priority at this time? A. Consult with the health care provider about testing the patient for hypogonadism. B. Refer the patient to a qualified therapist to explore possible psychologic causes of decreased function. C. Explain normal age-related changes in sexual performance to the patient. D. Discuss the variety of aids and devices available to increase sexual performance.

Answer: C Normal physiologic age-related changes are associated with changes in erectile function and may be an underlying cause of ED for some men.

A 62-year-old patient complains to the nurse that mammograms are painful and a source of radiation exposure. She says she does breast self-examination (BSE) monthly and asks whether it is necessary to have an annual mammogram. The nurse's best response to the patient is, a. "If your mammogram was painful, it is especially important that you have it done annually." b. "An ultrasound examination of the breasts, which is not painful or a source of radiation, can be substituted for a mammogram." c. "Because of your age, it is even more important for you to have annual mammograms." d. "Unless you find a lump while examining your breasts, a mammogram every 2 years is recommended after age 60."

Answer: C Rationale: Annual mammograms are recommended for women over age 40 as long as they are in good health. The incidence of breast cancer increases in women over 60. Pain with a mammogram does not indicate any greater risk for breast cancer. Ultrasound may be used in some situations to differentiate cystic breast problems from cancer but is not a substitute for annual mammograms. Cognitive Level: Application Text Reference: p. 1344 Nursing Process: Implementation NCLEX: Health Promotion and Maintenance

A 33-year-old patient tells the nurse that she has fibrocystic breasts but reducing her sodium and caffeine intake and other measures have not made a difference in the fibrocystic condition. An appropriate patient outcome for the patient is a. calls the health care provider if any lumps are painful or tender. b. states the reason for immediate biopsy of new lumps. c. monitors changes in size and tenderness of all lumps in relation to her menstrual cycle. d. has genetic testing for BRCA-1 and BRCA-2 to determine her risk for breast cancer.

Answer: C Rationale: Because fibrocystic breasts may increase in size and tenderness during the premenstrual phase, the patient is taught to monitor for this change and to call if the changes persist after menstruation. Pain and tenderness are typical of fibrocystic breasts, and the patient should not call for these symptoms. New lumps may be need biopsy if they persist after the menstrual period, but the biopsy is not done immediately. The existence of fibrocystic breasts is not associated with the BRCA genes. Cognitive Level: Application Text Reference: pp. 1346-1347 Nursing Process: Planning NCLEX: Health Promotion and Maintenance

A patient at the clinic who has metastatic breast cancer has a new prescription for trastuzumab (Herceptin). The nurse will plan to a. teach the patient about the need to monitor serum electrolyte levels. b. ask the patient to call the health care provider before using any over-the-counter (OTC) pain relievers. c. instruct the patient to call if she notices ankle swelling. d. have the patient schedule frequent eye examinations.

Answer: C Rationale: Herceptin can lead to ventricular dysfunction, so the patient is taught to self-monitor for symptoms of heart failure. There is no need to monitor serum electrolyte levels. OTC pain relievers do not interact with Herceptin. Changes in visual acuity may occur with tamoxifen, but not with Herceptin. Cognitive Level: Analysis Text Reference: p. 1356 Nursing Process: Implementation NCLEX: Physiological Integrity

Following a modified radical mastectomy, a patient tells the nurse the health care provider has recommended a flap procedure for breast reconstruction but that she did not understand how this was done. The nurse explains that the most common procedure, a transverse rectus abdominis musculocutaneous (TRAM) flap, involves a. relocating muscle tissue from the back and using it to form a breast. b. removing a portion of an abdominal muscle to use as breast tissue. c. pulling part of the abdominal muscle up to the breast area through a tunnel in the chest. d. relocating the arteries from the abdominal muscle to improve circulation to the implant.

Answer: C Rationale: In the TRAM flap, part of the rectus abdominis muscle is tunneled to the breast area and molded to form a breast. In the latissimus dorsi musculocutaneous flap, muscle tissue from the back is used to replace breast tissue. The abdominal muscle is not detached but is still attached to the rectus muscle. The arteries are not relocated. Cognitive Level: Application Text Reference: pp. 1362-1363 Nursing Process: Implementation NCLEX: Physiological Integrity

Which statement by a 32-year-old patient newly diagnosed with stage I breast cancer indicates to the nurse that the goals of therapy are being met? a. "I am not sure how my husband will react when I tell him about this cancer." b. "I am ready to die if that is God's plan for me." c. "I need to know all the options before making a decision about treatment." d. "I will do whatever the doctor thinks is best."

Answer: C Rationale: One goal for the patient with breast cancer is active participation in the decision-making process. The response beginning, "I am not sure how my husband will react" indicates that the goal of satisfaction with the support provided by significant others is still unmet. The response, "I am ready to die if that is God's plan for me" suggests that the patient may not be willing to have treatment. The response, "I will do whatever the doctor thinks is best" indicates that the patient is not participating actively in treatment decisions. Cognitive Level: Application Text Reference: p. 1359 Nursing Process: Evaluation NCLEX: Psychosocial Integrity

A patient returns to the surgical unit following a right modified radical mastectomy with dissection of axillary lymph nodes. An appropriate intervention for the nurse to include in implementing postoperative care for the patient includes a. teaching the patient to use the ordered patient-controlled analgesia (PCA) every 10 minutes for the best pain relief. b. insisting that the patient examine the surgical incision when the dressings are removed. c. posting a sign at the bedside warning against blood pressures or venipunctures in the right arm. d. encouraging the patient to obtain a permanent breast prosthesis as soon as she is discharged from the hospital.

Answer: C Rationale: The patient is at risk for lymphedema and infection if blood pressures or venipuncture are done on the right arm. The patient is taught to use the PCA as needed for pain control rather than at a set time. The nurse allows the patient to examine the incision and participate in care when the patient feels ready. Permanent breast prostheses are usually obtained about 6 weeks after surgery. Cognitive Level: Application Text Reference: pp. 1358-1359 Nursing Process: Implementation NCLEX: Physiological Integrity

A 34-year-old woman has undergone a modified radical mastectomy for a breast tumor. The pathology report identified the tumor as a stage I, estrogen-receptor-positive adenocarcinoma. The nurse will plan on teaching the patient about a. raloxifene (Evista). b. estradiol (Estrace). c. trastuzumab (Herceptin). d. tamoxifen (Nolvadex).

Answer: D Rationale: Tamoxifen is used for estrogen-dependent breast tumors in premenopausal women. Raloxifene is used to prevent breast cancer, but it is not used post-mastectomy to treat breast cancer. Estradiol will increase the growth of estrogen-dependent tumors. Trastuzumab is used to treat tumors that have the HER-2/neu antigen. Cognitive Level: Application Text Reference: p. 1355 Nursing Process: Planning NCLEX: Physiological Integrity

Following a modified radical mastectomy, the health care provider recommends chemotherapy even though the lymph nodes were negative for cancer cells. The patient tells the nurse that she does not know what to do about chemotherapy because she has heard that she may not even need chemotherapy and that the side effects are uncomfortable. The nursing diagnosis that best reflects the patient's problem is a. anxiety related to prospect of additional cancer therapy. b. fear related to uncomfortable side effects of chemotherapy. c. decisional conflict related to lack of knowledge about prognosis and treatment options. d. risk for ineffective health maintenance related to reluctance to consider additional treatment.

Answer: C Rationale: The patient's statements indicate that she is having difficulty making a decision about treatment because of a lack of understanding about prognosis and treatment. Although she may have some anxiety and fear, these are not the priorities at this time. The patient expresses concerns about chemotherapy rather than reluctance to consider additional treatment. Cognitive Level: Application Text Reference: p. 1356 Nursing Process: Diagnosis NCLEX: Psychosocial Integrity

During the first 4 hours after TURP, the patient receives 1200 mL of bladder irrigation solution, and his urine output is 1000 mL. What is your priority intervention? A. Slowing the rate of bladder irrigation B. Continuing to observe the patient C. Checking catheter patency D. Encouraging oral fluids

Answer: C You should continuously monitor the inflow and outflow of the irrigant. If outflow is less than inflow, assess the catheter for kinks or clots. If the outflow is blocked and patency cannot be reestablished by manual irrigation, stop the CBI and notify the physician.

When working in the emergency department you recognize the need for immediate diagnosis and treatment of the patient with A. orchitis. B. phimosis. C. epididymitis. D. testicular torsion.

Answer: D Torsion constitutes a surgical emergency. If the blood supply to the affected testicle is not restored within 4 to 6 hours, testicular ischemia will occur, leading to necrosis and possible removal.

What should you explain to the patient who has had a vasectomy? A. The procedure blocks the production of sperm B. Erectile dysfunction is temporary and will return with sexual activity. C. The ejaculate will be about half the volume it was before the procedure. D. An alternative form of contraception will be necessary for 6 to 8 weeks.

Answer: D After a vasectomy, the patient should use an alternative form of contraception until semen examination reveals no sperm. This usually requires at least 10 ejaculations or 6 weeks to evacuate sperm distal to the surgical site.

In assessing a patient for testicular cancer, you understand that the manifestations of this disease often include A. acute back spasms and testicular pain. B. rapid onset of scrotal swelling and fever. C. fertility problems and bilateral scrotal tenderness. D. painless mass and heaviness sensation in the scrotal area.

Answer: D Clinical manifestations of testicular cancer include a painless lump in the scrotum, scrotal swelling, and a feeling of heaviness. The scrotal mass usually is not tender and is very firm. Some patients complain of a dull ache or heavy sensation in the lower abdomen, perianal area, or scrotum.

A priority nursing diagnosis for the patient with bacterial prostatitis is A. activity intolerance related to fatigue. B. sexual dysfunction related to painful ejaculation. C. deficient fluid volume related to decreased fluid intake. D. impaired urinary elimination related to urethral compression.

Answer: D Acute urinary retention can develop in acute prostatitis, and it requires bladder drainage with suprapubic catheterization. The patient may experience fatigue, but this is not the priority. Sexual dysfunction can occur, but the pain occurs after ejaculation. Fluid volume should be increased.

Postoperatively, a patient who has had a laser prostatectomy has continuous bladder irrigation with a three-way urinary catheter with a 30-mL balloon. When he complains of bladder spasms with the catheter in place, you should A. deflate the catheter balloon to 10 mL to decrease bulk in the bladder. B. deflate the catheter balloon and then reinflate it to ensure that it is patent. C. encourage the patient to try to have a bowel movement to relieve colon pressure. D. explain that this feeling is normal and that he should not try to urinate around the catheter.

Answer: D Bladder spasms occur as a result of irritation of the bladder mucosa from the insertion of the resectoscope, presence of a catheter, or clots leading to obstruction of the catheter. Instruct the patient not to urinate around the catheter because this increases the likelihood of spasm.

You recognize the need to begin testosterone therapy when testosterone levels drop below A. 750 ng/dL. B. 500 ng/dL. C. 400 ng/dL. D. 250 ng/dL.

Answer: D Normal testosterone levels can range from 280 to 1100 ng/dL. Replacement therapy may be considered when levels are below 250 ng/dL.

A patient with an abnormal mammogram is scheduled for stereotactic core biopsy. Which information will the nurse include when teaching the patient about the procedure? a. "You will need to avoid eating or drinking anything for 6 hours before the procedure." b. "Any discomfort after the biopsy may be treated with mild pain relievers such as aspirin." c. "The core biopsy is evaluated immediately and you will get the results before leaving." d. "Several samples of tissue in the abnormal area will be obtained during the procedure."

Answer: D Rationale: During stereotactic breast biopsy, a biopsy gun is used to remove several core samples in the area of abnormality. The procedure is done using a local anesthetic, so there is no need to be NPO before the procedure. Aspirin should not be used because it will increase bleeding at the site. The biopsy is sent to pathology, and results are not usually available immediately. Cognitive Level: Application Text Reference: p. 1345 Nursing Process: Implementation NCLEX: Health Promotion and Maintenance

While the nurse is obtaining a nursing history from a 52-year-old patient who has found a small lump in her breast, which question is most pertinent? a. "Do you currently smoke cigarettes?" b. "Have you ever had any breast injuries?" c. "Is there any family history of fibrocystic breast changes?" d. "At what age did you start having menstrual periods?"

Answer: D Rationale: Early menarche and late menopause are risk factors for breast cancer because of the prolonged exposure to estrogen that occurs. Cigarette smoking, breast trauma, and fibrocystic breast changes are not associated with increased breast cancer risk. Cognitive Level: Application Text Reference: p. 1349 Nursing Process: Assessment NCLEX: Physiological Integrity

The nurse provides discharge teaching for a patient who has had a left modified radical mastectomy and axillary lymph node dissection. The nurse determines that teaching has been successful when the patient says, a. "I should keep my left arm supported in a sling when I am up until my incision is healed." b. "I may expose my left arm to the sun for several hours each day to increase circulation and promote healing." c. "I can do whatever exercises and activities I want as long as I do not elevate my left hand above my head." d. "I will continue to exercise my left arm with finger-walking up the wall or combing my hair."

Answer: D Rationale: The patient should continue with arm exercises to regain strength and range of motion. The left arm should be elevated to the level of the heart when the patient is up. Sun exposure is avoided because of the risk of sunburn. The left hand should be elevated at or above heart level to reduce swelling and lymphedema. Cognitive Level: Application Text Reference: pp. 1359-1360 Nursing Process: Evaluation NCLEX: Physiological Integrity

A 45-year-old man asks you if it is advisable to have his prostate-specific antigen (PSA) level tested, because his father and brother have prostate cancer. What is your response? A. "You should wait until you are age 50." B. "You should have a transurethral resection of the prostate as a preventive measure." C. "You should have a voiding cystourethrogram yearly." D. "You should have annual PSA levels assessed and a digital examination of the prostate."

Answer: D The American Cancer Society recommends an annual digital rectal examination (DRE) and a blood test for PSA beginning at age 50 for men who are at average risk for prostate cancer. During DRE, an abnormal prostate may feel hard, nodular, and asymmetric.

When providing teaching regarding potential complications after perineal resection of the prostate, what should you include? A. Deep vein thrombosis B. Pulmonary embolism C. Colonic constipation D. Urinary incontinence

Answer: D The two major complications after a radical prostatectomy are erectile dysfunction and urinary incontinence.

Which action by a new registered nurse (RN) caring for a patient with a goiter and possible hyperthyroidism indicates that the charge nurse needs to do more teaching? a. The RN checks the blood pressure on both arms. b. The RN palpates the neck thoroughly to check thyroid size. c. The RN lowers the thermostat to decrease the temperature in the room. d. The RN orders nonmedicated eye drops to lubricate the patient's bulging eyes.

B

Which information about a 30-year-old patient who is scheduled for an oral glucose tolerance test should be reported to the health care provider before starting the test? a. The patient reports having occasional orthostatic dizziness. b. The patient takes oral corticosteroids for rheumatoid arthritis. c. The patient has had a 10-pound weight gain in the last month. d. The patient drank several glasses of water an hour previously.

B

A 22 year old female patient has come to the clinic for oral contraceptives. The nurse uses the opportunity for patient teaching about BSE. What shoudl the nurse include in patient teaching? A. Women at high risk for breast cancer begin BSE at 3o years old B. Perform BSE monthly on the day of the first pill in each package C. Older women use BSE annually as breast tissue density declines D. Healthy women need to have annual mammograms beginning at age 50

B. The nurse instructs the pt to use the packageing for the oral contraceptives as a convienient reminder to perform BSE monthly to increase adherence

Which of the following female patients should report her results from breast self-examination? A. Denser breast tissue B. Left nipple deviation C. Palpable rib margins D. Different size breasts

B. Unilateral deviation of a nipple and nipple retraction are clinical indicators of breast cancer and should be reported promplty to the HCP. The breasts of most women are slightly different sizes

A male patient complains of fever, dysuria, and cloudy urine. What additional information may indicate that these manifestations may be something other than a urinary tract infection (UTI)? A. E. coli bacteria in his urine B. A very tender prostate gland C. Complaints of chills and rectal pain D. Complaints of urgency and frequency

B. A very tender prostate gland A tender and swollen prostate is indicative of prostatis, which is a more serious male reproductive problem because an acute episode can result in chronic prostatis and lead to epididymitis or cystitis. E. coli in his urine, chills and rectal pain, and urgency and frequency are all present with a UTI and not specifically indicative of prostatis.

The nurse has been asked to participate in a healthy living workshop. While teaching about women's health, which of the following guidelines should the nurse provide to the audience? A. "Mammograms are necessary if you have a family history of breast cancer." B. "It's recommended that you get a mammogram each year after you turn 40." C. "If you are not able to perform breast self-examination (BSE), you should go for regular mammograms." D. "You should ensure that your primary care provider performs a breast exam each time you visit."

B. Annual mammograms are recommended after age 40. They are recommended for all women, not solely those with a family history of breast cancer. BSE is not a replacement for mammography and clinical breast examinations are not necessary at each office visit.

A 24-year-old patient who has undergone breast augmentation earlier in the day will be discharged home in the early evening. What instructions should the nurse provide in order to minimize the patient's risk of complications in the immediate recovery period? A. Avoid wearing a bra until postoperative day 3. B. Ask the patient to avoid strenuous exercise during her recovery period. C. Sleep in a semi-Fowler's position until her scheduled follow-up appointment. D. Enlist a friend or family member to perform passive range-of-motion exercises.

B. As with all types of breast surgery, strenuous exercise is contraindicated during the recovery period following breast augmentation. A bra should be worn to prevent dehiscence. Passive range-of-motion exercises should be avoided, and sleeping in a semi-Fowler's position is not necessary

A 73-year-old male patient admitted for total knee replacement states during the health history interview that he has no problems with urinary elimination except that the "stream is less than it used to be." The nurse should give the patient anticipatory guidance that what condition may be developing? A. A tumor of the prostate B. Benign prostatic hyperplasia C. Bladder atony because of age D. Age-related altered innervation of the bladder

B. Benign prostatic hyperplasia Benign prostatic hyperplasia is an enlarged prostate gland because of an increased number of epithelial cells and stromal tissue. It occurs in about 50% of men over age 50 and 80% of men over age 80. Only about 16% of men develop prostate cancer. Bladder atony and age-related altered innervations of the bladder do not lead to a weakened stream.

A 45-year-old man reports having recent problems attaining an erection. Which medication will the nurse further explore as the possible etiology of this patient's sexual dysfunction? A. Furosemide (Lasix) B. Fluoxetine (Prozac) C. Clopidogrel (Plavix) D. Nitroglycerin (Nitrostat)

B. Fluoxetine (Prozac) Fluoxetine is a selective serotonin reuptake inhibitor used in the treatment of depression. A common adverse effect of this medication is sexual problems (impotence, delayed or absent orgasm, delayed or absent ejaculation, decreased sexual interest) in nearly 70% of men and women.

Which of the following tasks should the registered nurse delegate to nursing assistive personnel (NAP) during the care of a patient who has had recent transverse rectus abdominis musculocutaneous (TRAM) flap surgery? A. Document the condition of the patient's incisions. B. Mobilize the patient in a slightly hunched position. C. Change the patient's abdominal and chest dressings. D. Change the parameters of the patient-controlled analgesic (PCA) pump.

B. Mobilization of a postsurgical patient may be delegated, and the patient who has had a TRAM flap should not stand or walk fully erect, in order to minimize strain on the incisions. Changing dressings, assessing wounds, and reprogramming a PCA pump are not tasks amenable to delegation.

A 50-year-old patient is preparing to begin breast cancer treatment with tamoxifen (Nolvadex). Which of the following points should the nurse emphasize when teaching the patient about her new drug regimen? A. "You may find that your medication causes some breast sensitivity." B. "It's important that you let your care provider know about any changes in vision." C. "You'll find that this drug often alleviates some of the symptoms that accompany menopause." D. "It's imperative that you abstain from drinking alcohol after you begin taking tamoxifen."

B. Tamoxifen has the potential to cause cataracts and retinopathy. The drug is likely to exacerbate rather than alleviate perimenopausal symptoms. Breast tenderness is not associated with tamoxifen, and it is not necessary for the patient to abstain from alcohol.

A 32-year-old patient has oral contraceptives prescribed for endometriosis. The nurse will teach the patient to a. expect to experience side effects such as facial hair. b. take the medication every day for the next 9 months. c. take calcium supplements to prevent developing osteoporosis during therapy. d. use a second method of contraception to ensure that she will not become pregnant.

B. When oral contraceptives are prescribed to treat endometriosis, the patient should take the medications continuously for 9 months. Facial hair is a side effect of synthetic androgens. The patient does not need to use additional contraceptive methods. The hormones in oral contraceptives will protect against osteoporosis

A 32 year old pt has stage IIB breast cancer that is estrogen receptor positive. What medication should the nurse avoid administering to the patient? A. diethylstilbestrol (DES) B. exemestane (Aromasin) C. trastuzumab (Herceptin) D. doxorubicin (Adriamycin)

B. exemestane (Aromasin) Is an aromatase inhibitor is used in the treatment of postmenopausal women. This drug interferes with the enzyme that helps synthesize endogenous estrogen but does not block the production of ovarian estrogen. Aromatase inhibitors are of little benefir in premenopausal women with breast cancer.

A 29-year-old patient in the outpatient clinic will be scheduled for blood cortisol testing. Which instruction will the nurse provide? a. "Avoid adding any salt to your foods for 24 hours before the test." b. "You will need to lie down for 30 minutes before the blood is drawn." c. "Come to the laboratory to have the blood drawn early in the morning." d. "Do not have anything to eat or drink before the blood test is obtained."

C

A 44-year-old patient is admitted with tetany. Which laboratory value should the nurse monitor? a. Total protein b. Blood glucose c. Ionized calcium d. Serum phosphate

C

A 60-year-old patient is taking spironolactone (Aldactone), a drug that blocks the action of aldosterone on the kidney, for hypertension. The nurse will monitor for a. increased serum sodium. b. decreased urinary output. c. elevated serum potassium. d. evidence of fluid overload.

C

The nurse reviews a patient's glycosylated hemoglobin (Hb A1C) results to evaluate a. fasting preprandial glucose levels. b. glucose levels 2 hours after a meal. c. glucose control over the past 90 days. d. hypoglycemic episodes in the past 3 months.

C

The nurse will teach a patient to plan to minimize physical and emotional stress while the patient is undergoing a. a water deprivation test. b. testing for serum T3 and T4 levels. c. a 24-hour urine test for free cortisol. d. a radioactive iodine (I-131) uptake test.

C

Which additional information will the nurse need to consider when reviewing the laboratory results for a patient's total calcium level? a. The blood glucose is elevated. b. The phosphate level is normal. c. The serum albumin level is low. d. The magnesium level is normal.

C

Which laboratory value should the nurse review to determine whether a patient's hypothyroidism is caused by a problem with the anterior pituitary gland or with the thyroid gland? a. Thyroxine (T4) level b. Triiodothyronine (T3) level c. Thyroid-stimulating hormone (TSH) level d. Thyrotropin-releasing hormone (TRH) level

C

Which question will provide the most useful information to a nurse who is interviewing a patient about a possible thyroid disorder? a. "What methods do you use to help cope with stress?" b. "Have you experienced any blurring or double vision?" c. "Have you had a recent unplanned weight gain or loss?" d. "Do you have to get up at night to empty your bladder?"

C

Which benign breast disorder is the most similar to breast cancer? A. Ductal ectasia B. Fibroadenoma C. Breasat abscess D. Cyclic mastalgia

C. A breast abscess is a benign breast disorder most likely to be confused with a malignany breast lesion because it shares moe characteristics of breast cancers than other benign breast disorders. A breast abscess can be red and edematous in apperance characteristic of inflammatory breast cancer. The abscess also is likely to be a palpable mass, a potential characteristic of breast cancer.

A nurse is teaching a health promotion workshop to a group of women in their 40s and 50s. Which of the following points about nipple discharge should the nurse teach to participants? A. Inappropriate lactation necessitates breast biopsy. B. Nipple discharge of any type is considered a precursor to cancer. C. Unexpected nipple discharge of any type warrants medical follow-up. D. Galactorrhea is a normal age-related change and a frequent perimenopausal symptom.

C. Although most cases of nipple discharge are not related to malignancy, further assessment is indicated. Galactorrhea is not considered a normal age-related change, nor is it a common perimenopausal symptom.

The nurse teaches a 30-year-old man with a family history of prostate cancer about dietary factors associated with prostate cancer. The nurse determines that teaching is successful if the patient selects which menu? A. Grilled steak, French fries, and vanilla shake B. Hamburger with cheese, pudding, and coffee C. Baked chicken, peas, apple slices, and skim milk D. Grilled cheese sandwich, onion rings, and hot tea

C. Baked chicken, peas, apple slices, and skim milk A diet high in red meat and high-fat dairy products along with a low intake of vegetables and fruits may increase the risk of prostate cancer.

The nurse is caring for a patient diagnosed with breast cancer who just underwent an axillary lymph node dissection. Which of the following interventions would the nurse use to decrease the lymphedema? A. Keep affected arm flat at the patient's side. B. Apply an elastic bandage on the affected arm. C. Assess blood pressure on unaffected arm only. D. Restrict exercise of the affected arm for 1 week.

C. Blood pressure readings, venipunctures, and injections should not be done on the affected arm. Elastic bandages should not be used in the early postoperative period because they inhibit collateral lymph drainage. The affected arm should be elevated above the heart, and isometric exercises are recommended to reduce fluid volume in the arm.

To accurately monitor progression of a symptom of decreased urinary stream, the nurse should encourage the patient to have which primary screening measure done on a regular basis? A. Uroflowmetry B. Transrectal ultrasound C. Digital rectal examination (DRE) D. Prostate-specific antigen (PSA) monitoring

C. Digital rectal examination (DRE) Digital rectal examination is part of a regular physical examination and is a primary means of assessing symptoms of decreased urinary stream, which is often caused by benign prostatic hyperplasia in men over 50 years of age. The uroflowmetry helps determine the extent of urethral blockage and the type of treatment needed but is not done on a regular basis. Transrectal ultrasound is indicated with an abnormal DRE and elevated PSA to differentiate between BPH and prostate cancer. The PSA monitoring is done to rule out prostate cancer, although levels may be slightly elevated in patients with BPH.

A 29-year-old primiparous patient has a 3-week-old infant whom she is breastfeeding. The woman has sought care because of recent breast tenderness, redness, and fever. Which of the following teaching points should the nurse prioritize when following up her care? A. Encourage patient to continue breastfeeding her infant. B. Refer patient for a mammogram as quickly as possible. C. Ensure patient adheres to her prescribed antibiotic regimen. D. Teach patient to use warm compresses and educate her about self-limiting nature of illness.

C. Mastitis normally requires antibiotic therapy, the success of which is often dependent on close adherence to the prescribed regimen. Breastfeeding should indeed be continued if possible, but effective treatment of her infection would be the immediate priority. Mastitis is not necessarily self limiting and mammography is not normally indicated.

The patient has had cardiovascular disease for some time and has now developed erectile dysfunction. He is frustrated because he cannot take erectogenic medications because he takes nitrates for his cardiac disease. What should the nurse do first to help this patient? A. Give the patient choices for penile implant surgery. B. Recommend counseling for the patient and his partner. C. Obtain a thorough sexual, health, and psychosocial history. D. Assess levels of testosterone, prolactin, LH, and thyroid hormones.

C. Obtain a thorough sexual, health, and psychosocial history. The nurse's first action to help this patient is to obtain a thorough sexual, health, and psychosocial history. Alternative treatments for the cardiac disease would then be explored if that had not already been done. Further examination or diagnostic testing would be based on the history and physical assessment, including hormone levels, counseling, or penile implant options.

The patient has a low-grade carcinoma on the left lateral aspect of the prostate gland and has been on "watchful waiting" status for 5 years. Six months ago his last prostate-specific antigen (PSA) level was 5 ng/mL. Which manifestations now indicate that the prostate cancer may be growing and he needs a change in his care (select all that apply)? A. Casts in his urine B. Presence of α-fetoprotein C. Serum PSA level 10 ng/mL D. Onset of erectile dysfunction E. Nodularity of the prostate gland

C. Serum PSA level 10 ng/mL E. Nodularity of the prostate gland The manifestations of increased PSA level along with the new nodularity of the prostate gland potentially indicate that the tumor may be growing. Casts in the urine, presence of α-fetoprotein, and new onset of erectile dysfunction do not indicate prostate cancer growth.

A patient is one day postoperative following a transurethral resection of the prostate (TURP). Which event is not an expected normal finding in the care of this patient? A. The patient requires two tablets of Tylenol #3 during the night. B. The patient complains of fatigue and claims to have minimal appetite. C. The patient has continuous bladder irrigation (CBI) infusing, but output has decreased. D. The patient has expressed anxiety about his planned discharge home the following day.

C. The patient has continuous bladder irrigation (CBI) infusing, but output has decreased. A decrease or cessation of output in a patient with CBI requires immediate intervention. The nurse should temporarily stop the CBI and attempt to resume output by repositioning the patient or irrigating the catheter. Complaints of pain, fatigue, and low appetite at this early postoperative stage are not unexpected. Discharge planning should be addressed, but this should not precede management of the patient's CBI.

A 62-year-old patient with hyperthyroidism is to be treated with radioactive iodine (RAI). The nurse instructs the patient? a. about radioactive precautions to take with all body secretions. b. that symptoms of hyperthyroidism should be relieved in about a week.

C. that symptoms of hypothyroidism may occur as the RAI therapy takes effect. d. to discontinue the antithyroid medications taken before the radioactive therapy. ANSWER C

A 61-year-old female patient admitted with pneumonia has a total serum calcium level of 13.3 mg/dL (3.3 mmol/L). The nurse will anticipate the need to teach the patient about testing for _____ levels. a. calcitonin b. catecholamine c. thyroid hormone d. parathyroid hormone

D

An 18-year-old male patient with a small stature is scheduled for a growth hormone stimulation test. In preparation for the test, the nurse will obtain a. ice in a basin. b. glargine insulin. c. a cardiac monitor. d. 50% dextrose solution.

D

Which statement by a 50-year-old female patient indicates to the nurse that further assessment of thyroid function may be necessary? a. "I notice my breasts are tender lately." b. "I am so thirsty that I drink all day long." c. "I get up several times at night to urinate." d. "I feel a lump in my throat when I swallow."

D

A patient suspected of having acromegaly has an elevated plasma growth hormone (GH) level. In acromegaly, the nurse would also expect the patient's diagnostic results to include a. hyperinsulinemia b. a plasma glucose of < 70 mg/dL c. decreased GH levels with an oral glucose challenge test d. elevated serum somatomedin C (insulin like growth factor-1 (IGF-1))

D- A normal response to growth hormone (GH) secretion is stimulation of the liver to produce somatomedin C, or IGF-1, which stimulates growth of bones and soft tissues. The increased levels of somatomedin C normally inhibit GH, but in acromegaly, the pituitary gland secretes GH despite elevated IGF-1 levels. When both GH and IGF-1 levels are increased, overproduction of GH is confirmed. GH also causes elevation of blood glucose, and normally GH levels fall during an oral glucose challenge but not in acromegaly.

The nurse coordinates postoperative care for a 70-year-old man with osteoarthritis after prostate surgery. Which task is appropriate for the nurse to delegate to a licensed practical/vocational nurse (LPN/LVN)? A. Teach the patient how to perform Kegel exercises. B. Provide instructions to the patient on catheter care. C. Administer oxybutynin (Ditropan) for bladder spasms. D. Manually irrigate the urinary catheter to determine patency.

D. Manually irrigate the urinary catheter to determine patency. The nurse may delegate the following to an LPN/LVN: monitor catheter drainage for increased blood or clots, increase flow of irrigating solution to maintain light pink color in outflow, administer antispasmodics and analgesics as needed. A registered nurse may not delegate teaching, assessments, or clinical judgments to a LPN/LVN.

A 51-year-old woman has recently had a unilateral, right total mastectomy and axillary node dissection for the treatment of breast cancer. Which of the following interventions should the nurse include in the patient's care? A. Immobilize the patient's right arm until postoperative day 3. B. Maintain the patient's right arm in a dependent position when at rest. C. Administer diuretics prophylactically for the prevention of lymphedema. D. Promote gradually increasing mobility as soon as possible following surgery.

D. Mobility should be encouraged beginning in postanesthetic recovery and increased gradually throughout the patient's recovery. Immobilization is counterproductive to recovery and the limb should never be in a dependent position. Diuretics are not used to prevent lymphedema but may be used in active treatment of the problem.

Which task can the nurse delegate to an unlicensed assistive personnel (UAP) in the care of a patient who has recently undergone prostatectomy? A. Assessing the patient's incision B. Irrigating the patient's Foley catheter C. Assessing the patient's pain and selecting analgesia D. Performing cleansing of the meatus and perineal region

D. Performing cleansing of the meatus and perineal region Performing perineal care is an appropriate task for delegation. Selecting analgesia, irrigating the patient's catheter, and assessing his incision are not appropriate skills or tasks for unlicensed personnel.

Which manifestations are characteristic of the late or tertiary stage of syphilis (select all that apply)? a. Heart failure b. Tabes dorsalis c. Saccular aneurysms d. Mental deterioration e. Generalized cutaneous rash f. Destructive skin, bone, and soft tissue lesions

a, b, c, d, f. In the tertiary (or late) stage of syphilis there can be gummas (chronic destructive lesions), cardiovascular problems (heart failure, aneurysms, valve insufficiency), and neurosyphilis manifestations (mental deterioration, tabes dorsalis, and speech disturbances). Generalized cutaneous rash occurs in the secondary stage of syphilis, a few weeks after the chancre appears.

An African American woman with a history of breast cancer has panhypopituitarism from radiation therapy for primary pituitary tumors. Which medications should the nurse teach her about needing for the rest of her life (select all that apply)? a. Cortisol b. Vasopressin c. Sex hormones d. Levothyroxine (Synthroid) e. Growth hormone (somatropin [Omnitrope]) f. Dopamine agonists (bromocriptine [Parlodel])

a, b, d, e. With panhypopituitarism, lifetime hormone replacement is needed for cortisol, vasopressin, thyroid, and GH. Sex hormones will not be replaced because of the patient's history of breast cancer. Dopamine agonists will not be used because they reduce secretion of GH, which has already been achieved with the radiation.

Which characteristics describe the use of RAI (select all that apply)? a. Often causes hypothyroidism over time b. Decreases release of thyroid hormones c. Blocks peripheral conversion of T4 to T3 d. Treatment of choice in nonpregnant adults e. Decreases thyroid secretion by damaging thyroid gland f. Often used with iodine to produce euthyroid before surgery

a, d, e. RAI causes hypothyroidism over time by damaging thyroid tissue and is the treatment of choice for nonpregnant adults. Potassium iodide decreases the release of thyroid hormones and decreases the size of the thyroid gland preoperatively. Propylthiouracil (PTU) blocks peripheral conversion of T4 to T3 and may be used with iodine to produce a euthyroid state before surgery.

A 45-year-old male patient with suspected acromegaly is seen at the clinic. To assist in making the diagnosis, which question should the nurse ask?

a. "Have you had a recent head injury?" B. "Do you have to wear larger shoes now?" c. "Is there a family history of acromegaly?" d. "Are you experiencing tremors or anxiety?" ANSWER B, Do you have to wear larger shoes now.

Which question will the nurse in the endocrine clinic ask to help determine a patient's risk factors for goiter?

a. "How much milk do you drink?" b. "What medications are you taking?" c. "Are your immunizations up to date?" d. "Have you had any recent neck injuries?" ANSWER B

A 38-year-old male patient is admitted to the hospital in Addisonian crisis. Which patient statement supports a nursing diagnosis of ineffective self-health management related to lack of knowledge about management of Addison's disease?

a. "I frequently eat at restaurants, and my food has a lot of added salt." b. "I had the stomach flu earlier this week, so I couldn't take the hydrocortisone." c. "I always double my dose of hydrocortisone on the days that I go for a long run." d. "I take twice as much hydrocortisone in the morning dose as I do in the afternoon." ANSWER B

A 29-year-old woman with systemic lupus erythematosus has been prescribed 2 weeks of high-dose prednisone therapy. Which information about the prednisone is most important for the nurse to include?

a. "Weigh yourself daily to monitor for weight gain caused by increased appetite." b. "A weight-bearing exercise program will help minimize the risk for osteoporosis." c. "The prednisone dose should be decreased gradually rather than stopped suddenly." d. "Call the health care provider if you experience mood alterations with the prednisone." ANSWER C

After receiving change-of-shift report about the following four patients, which patient should the nurse assess first?

a. A 31-year-old female with Cushing syndrome and a blood glucose level of 244 mg/dL b. A 70-year-old female taking levothyroxine (Synthroid) who has an irregular pulse of 134 c. A 53-year-old male who has Addison's disease and is due for a scheduled dose of hydrocortisone (Solu-Cortef). d. A 22-year-old male admitted with syndrome of inappropriate antidiuretic hormone (SIADH) who has a serum sodium level of 130 mEq/L ANSWER B

As a precaution for vocal cord paralysis from damage to the recurrent laryngeal nerve during thyroidectomy surgery, what equipment should be in the room in case it is needed for this emergency situation? a. Tracheostomy tray c. IV calcium gluconate b. Oxygen equipment d. Paper and pencil for communication

a. A tracheostomy tray is in the room to use if vocal cord paralysis occurs from recurrent laryngeal nerve damage or for laryngeal stridor from tetany. The oxygen equipment may be useful but will not improve oxygenation with vocal cord paralysis without a tracheostomy. IV calcium salts will be used if hypocalcemia occurs from parathyroid damage. The paper and pencil for communication may be helpful, especially if a tracheostomy is performed, but will not aid in emergency oxygenation of the patient.

A patient with acromegaly is treated with a transsphenoidal hypophysectomy. What should the nurse do postoperatively? a. Ensure that any clear nasal drainage is tested for glucose. b. Maintain the patient flat in bed to prevent cerebrospinal fluid (CSF) leakage. c. Assist the patient with toothbrushing every 4 hours to keep the surgical area clean. d. Encourage deep breathing, coughing, and turning to prevent respiratory complications.

a. A transsphenoidal hypophysectomy involves entry into the sella turcica through an incision in the upper lip and gingiva into the floor of the nose and the sphenoid sinuses. Postoperative clear nasal drainage with glucose content indicates cerebrospinal fluid (CSF) leakage from an open connection to the brain, putting the patient at risk for meningitis. After surgery, the patient is positioned with the head elevated to avoid pressure on the sella turcica. Coughing and straining are avoided to prevent increased intracranial pressure and CSF leakage. Although mouth care is required every 4 hours, toothbrushing should not be performed because injury to the suture line may occur.

A patient develops carpopedal spasms and tingling of the lips following a parathyroidectomy. Which action should the nurse take first?

a. Administer the ordered muscle relaxant. b. Give the ordered oral calcium supplement. C. HAVE THE PATIENT REBREATHE FROM A PAPER BAG. d. Start the PRN oxygen at 2 L/min per cannula. ANSWER C

Which finding by the nurse when assessing a patient with a large pituitary adenoma is most important to report to the health care provider?

a. Changes in visual field b. Milk leaking from breasts c. Blood glucose 150 mg/dL d. Nausea and projectile vomiting ANSWER D

A 37-year-old patient is being admitted with a diagnosis of Cushing syndrome. Which findings will the nurse expect during the assessment?

a. Chronically low blood pressure b. Bronzed appearance of the skin c. Purplish streaks on the abdomen d. Decreased axillary and pubic hair ANSWER C

Which information will the nurse teach a 48-year-old patient who has been newly diagnosed with Graves' disease?

a. Exercise is contraindicated to avoid increasing metabolic rate. b. Restriction of iodine intake is needed to reduce thyroid activity. C. Antithyroid medications may take several months for full effect. d. Surgery will eventually be required to remove the thyroid gland. ANSWER C

Which statement accurately describes Graves' disease? a. Exophthalmos occurs in Graves' disease. b. It is an uncommon form of hyperthyroidism. c. Manifestations of hyperthyroidism occur from tissue desensitization to the sympathetic nervous system. d. Diagnostic testing in the patient with Graves' disease will reveal an increased thyroid-stimulating hormone (TSH) level.

a. Exophthalmos or protrusion of the eyeballs may occur in Graves' disease from increased fat deposits and fluid in the orbital tissues and ocular muscles, forcing the eyeballs outward. Graves' disease is the most common form of hyperthyroidism. Increased metabolic rate and sensitivity of the sympathetic nervous system lead to the clinical manifestations. Thyroid-stimulating hormone (TSH) level is decreased in Graves' disease.

Which nursing assessment of a 69-year-old patient is most important to make during initiation of thyroid replacement with levothyroxine (Synthroid)?

a. Fluid balance B. Apical pulse rate c. Nutritional intake d. Orientation and alertness ANSWER B

After obtaining the information shown in the accompanying figure regarding a patient with Addison's disease, which prescribed action will the nurse take first?

a. Give 4 oz of fruit juice orally. b. Recheck the blood glucose level. c. Infuse 5% dextrose and 0.9% saline. d. Administer oxygen therapy as needed. ANSWER C

When caring for a patient with primary hyperaldosteronism, the nurse would question a health care provider's prescription for which drug? a. Furosemide (Lasix) c. Spironolactone (Aldactone) b. Amiloride (Midamor) d. Aminoglutethimide (Cytadren)

a. Hyperaldosteronism is an excess of aldosterone, which is manifested by sodium and water retention and potassium excretion. Furosemide is a potassium-wasting diuretic that would increase the potassium deficiency. Aminoglutethimide blocks aldosterone synthesis. Spironolactone and amiloride are potassium-sparing diuretics.

Which finding for a patient who has hypothyroidism and hypertension indicates that the nurse should contact the health care provider before administering levothyroxine (Synthroid)?

a. Increased thyroxine (T4) level b. Blood pressure 112/62 mm Hg c. Distant and difficult to hear heart sounds d. Elevated thyroid stimulating hormone level ANSWER A

Which finding indicates to the nurse that the current therapies are effective for a patient with acute adrenal insufficiency?

a. Increasing serum sodium levels b. Decreasing blood glucose levels c. Decreasing serum chloride levels d. Increasing serum potassium levels ANSWER A

Which stage of syphilis is identified by the absence of clinical manifestations and a positive fluorescent treponemal antibody absorption (FTA-Abs) test? a. Latent b. Primary c. Secondary d. Late (tertiary)

a. Lack of clinical manifestations but a positivetreponemal antibody test with normal cerebrospinal fluid (CSF) occurs in the latent stage. The primary stage is characterized by a chancre, regional lymphadenopathy, and genital ulcers. The secondary stage has flu-like symptoms and cutaneous lesions. The late or tertiary stage is characterized by gummas, cardiovascular changes, and neurosyphilis.

Which intervention will the nurse include in the plan of care for a 52-year-old male patient with syndrome of inappropriate antidiuretic hormone (SIADH)?

a. Monitor for peripheral edema. b. Offer patient hard candies to suck on. c. Encourage fluids to 2 to 3 liters per day. d. Keep head of bed elevated to 30 degrees. ANSWER B

Which finding by the nurse when assessing a patient with Hashimoto's thyroiditis and a goiter will require the most immediate action?

a. New-onset changes in the patient's voice b. Apical pulse rate at rest 112 beats/minute c. Elevation in the patient's T3 and T4 levels d. Bruit audible bilaterally over the thyroid gland ANSWER A

A patient has just arrived on the unit after a thyroidectomy. Which action should the nurse take first?

a. Observe the dressing for bleeding. b. Check the blood pressure and pulse. c. Assess the patient's respiratory effort. d. Support the patient's head with pillows. ANSWER C

The nurse is planning postoperative care for a patient who is being admitted to the surgical unit from the recovery room after transsphenoidal resection of a pituitary tumor. Which nursing action should be included? ANSWER B

a. Palpate extremities for edema. B. Measure urine volume every hour. c. Check hematocrit every 2 hours for 8 hours. d. Monitor continuous pulse oximetry for 24 hours. ANSWER B, measure iron volume every hour.

The cardiac telemetry unit charge nurse receives status reports from other nursing units about four patients who need cardiac monitoring. Which patient should be transferred to the cardiac unit first?

a. Patient with Hashimoto's thyroiditis and a heart rate of 102 b. Patient with tetany who has a new order for IV calcium chloride c. Patient with Cushing syndrome and a blood glucose of 140 mg/dL d. Patient with Addison's disease who takes hydrocortisone twice daily ANSWER B

Which information obtained by the nurse in the endocrine clinic about a patient who has been taking prednisone (Deltasone) 40 mg daily for 3 weeks is most important to report to the health care provider?

a. Patient's blood pressure is 148/94 mm Hg. b. Patient has bilateral 2+ pitting ankle edema. c. Patient stopped taking the medication 2 days ago. d. Patient has not been taking the prescribed vitamin D. ANSWER C

Which nursing action will be included in the plan of care for a 55-year-old patient with Graves' disease who has exophthalmos?

a. Place cold packs on the eyes to relieve pain and swelling. B. Elevate the head of the patient's bed to reduce periorbital fluid. c. Apply alternating eye patches to protect the corneas from irritation. d. Teach the patient to blink every few seconds to lubricate the corneas. ANSWER B

Which assessment finding for a 33-year-old female patient admitted with Graves' disease requires the most rapid intervention by the nurse? a. Bilateral exophthalmos b. Heart rate 136 beats/minute c. Temperature 103.8° F (40.4° C) d. Blood pressure 166/100 mm Hg

a. Propranolol (Inderal) b. Propylthiouracil (PTU) c. Methimazole (Tapazole) d. Iodine (Lugol's solution) ANSWER C

A 44-year-old female patient with Cushing syndrome is admitted for adrenalectomy. Which intervention by the nurse will be most helpful for a nursing diagnosis of disturbed body image related to changes in appearance?

a. Reassure the patient that the physical changes are very common in patients with Cushing syndrome. b. Discuss the use of diet and exercise in controlling the weight gain associated with Cushing syndrome. c. Teach the patient that the metabolic impact of Cushing syndrome is of more importance than appearance. d. Remind the patient that most of the physical changes caused by Cushing syndrome will resolve after surgery. ANSWER D

A 63-year-old patient with primary hyperparathyroidism has a serum phosphorus level of 1.7 mg/dL (0.55 mmol/L) and calcium of 14 mg/dL (3.5 mmol/L). Which nursing action should be included in the plan of care?

a. Restrict the patient to bed rest. B. Encourage 4000 mL of fluids daily. c. Institute routine seizure precautions. d. Assess for positive Chvostek's sign. ANSWER B

Patients with which STI are most likely to avoid obtaining and following treatment measures for their infection? a. Syphilis b. Gonorrhea c. HPV infection d. Genital herpes

a. STIs, such as syphilis, that can be treated with a single dose or short course of antibiotic therapy often lead to a casual attitude about the outcome of the disease, which leads to nonadherence with instructions and delays in treatment. This is particularly true of diseases that initially show few distressing or uncomfortable symptoms, such as syphilis.

A patient who had a subtotal thyroidectomy earlier today develops laryngeal stridor and a cramp in the right hand upon returning to the surgical nursing unit. Which collaborative action will the nurse anticipate next?

a. Suction the patient's airway. B. Administer IV calcium gluconate. c. Plan for emergency tracheostomy. d. Prepare for endotracheal intubation. ANSWER B

Which assessment finding of a 42-year-old patient who had a bilateral adrenalectomy requires the most rapid action by the nurse?

a. The blood glucose is 176 mg/dL. b. The lungs have bibasilar crackles. c. The blood pressure (BP) is 88/50 mm Hg. d. The patient reports 5/10 incisional pain. ANSWER C

A patient is admitted to the hospital with a diagnosis of Cushing syndrome. On physical assessment of the patient, what should the nurse expect to find? a. Hypertension, peripheral edema, and petechiae b. Weight loss, buffalo hump, and moon face with acne c. Abdominal and buttock striae, truncal obesity, and hypotension d. Anorexia, signs of dehydration, and hyperpigmentation of the skin

a. The effects of adrenocortical hormone excess, especially glucocorticoid excess, include weight gain from accumulation and redistribution of adipose tissue, sodium and water retention, glucose intolerance, protein wasting, loss of bone structure, loss of collagen, and capillary fragility leading to petechiae. Clinical manifestations of adrenocortical hormone deficiency include hypotension, dehydration, weight loss, and hyperpigmentation of the skin.

Which information will the nurse include when teaching a 50-year-old male patient about somatropin (Genotropin)?

a. The medication will be needed for 3 to 6 months. B. Inject the medication subcutaneously every day. c. Blood glucose levels may decrease when taking the medication. d. Stop taking the medication if swelling of the hands or feet occurs. ANSWER B, inject the medication subcutaneously every day.

Which information is most important for the nurse to communicate rapidly to the health care provider about a patient admitted with possible syndrome of inappropriate antidiuretic hormone (SIADH)?

a. The patient has a recent weight gain of 9 lb. b. The patient complains of dyspnea with activity. c. The patient has a urine specific gravity of 1.025. d. The patient has a serum sodium level of 118 mEq/L. ANSWER D

The nurse is caring for a patient admitted with diabetes insipidus (DI). Which information is most important to report to the health care provider?

a. The patient is confused and lethargic. b. The patient reports a recent head injury. c. The patient has a urine output of 400 mL/hr. d. The patient's urine specific gravity is 1.003. ANSWER A

A 37-year-old patient has just arrived in the postanesthesia recovery unit (PACU) after a thyroidectomy. Which information is most important to communicate to the surgeon?

a. The patient reports 7/10 incisional pain. b. The patient has increasing neck swelling. c. The patient is sleepy and difficult to arouse. d. The patient's cardiac rate is 112 beats/minute. ANSWER B

During care of the patient with SIADH, what should the nurse do? a. Monitor neurologic status at least every 2 hours. b. Teach the patient receiving treatment with diuretics to restrict sodium intake. c. Keep the head of the bed elevated to prevent antidiuretic hormone (ADH) release. d. Notify the health care provider if the patient's blood pressure decreases more than 20 mm Hg from baseline.

a. The patient with syndrome of inappropriate antidiuretic hormone (SIADH) has marked dilutional hyponatremia and should be monitored for decreased neurologic function and seizures every 2 hours. Sodium intake is supplemented because of the hyponatremia and sodium loss caused by diuretics. ADH release is reduced by keeping the head of the bed flat to increase left atrial filling pressure. A reduction in blood pressure (BP) indicates a reduction in total fluid volume and is an expected outcome of treatment.

Priority Decision: What is most important for the nurse to teach the female patient with genital warts? a. Have an annual Papanicolaou (Pap) test. b. Apply topical acyclovir faithfully as directed. c. Have her sexual partner treated for the condition. d. Use a contraceptive to prevent pregnancy, which might exacerbate the disease.

a. There is a strong association of genital warts with the development of dysplasia and neoplasia of the genital tract, especially when lesions involve the cervix, introitus, and perianal and intraanal mucosa of women or the penis and perianal and anal mucosa of men. Regular Papanicolaou (Pap) tests in women are critical in detecting early malignancies of the cervix. Oral acyclovir is used to treat HSV-2 but topical use has no value in treating viral STIs. Sexual partners of patients with HPV should be examined and treated but because treatment does not destroy the virus, condoms should always be used during sexual activity. Genital warts often grow more rapidly during pregnancy but pregnancy is not contraindicated.

A 23-year-old patient is admitted with diabetes insipidus. Which action will be most appropriate for the registered nurse (RN) to delegate to an experienced licensed practical/vocational nurse (LPN/LVN)?

a. Titrate the infusion of 5% dextrose in water. b. Teach the patient how to use desmopressin (DDAVP) nasal spray. c. Assess the patient's hydration status every 8 hours. d. Administer subcutaneous DDAVP. ANSWER D

What preoperative instruction should the nurse give to the patient scheduled for a subtotal thyroidectomy? a. How to support the head with the hands when turning in bed b. Coughing should be avoided to prevent pressure on the incisionm c. Head and neck will need to remain immobile until the incision heals d. Any tingling around the lips or in the fingers after surgery is expected and temporary

a. To prevent strain on the suture line postoperatively, the patient's head must be manually supported while turning and moving in bed but range-of-motion exercises for the head and neck are also taught preoperatively to be gradually implemented after surgery. There is no contraindication for coughing and deep breathing and these should be carried out postoperatively. Tingling around the lips or fingers is a sign of hypocalcemia, which may occur if the parathyroid glands are inadvertently removed during surgery. This sign should be reported immediately.

Identify the rationale for having the following items immediately available in the patient's room following thyroidectomy a. Tracheostomy tray b. Calcium salts for IV administration c. Oxygen equipment

a. To use in case airway obstruction occurs because of vocal cord paralysis from recurrent laryngeal nerve damage during surgery or laryngeal stridor occurs with tetany b. needed in case hypocalcemia occurs form parathyroid gland removal or damage during surgery, resulting in tetany c. in case of airway obstruction, laryngeal stridor or edema around trachea

A 22-year-old woman with multiple sexual partners seeks care after several weeks of experiencing painful and frequent urination and vaginal discharge. Although the results of a culture of cervical secretions are not yet available, the nurse explains to the patient that she will be treated as if she has gonorrhea and chlamydia to prevent a. obstruction of the fallopian tubes. b. endocarditis and aortic aneurysms. c. disseminated gonococcal infection. d. polyarthritis and generalized adenopathy.

a. Upward extension of gonorrhea or chlamydia commonly causes PID, which can cause adhesions and fibrous scarring, leading to tubal pregnancies and infertility. Disseminated gonococcal infection is rare and endocarditis and aneurysms are associated with syphilis. Polyarthritis and adenopathy are not seen in gonorrhea or chlamydia.

A patient who had radical neck surgery to remove a malignant tumor developed hypoparathyroidism. The nurse should plan to teach the patient about

a. bisphosphonates to reduce bone demineralization. b. calcium supplements to normalize serum calcium levels. c. increasing fluid intake to decrease risk for nephrolithiasis. d. including whole grains in the diet to prevent constipation. ANSWER B

A 42-year-old female patient is scheduled for transsphenoidal hypophysectomy to treat a pituitary adenoma. During preoperative teaching, the nurse instructs the patient about the need to

a. cough and deep breathe every 2 hours postoperatively. b. remain on bed rest for the first 48 hours after the surgery. C. avoid brushing teeth for at least 10 days after the surgery. d. be positioned flat with sandbags at the head postoperatively. ANSWER C, avoid brushing teeth for at least 10 days after surgery.

An 82-year-old patient in a long-term care facility has several medications prescribed. After the patient is newly diagnosed with hypothyroidism, the nurse will need to consult with the health care provider before administering

a. docusate (Colace). b. ibuprofen (Motrin). C. Diazepam (Valium). d. cefoxitin (Mefoxin). ANSWER C

A 56-year-old patient who is disoriented and reports a headache and muscle cramps is hospitalized with possible syndrome of inappropriate antidiuretic hormone (SIADH). The nurse would expect the initial laboratory results to include a(n)

a. elevated hematocrit. B. decreased serum sodium. c. low urine specific gravity. d. increased serum chloride. ANSWER B

An expected nursing diagnosis for a 30-year-old patient admitted to the hospital with symptoms of diabetes insipidus is

a. excess fluid volume related to intake greater than output. b. impaired gas exchange related to fluid retention in lungs. C. sleep pattern disturbance related to frequent waking to void. d. risk for impaired skin integrity related to generalized edema. ANSWER C

The nurse will plan to monitor a patient diagnosed with a pheochromocytoma for

a. flushing. b. headache. c. bradycardia. d. hypoglycemia. ANSWER B

The nurse is assessing a 41-year-old African American male patient diagnosed with a pituitary tumor causing panhypopituitarism. Assessment findings consistent with panhypopituitarism include

a. high blood pressure. B. decreased facial hair. c. elevated blood glucose. d. tachycardia and cardiac palpitations. ANSWER is B, deceased facial hair.

A 56-year-old female patient has an adrenocortical adenoma, causing hyperaldosteronism. The nurse providing care should

a. monitor the blood pressure every 4 hours. b. elevate the patient's legs to relieve edema. c. monitor blood glucose level every 4 hours. d. order the patient a potassium-restricted diet. ANSWER A

The nurse is caring for a patient following an adrenalectomy. The highest priority in the immediate postoperative period is to

a. protect the patient's skin. b. monitor for signs of infection. c. balance fluids and electrolytes. d. prevent emotional disturbances. ANSWER C

A patient who is on corticosteroid therapy for treatment of an autoimmune disorder has the following additional drugs ordered. How is the need for these drugs related to the effects of corticosteroids a. Furosemide (Lasix) b. Pantoprazole (Protonix) c. Alendronate (Fosamax) d. Insulin e. Potassium

a. sodium and fluid retention because of mineralcorticoid effect b. gastrointestinal irritation with an increase in secretion of pepsin and hydrochloric acid c. corticosteroid induced osteoperosis

After a 22-year-old female patient with a pituitary adenoma has had a hypophysectomy, the nurse will teach about the need for

a. sodium restriction to prevent fluid retention. b. insulin to maintain normal blood glucose levels. c. oral corticosteroids to replace endogenous cortisol. d. chemotherapy to prevent malignant tumor recurrence. ANSWER C

A patient is to receive methylprednisolone (Solu-Medrol) 100 mg. The label on the medication states: methylprednisolone 125 mg in 2 mL. How many milliliters will the nurse administer?

answer 1.6

What do you tell the patient with chronic bacterial prostatitis who is undergoing antibiotic therapy (select all that apply)? A. Hospitalization is required of all patients. B. Pain will lessen after treatment has ended. C. The course of treatment usually is 2 to 4 weeks. D. Long-term therapy may be indicated for immunocompromised patients. E. If the condition is unresolved and untreated, the patient is at risk for prostate cancer.

answer: B, D Patients with chronic bacterial prostatitis are usually given antibiotics for 4 to 12 weeks. However, antibiotics may be given for a lifetime if the patient is immunocompromised. Although patients with chronic bacterial prostatitis tend to experience a great amount of discomfort, the pain resolves as the infection is treated. If the patient with acute bacterial prostatitis has high fever or other signs of impending sepsis, hospitalization and intravenous antibiotics are prescribed.

A patient has been diagnosed with hypoparathyroidism. What manifestations should the nurse expect to observe (select all that apply)? a. Skeletal pain b. Dry, scaly skin c. Personality changes d. Abdominal cramping e. Cardiac dysrhythmias f. Muscle spasms and stiffness

b, c, d, e, f. In hypoparathyroidism the patient has inadequate circulating parathyroid hormone (PTH) that leads to hypocalcemia from the inability to maintain serum calcium levels. With hypocalcemia there is muscle stiffness and spasms, which can lead to cardiac dysrhythmias and abdominal cramps. There can also be personality and visual changes and dry, scaly skin.

What is an appropriate nursing intervention for the patient with hyperparathyroidism? a. Pad side rails as a seizure precaution. b. Increase fluid intake to 3000 to 4000 mL daily. c. Maintain bed rest to prevent pathologic fractures. d. Monitor the patient for Trousseau's and Chvostek's signs.

b. A high fluid intake is indicated in hyperparathyroidism to dilute the hypercalcemia and flush the kidneys so that calcium stone formation is reduced. Seizures are not associated with hyperparathyroidism. Impending tetany of hypoparathyroidism after parathyroidectomy can be noted with Trousseau's and Chvostek's signs. The patient with hyperparathyroidism is at risk for pathologic fractures resulting from decreased bone density but mobility is encouraged to promote bone calcification.

A patient with SIADH is treated with water restriction. What does the patient experience when the nurse determines that treatment has been effective? a. Increased urine output, decreased serum sodium, and increased urine specific gravity b. Increased urine output, increased serum sodium, and decreased urine specific gravity c. Decreased urine output, increased serum sodium, and decreased urine specific gravity d. Decreased urine output, decreased serum sodium, and increased urine specific gravity

b. The patient with SIADH has water retention with hyponatremia, decreased urine output, and concentrated urine with high specific gravity. Improvement in the patient's condition is reflected by increased urine output, normalization of serum sodium, and more water in the urine, thus decreasing the specific gravity.

During discharge teaching for the patient with Addison's disease, which statement by the patient indicates that the nurse needs to do additional teaching? a. "I should always call the doctor if I develop vomiting or diarrhea." b. "If my weight goes down, my dosage of steroid is probably too high." c. "I should double or triple my steroid dose if I undergo rigorous physical exercise." d. "I need to carry an emergency kit with injectable hydrocortisone in case I can't take my medication by mouth."

b. A weight reduction in the patient with Addison's disease may indicate a fluid loss and a dose of replacement therapy that is too low rather than too high. Because vomiting and diarrhea are early signs of crisis and because fluid and electrolytes must be replaced, patients should notify their health care provider if these symptoms occur. Patients with Addison's disease are taught to take two to three times their usual dose of steroids if they become ill, have teeth extracted, or engage in rigorous physical activity and should always have injectable hydrocortisone available if oral doses cannot be taken.

A patient with hypothyroidism is treated with levothyroxine (Synthroid). What should the nurse include when teaching the patient about this therapy? a. Explain that alternate-day dosage may be used if side effects occur. b. Provide written instruction for all information related to the drug therapy. c. Assure the patient that a return to normal function will occur with replacement therapy. d. Inform the patient that the drug must be taken until the hormone balance is reestablished.

b. Because of the mental sluggishness, inattentiveness, and memory loss that occur with hypothyroidism, it is important to provide written instructions and repeat information when teaching the patient. Replacement therapy must be taken for life and alternate-day dosing is not therapeutic. Although most patients return to a normal state with treatment, cardiovascular conditions and psychoses may persist.

The nurse has identified the nursing diagnosis of fatigue for a patient who is hypothyroid. What should the nurse do while caring for this patient? a. Monitor for changes in orientation, cognition, and behavior. b. Monitor for vital signs and cardiac rhythm response to activity. c. Monitor bowel movement frequency, consistency, shape, volume, and color. d. Assist in developing well-balanced meal plans consistent with level of energy expenditure.

b. Cardiorespiratory response to activity is important to monitor in this patient to determine the effect of activities and plan activity increases. Monitoring changes in orientation, cognition, and behavior are interventions for impaired memory. Monitoring bowels is needed to plan care for the patient with constipation. Assisting with meal planning will help the patient with imbalanced nutrition: more than body requirements to lose weight if needed.

When caring for a patient with nephrogenic diabetes insipidus, what should the nurse expect the treatment to include? a. Fluid restriction b. Thiazide diuretics c. A high-sodium diet d. Chlorpropamide (Diabinese)

b. In nephrogenic diabetes insipidus, the kidney is unable to respond to ADH, so vasopressin or hormone analogs are not effective. Thiazide diuretics slow the glomerular filtration rate (GFR) in the kidney and produce a decrease in urine output. Low-sodium diets (<3 g/day) are also thought to decrease urine output. Fluids are not restricted because the patient could easily become dehydrated.

A male patient returns to the clinic with a recurrent urethral discharge after being treated for a chlamydial infection 2 weeks ago. Which statement by the patient indicates the most likely cause of the recurrence of his infection? a. "I took the Vibramycin twice a day for a week." b. "I haven't told my girlfriend about my infection yet." c. "I had a couple of beers while I was taking the medication." d. "I've only had sexual intercourse once since my medication was finished."

b. Notification and treatment of sexual partners are necessary to prevent recurrence and the "ping-pong effect" of passing STIs between partners. Vibramycin is prescribed twice a day for 7 days and although alcohol may cause more urinary irritation in the patient with chlamydia, it will not interfere with treatment.

Priority Decision: What is the priority nursing intervention during the management of the patient with pheochromocytoma? a. Administering IV fluids b. Monitoring blood pressure c. Administering β-adrenergic blockers d. Monitoring intake and output and daily weights

b. Pheochromocytoma is a catecholamine-producing tumor of the adrenal medulla, which may cause severe, episodic hypertension; severe, pounding headache; and profuse sweating. Monitoring for a dangerously high BP before surgery is critical, as is monitoring for BP fluctuations during medical and surgical treatment.

When the patient with parathyroid disease experiences symptoms of hypocalcemia, what is a measure that can be used to temporarily raise serum calcium levels? a. Administer IV normal saline. b. Have patient rebreathe in a paper bag. c. Administer furosemide (Lasix) as ordered. d. Administer oral phosphorus supplements.

b. Rebreathing in a paper bag promotes carbon dioxide retention in the blood, which lowers pH and creates an acidosis. An acidemia enhances the solubility and ionization of calcium, increasing the proportion of total body calcium available in physiologically active form and relieving the symptoms of hypocalcemia. Saline promotes calcium excretion, as does furosemide. Phosphate levels in the blood are reciprocal to calcium and an increase in phosphate promotes calcium excretion.

What is the most common way to determine a diagnosis of chlamydial infection in a male patient? a. Cultures for chlamydial organisms are positive. b. The nucleic acid amplification test (NAAT) is positive. c. Gram stain smears and cultures are negative for gonorrhea. d. Signs and symptoms of epididymitis or proctitis are also present.

b. The nucleic acid amplification test (NAAT) is more sensitive than other diagnostic tests, can be done with a urine sample, and has results within 24 hours. A cell culture can be used to detect chlamydia organisms but it requires specific handling and is not as easy or as fast to perform as the NAAT. Gonorrhea and chlamydia have very similar symptoms in men and frequently occur together. Gram stain smears and cultures for N. gonorrhoeae do not definitively diagnose Chlamydia. Manifestations of epididymitis or proctitis may be present, as with other STIs, but are not diagnostic.

What medication is used with thyrotoxicosis to block the effects of the sympathetic nervous stimulation of the thyroid hormones? a. Potassium iodide b. Atenolol (Tenormin) c. Propylthiouracil (PTU) d. Radioactive iodine (RAI)

b. The β-adrenergic blocker atenolol is used to block the sympathetic nervous system stimulation by thyroid hormones. Potassium iodide is used to prepare the patient for thyroidectomy or for treatment of thyrotoxic crisis to inhibit the synthesis of thyroid hormones. Antithyroid medications inhibit the synthesis of thyroid hormones. Radioactive iodine (RAI) therapy destroys thyroid tissue, which limits thyroid hormone secretion.

A patient with Addison's disease comes to the emergency department with complaints of nausea, vomiting, diarrhea, and fever. What collaborative care should the nurse expect? a. IV administration of vasopressors b. IV administration of hydrocortisone c. IV administration of D5W with 20 mEq KCl d. Parenteral injections of adrenocorticotropic hormone (ACTH)

b. Vomiting and diarrhea are early indicators of Addisonian crisis and fever indicates an infection, which is causing additional stress for the patient. Treatment of a crisis requires immediate glucocorticoid replacement and IV hydrocortisone, fluids, sodium, and glucose are necessary for 24 hours. Addison's disease is a primary insufficiency of the adrenal gland and adrenocorticotropic hormone (ACTH) is not effective, nor would vasopressors be effective with the fluid deficiency of Addison's disease. Potassium levels are increased in Addison's disease and KCl would be contraindicated.

The patient is diagnosed with syndrome of inappropriate antidiuretic hormone (SIADH). What manifestation should the nurse expect to find? a. Decreased body weight b. Decreased urinary output c. Increased plasma osmolality d. Increased serum sodium levels

b. With increased antidiuretic hormone (ADH), the permeability of the renal distal tubules is increased, so water is reabsorbed into circulation. Decreased output of concentrated urine with increased urine osmolality and specific gravity occur. In addition, fluid retention with weight gain, serum hypoosmolality, dilutional hyponatremia, and hypochloremia occur.

What are characteristics of a herpes simplex virus infection (select all that apply)? a. Treatment with acyclovir can cure genital herpes. b. Herpes simplex virus type 2 (HSV-2) is capable of causing only genital lesions. c. Recurrent symptomatic genital herpes may be precipitated by sexual activity and stress. d. To prevent transmission of genital herpes, condoms should be used when lesions are present. e. The primary symptom of genital herpes is painful vesicular lesions that rupture and ulcerate.

c, e. Sexual activity and stress may precipitate the recurrence of genital herpes symptoms of painful vesicular lesions that rupture and ulcerate. Acyclovir only decreases recurrences of genital herpes. Herpes simplex virus type 2 (HSV-2) may cause oral or genital lesions. Prevention of the spread of genital herpes is best done with avoidance of sexual activity when lesions are present.

A patient is admitted to the hospital with thyrotoxicosis. On physical assessment of the patient, what should the nurse expect to find? a. Hoarseness and laryngeal stridor b. Bulging eyeballs and dysrhythmias c. Elevated temperature and signs of heart failure d. Lethargy progressing suddenly to impairment of consciousness

c. A hyperthyroid crisis results in marked manifestations of hyperthyroidism, with severe tachycardia, heart failure, shock, hyperthermia, restlessness, irritability, abdominal pain, vomiting, diarrhea, delirium, and coma. Although exophthalmos may be present in the patient with Graves' disease, it is not a significant factor in hyperthyroid crisis. Hoarseness and laryngeal stridor are characteristic of the tetany of hypoparathyroidism and lethargy progressing to coma is characteristic of myxedema coma, a complication of hypothyroidism.

Although an 18-year-old girl knows that abstinence is one way to prevent STIs, she does not consider that as an alternative. She asks the nurse at the clinic if there are other measures for preventing STIs. What should the nurse teach her? a. Abstinence is the only way to prevent STIs. b. Voiding immediately after intercourse will decrease the risk for infection. c. A vaccine can prevent genital warts and cervical cancer caused by some strains of HPV. d. Thorough hand washing after contact with genitals can prevent oral-genital spread of STIs.

c. A vaccine is available for HPV types 6, 11, 16, and 18 that protects against genital warts and cervical cancer. Although sexual abstinence is the most certain method of avoiding all STIs, it is not usually a feasible alternative. Undamaged condoms also serve to protect against infection. Conscientious hand washing and voiding after intercourse are positive hygienic measures that will help to prevent secondary infections but will not prevent STIs.

A patient who is on corticosteroid therapy for treatment of an autoimmune disorder has the following additional drugs ordered. Which one is used to prevent corticosteroid-induced osteoporosis? a. Potassium b. Furosemide (Lasix) c. Alendronate (Fosamax) d. Pantoprazole (Protonix)

c. Alendronate (Fosamax) is used to prevent corticosteroid- induced osteoporosis. Potassium is used to prevent the mineralocorticoid effect of hypokalemia. Furosemide (Lasix) is used to decrease sodium and fluid retention from the mineralocorticoid effect. Pantoprazole (Protonix) is used to prevent gastrointestinal (GI) irritation from an increase in secretion of pepsin and hydrochloric acid.

The current incidence of sexually transmitted infections (STIs) is related in part to what? a. Increased social acceptance of homosexuality b. Increased virulence of organisms causing STIs c. The use of oral agents rather than condoms as contraceptives d. Development of resistance of microorganisms to common antibiotics

c. Although many factors relate to the current sexually transmitted infections (STI) rates, one major factor is the widespread use of oral contraceptives instead of condoms (both male and female). Condoms are the only contraceptive device that protects against STIs.

A patient is scheduled for a bilateral adrenalectomy. During the postoperative period, what should the nurse expect related to the administration of corticosteroids? a. Reduced to promote wound healing b. Withheld until symptoms of hypocortisolism appear c. Increased to promote an adequate response to the stress of surgery d. Reduced because excessive hormones are released during surgical manipulation of adrenal glands

c. Although the patient with Cushing syndrome has excess corticosteroids, removal of the glands and the stress of surgery require that high doses of corticosteroids (cortisone) be administered postoperatively for several days before weaning the dose. The nurse should monitor the patient's vital signs postoperatively to detect whether large amounts of hormones were released during surgical manipulation, obtain morning urine specimens for cortisol measurement to evaluate the effectiveness of the surgery, and provide dressing changes with aseptic technique to avoid infection as usual inflammatory responses are suppressed.

A female patient with a purulent vaginal discharge is seen at an outpatient clinic. The nurse suspects a diagnosis of gonorrhea. How would this STI be treated? a. Oral acyclovir (Zovirax) b. Benzathine penicillin G given IM c. Ceftriaxone (Rocephin) IM or oral cefixime (Suprax) d. Would need a second confirmatory test result before treatment

c. An established diagnosis of gonorrhea is treated with cefixime (Suprax) orally or with a single dose of IM ceftriaxone (Rocephin). If chlamydia is also present, azithromycin (Zithromax) or doxycycline (Vibramycin) may also be used. Gram stain smears are not useful in diagnosing gonorrhea in women because the female genitourinary tract normally harbors a large number of organisms that resemble N. gonorrhoeae and cultures must be performed to confirm the diagnosis in women. Penicillin was used to treat gonorrhea but gonorrhea is now resistant to penicillin. Penicillin G is used to treat syphilis. Although gonorrhea may lead to pelvic inflammatory disease (PID), its diagnosis would not necessarily indicate that the patient has PID.

In establishing screening programs for populations at high risk for STIs, the nurse recognizes that which microorganism causes nongonococcal urethritis in men and cervicitis in women? a. Herpes simplex virus b. Treponema pallidum c. Chlamydia trachomatis d. Neisseria gonorrhoeae

c. Chlamydia trachomatis can cause nongonococcal urethritis in men and cervicitis in women. Herpes simplex virus causes genital herpes. Treponema pallidum causes syphilis. Neisseria gonorrhoeae causes gonorrhea.

The laboratory result of a specimen from a 20-year-old female patient shows human papillomavirus (HPV). What would the nurse suspect the patient's diagnosis to be? a. Syphilis b. Gonorrhea c. Genital warts d. Genital herpes

c. Genital warts are caused by human papillomavirus (HPV). Syphilis is caused by T. pallidum. Gonorrhea is caused by N. gonorrhoeae. Genital herpes are caused by herpes simplex virus.

A premarital blood test for syphilis reveals that a woman has a positive Venereal Disease Research Laboratory (VDRL) test. How should the nurse advise the patient? a. A single dose of penicillin will cure the syphilis. b. She should question her fiancé about prior sexual contacts. c. Additional testing to detect specific antitreponemal antibodies is necessary. d. A lumbar puncture to evaluate cerebrospinal fluid (CSF) is necessary to rule out active syphilis.

c. Many other diseases or conditions may cause falsepositive test results on nontreponemal Venereal Disease Research Laboratory (VDRL) or rapid plasma reagent (RPR) tests and additional testing is needed before a diagnosis is confirmed or treatment is administered. The diagnosis is confirmed by specific treponemal tests, such as the fluorescent antibody absorption (FTA-Abs) test or the TP-PA test. Analysis of CSF is used to diagnose asymptomatic neurosyphilis.

A patient with diabetes insipidus is treated with nasal desmopressin acetate (DDAVP). The nurse determines that the drug is not having an adequate therapeutic effect when the patient experiences a. headache and weight gain. b. nasal irritation and nausea. c. a urine specific gravity of 1.002. d. an oral intake greater than urinary output.

c. Normal urine specific gravity is 1.005 to 1.025 and urine with a specific gravity of 1.002 is very dilute, indicating that there continues to be excessive loss of water and that treatment of diabetes insipidus is inadequate. Headache, weight gain, and oral intake greater than urinary output are signs of volume excess that occur with overmedication. Nasal irritation and nausea may also indicate overdosage.

The patient with diabetes insipidus is brought to the emergency department with confusion and dehydration after excretion of a large volume of urine today even though several liters of fluid were drunk. What is a diagnostic test that the nurse should expect to be done to help make a diagnosis? a. Blood glucose b. Serum sodium level c. Urine specific gravity d. Computed tomography (CT) of the head

c. Patients with diabetes insipidus (DI) excrete large amounts of urine with a specific gravity of less than 1.005. Blood glucose would be tested to diagnose diabetes mellitus. The serum sodium level is expected to be low with DI but is not diagnostic. To diagnose central DI a water deprivation test is required. Then a CT of the head may be done to determine the cause. Nephrogenic DI is differentiated from central DI with determination of the level of ADH after an analog of ADH is given.

A patient with mild iatrogenic Cushing syndrome is on an alternate-day regimen of corticosteroid therapy. What does the nurse explain to the patient about this regimen? a. It maintains normal adrenal hormone balance. b. It prevents ACTH release from the pituitary gland. c. It minimizes hypothalamic-pituitary-adrenal suppression. d. It provides a more effective therapeutic effect of the drug.

c. Taking corticosteroids on an alternate-day schedule for pharmacologic purposes is less likely to suppress ACTH production from the pituitary and prevent adrenal atrophy. Normal adrenal hormone balance is not maintained during glucocorticoid therapy because excessive exogenous hormone is used.

A patient with hypoparathyroidism resulting from surgical treatment of hyperparathyroidism is preparing for discharge. What should the nurse teach the patient? a. Milk and milk products should be increased in the diet. b. Parenteral replacement of parathyroid hormone will be required for life. c. Calcium supplements with vitamin D can effectively maintain calcium balance. d. Bran and whole-grain foods should be used to prevent GI effects of replacement therapy.

c. The hypocalcemia that results from PTH deficiency is controlled with calcium and vitamin D supplementation and possibly oral phosphate binders. Replacement with PTH is not used because of antibody formation to PTH, the need for parenteral administration, and cost. Milk products, although good sources of calcium, also have high levels of phosphate, which reduce calcium absorption. Whole grains and foods containing oxalic acid also impair calcium absorption.

During assessment of the patient with acromegaly, what should the nurse expect the patient to report? a. Infertility b. Dry, irritated skin m c. Undesirable changes in appearance d. An increase in height of 2 to 3 inches a year

c. The increased production of GH in acromegaly causes an increase in thickness and width of bones and enlargement of soft tissues, resulting in marked changes in facial features, oily and coarse skin, and speech difficulties. Infertility is not a common finding because GH is usually the only pituitary hormone involved in acromegaly. Height is not increased in adults with GH excess because the epiphyses of the bones are closed.

Why should the nurse encourage serologic testing for human immunodeficiency virus (HIV) in the patient with syphilis? a. Syphilis is more difficult to treat in patients with HIV infection. b. The presence of HIV infection increases the risk of contracting syphilis. c. The incidence of syphilis is increased in those with high rates of sexual promiscuity and drug abuse. d. Central nervous system (CNS) involvement is more common in patients with HIV infection and syphilis.

c. The risk factors of drug abuse and sexual promiscuity are found in patients with both syphilis and human immunodeficiency virus (HIV) infection and persons at highest risk for acquiring syphilis are also at high risk for acquiring HIV. Syphilitic lesions on the genitals enhance HIV transmission. Also, HIV-infected patients with syphilis appear to be at greatest risk for central nervous system (CNS) involvement and may require more intensive treatment with penicillin to prevent this complication of HIV.

A patient suspected of having acromegaly has an elevated plasma growth hormone (GH) level. In acromegaly, what would the nurse also expect the patient's diagnostic results to indicate? a. Hyperinsulinemia b. Plasma glucose of <70 mg/dL (3.9 mmol/L) c. Decreased GH levels with an oral glucose challenge test d. Elevated levels of plasma insulin-like growth factor-1 (IGF-1)

d. A normal response to growth hormone (GH) secretion is stimulation of the liver to produce somatomedin C, or insulin-like growth factor-1 (IGF-1), which stimulates growth of bones and soft tissues. The increased levels of somatomedin C normally inhibit GH but in acromegaly the pituitary gland secretes GH despite elevated IGF-1 levels. When both GH and IGF-1 levels are increased, overproduction of GH is confirmed. GH also causes elevation of blood glucose and normally GH levels fall during an oral glucose challenge but not in acromegaly.

In a patient with central diabetes insipidus, what will the administration of ADH during a water deprivation test result in? a. Decrease in body weight c. Decrease in blood pressure b. Increase in urinary output d. Increase in urine osmolality

d. A patient with central diabetes insipidus has a deficiency of ADH with excessive loss of water from the kidney, hypovolemia, hypernatremia, and dilute urine with a low specific gravity. When vasopressin is administered, the symptoms are reversed, with water retention, decreased urinary output that increases urine osmolality, and an increase in BP.

Priority Decision: During evaluation and treatment of gonorrhea in a young man at the health clinic, what is most important for the nurse to question the patient about? a. A prior history of STIs b. When the symptoms began c. The date of his last sexual activity d. The names of his recent sexual partners

d. All sexual contacts of patients with gonorrhea must be notified, evaluated, and treated for STIs. The other information may be helpful in diagnosis and treatment but the nurse must try to identify the patient's sexual partners.

Priority Decision: When replacement therapy is started for a patient with long-standing hypothyroidism, what is most important for the nurse to monitor the patient for? a. Insomnia c. Nervousness b. Weight loss d. Dysrhythmias

d. All these manifestations may occur with treatment of hypothyroidism. However, as a result of the effects of hypothyroidism on the cardiovascular system, when thyroid replacement therapy is started myocardial oxygen consumption is increased and the resultant oxygen demand may cause angina, cardiac dysrhythmias, and heart failure, so monitoring for dysrhythmias is most important.

What is a cause of primary hypothyroidism in adults? a. Malignant or benign thyroid nodules b. Surgical removal or failure of the pituitary gland c. Surgical removal or radiation of the thyroid gland d. Autoimmune-induced atrophy of the thyroid gland

d. Both Graves' disease and Hashimoto's thyroiditis are autoimmune disorders that eventually destroy the thyroid gland, leading to primary hypothyroidism. Thyroid tumors most often result in hyperthyroidism. Secondary hypothyroidism occurs as a result of pituitary failure and iatrogenic hypothyroidism results from thyroidectomy or radiation of the thyroid gland.

What findings are commonly found in a patient with a prolactinoma? a. Gynecomastia in men b. Profuse menstruation in women c. Excess follicle-stimulating hormone (FSH) and luteinizing hormone (LH) d. Signs of increased intracranial pressure, including headache, nausea, and vomiting

d. Compression of the optic chiasm can cause visual problems as well as signs of increased intracranial pressure, including headache, nausea, and vomiting. About 30% of prolactinomas will have excess prolactin secretion with manifestations of impotence in men, galactorrhea or amenorrhea in women without relationship to pregnancy, and decreased libido in both men and women. There is decreased follicle-stimulating hormone (FSH) and luteinizing hormone (LH).

During the physical assessment of a female patient with HPV infection, what should the nurse expect to find? a. Purulent vaginal discharge b. A painless, indurated lesion on the vulva c. Painful perineal vesicles and ulcerations d. Multiple coalescing gray warts in the perineal area

d. HPV is responsible for causing genital warts, which manifest as discrete single or multiple white to gray warts that may coalesce to form large cauliflower-like masses on the vulva, vagina, cervix, and perianal area. Purulent vaginal discharge is associated with gonorrhea or chlamydia. Painful perineal vesicles and ulcerations are characteristic of genital herpes and a chancre of syphilis is a painless indurated lesion on the vulva, vagina, lips, or mouth.

A patient with Graves' disease asks the nurse what caused the disorder. What is the best response by the nurse? a. "The cause of Graves' disease is not known, although it is thought to be genetic." b. "It is usually associated with goiter formation from an iodine deficiency over a long period of time." c. "Antibodies develop against thyroid tissue and destroy it, causing a deficiency of thyroid hormones." d. "In genetically susceptible persons, antibodies are formed that cause excessive thyroid hormone secretion."

d. In Graves' disease, antibodies to the TSH receptor are formed, attach to the receptors, and stimulate the thyroid gland to release triiodothyronine (T3), thyroxine (T4), or both, creating hyperthyroidism. The disease is not directly genetic but individuals appear to have a genetic susceptibility to develop autoimmune antibodies. Goiter formation from insufficient iodine intake is usually associated with hypothyroidism.

What characteristic is related to Hashimoto's thyroiditis? a. Enlarged thyroid gland b. Viral-induced hyperthyroidism c. Bacterial or fungal infection of thyroid gland d. Chronic autoimmune thyroiditis with antibody destruction of thyroid tissue

d. In Hashimoto's thyroiditis, thyroid tissue is destroyed by autoimmune antibodies. An enlarged thyroid gland is a goiter. Viral-induced hyperthyroidism is subacute granulomatous thyroiditis. Acute thyroiditis is caused by bacterial or fungal infection.

A patient who recently had a calcium oxalate renal stone had a bone density study, which showed a decrease in her bone density. What endocrine problem could this patient have? a. SIADH c. Cushing syndrome b. Hypothyroidism d. Hyperparathyroidism

d. The patient with hyperparathyroidism may have calcium nephrolithiasis, skeletal pain, decreased bone density, psychomotor retardation, or cardiac dysrhythmias. The other endocrine problems would not be related to calcium kidney stones or decreased bone density.

When providing discharge instructions to a patient who had a subtotal thyroidectomy for hyperthyroidism, what should the nurse teach the patient? a. Never miss a daily dose of thyroid replacement therapy. b. Avoid regular exercise until thyroid function is normalized. c. Use warm saltwater gargles several times a day to relieve throat pain. d. Substantially reduce caloric intake compared to what was eaten before surgery.

d. With the decrease in thyroid hormone postoperatively, calories need to be reduced substantially to prevent weight gain. When a patient has had a subtotal thyroidectomy, thyroid replacement therapy is not given because exogenous hormone inhibits pituitary production of TSH and delays or prevents the restoration of thyroid tissue regeneration. Regular exercise stimulates the thyroid gland and is encouraged. Saltwater gargles are used for dryness and irritation of the mouth and throat following radioactive iodine therapy.

Which STI actively occurring at the time of delivery would indicate the need for a cesarean section delivery of the woman's baby? a. Syphilis b. Chlamydia c. Gonorrhea d. Genital herpes

d. Women with an active herpes simplex virus (HSV) genital lesion at the time of delivery have the highest riskof transmitting genital herpes to the neonate, so delivery will be done with a cesarean section (C-section). Syphilis is spread to the fetus in utero and has a high risk of stillbirth but C-sections are not required. Treatment with parenteral penicillin will cure both the mother and the fetus. Chlamydia spread to the fetus can be prevented by treating the pregnant woman, so a C-section is not required. Prevention of the spread of gonorrhea to the neonate's eyes is done with erythromycin ophthalmic ointment or silver nitrate aqueous solution.


संबंधित स्टडी सेट्स

N174 Physical Assessment Objectives

View Set

ATI RN Maternal Newborn Online Practice 2019 A with NGN

View Set

chapter 11 case study low serum vitamin D

View Set

Chapter 1: Introduction to Operations Management (multiple choice) & Chapter 2: Competitiveness, Strategy, and Productivity

View Set

Exam 1 Stroke Lippincott practice questions

View Set

Chapter 5: Infection Control (Standard Foundations)

View Set

AP Biology Unit 5 Progress Check

View Set